1. Trang chủ
  2. » Lịch sử

Tạp chí Epsilon số 8

191 26 0

Đang tải... (xem toàn văn)

Tài liệu hạn chế xem trước, để xem đầy đủ mời bạn chọn Tải xuống

THÔNG TIN TÀI LIỆU

Thông tin cơ bản

Định dạng
Số trang 191
Dung lượng 17,31 MB

Nội dung

Từ điển Tiểu sử Khoa học (Dictionary of Scientific Biography) mở đầu bài viết dài về Euclid bằng những lời này: “Mặc dù Euclid là nhà toán học trứ danh nhất mọi thời đại, là người mà tên[r]

(1)

“Nếu hai vật thể có đặc số Euler khác nhau, chúng khơng thể biến thành sau phép biến đổi thuận nghịch liên tục (kiểu co dãn cao su) Người ta nói hai vật thể khơng kiểu tơpơ.” RỘNG HẸP NHỎ TO VỪA VẶN CẢ

(GIỚI THIỆU TÔPÔ HỌC) Nguyễn Hữu Việt Hưng

“Cơ sở hình học coi sách có ảnh hưởng tới phát triển văn minh nhân loại.”

EUCLID VÀ CƠ SỞ CỦA HÌNH HỌC No 8

TẬP TRÁNH TỔNG TRONG NHÓM

Terence Tao

VỀ HẰNG SỐ LIÊN THƠNG TRÊN LƯỚI TỔ ONG

Huỳnh Cơng Bằng

CƠ SỞ

GRƯBNER LÀ GÌ?

Bernd Sturmfels

EUCLID VÀ

CƠ SỞ CỦA HÌNH HỌC

Ngơ Bảo Châu, Richard Fitzpatrick

LÊ VĂN THIÊM

CON NGƯỜI VÀ SỰ NGHIỆP

Hà Huy Khoái

(2)

Trần Nam Dũng

BIÊN TẬP VIÊN:

Võ Quốc Bá Cẩn Ngô Quang Dương Trần Quang Hùng Nguyễn Văn Huyện Nguyễn Tiến Lâm Lê Phúc Lữ Nguyễn Tất Thu Đặng Nguyễn Đức Tiến

No 8

(3)

LỜI NGỎ CHO EPSILON SỐ 8

Ban Biên tập Epsilon

Epsilon có số này? Ban biên tập xin mời bạn đọc lướt qua phần giới thiệu chúng tơi

Mảng lịch sử tốn học có GS Hà Huy Khối viết thân nghiệp GS Lê Văn Thiêm mà vừa kỷ niệm 98 năm ngày sinh tháng vừa qua Cũng mảng lịch sử toán học có nội dung giới thiệu tơ-pơ có viết hóm hỉnh, dễ hiểu sâu sắc GS Nguyễn Hữu Việt Hưng với tựa đề "Rộng hẹp nhỏ to vừa vặn cả" Các bạn thấy bất ngờ thấy hóa với Leonard Euler vĩ đại, có nhà thơ Việt Nam tìm ý tưởng táo bạo tơ-pơ (có dẫn tựa đề) Cùng chủ đề giới thiệu tốn cao cấp có Matt Baker định lý hôn nhân, sở Grobner Bernd Sturmfels Epsilon số giới thiệu viết Terence Tao blog anh giới thiệu báo anh Vũ Hà Văn tập tránh tổng (Sum-free sets) Mục điểm sách có GS Ngơ Bảo Châu Richard Fitzpatrick giới thiệu Euclid Cơ sở hình học Mục vấn vấn đề cổ điển đại có đề thi Tú tài Pháp GS Nguyễn Tiến Dũng giới thiệu

Bên cạnh chuyên mục quen thuộc toán sơ cấp, lời giải số trước, viết tác giả "ruột" Epsilon, toán hay, lời giải đẹp, tốn giải trí

Chúng tơi đặc biệt muốn nhấn mạnh đến ủng hộ tác giả dành cho Epsilon Điều nói lên tinh thần "Khi đóng góp cộng động, cộng đồng ta" GS Hà Huy Khối đích thân gửi đến cho Ban biên tập GS Nguyễn Hữu Việt Hưng, xin vui vẻ gửi gốc để tiện biên tập với lời nhắn "với điều kiện cậu không sửa, dù chữ" Cịn Terence Tao trả lời ngắn gọn "This is fine with me Best, Terry" Các tác giả "ruột" Ngô Quang Hưng, Lý Ngọc Tuệ dù bận bịu ln gắng dành thời gian viết cho Mà anh viết chuẩn, BBT cần ráp vào xong Các bạn trẻ hăng hái viết (số có bạn học sinh Hoàng Cao Phong, Nguyễn Trần Hữu Thịnh Đỗ Xuân Anh), sẵn sàng nhận dịch nhờ (số có dịch bạn Nguyễn Vũ Anh)

Mà khơng có bạn trẻ sung Lứa trung niên sung Số có Trịnh Đào Chiến anh Trần Minh Hiền hứa viết định lý Cauchy Davenport cho số Lão ngoan đồng Nguyễn Vũ Duy Linh hăng hái dịch sở Grobner Sturmfels Ban biên tập cảm ơn ủng hộ tác giả đón nhận nồng nhiệt độc giả Hãy tiếp tục chung tay sát cánh đường đầy gian nan phía trước

(4)

Ban Biên tập Epsilon

Lời ngỏ cho Epsilon số Lý Ngọc Tuệ

Xấp xỉ Diophantine trênRn- Véc tơ xấp xỉ Trò chơi siêu phẳng tuyệt đối

Bernd Sturmfels

C s Grăobner l gỡ? 21 Terence Tao

Tập tránh tổng nhóm 25 Matt Baker

Tốn học nhân 29 Huỳnh Công Bằng

Về số liên thông lưới tổ ong 33 Nguyễn Hữu Việt Hưng

Rộng hẹp nhỏ to vừa vặn (Giới thiệu Tôpô học) 47 Đặng Nguyễn Đức Tiến

Các toán đoán 55 Nguyễn Trần Hữu Thịnh

Xung quanh định lý Brokard 61 Đỗ Xuân Anh

Tứ giác ngoại tiếp đường tròn 75 Trần Quang Hùng, Nguyễn Tiến Dũng

(5)

Tạp chí Epsilon, Số 08, 04/2016

Nguyễn Tiến Lâm, Ngơ Quang Dương

Tính chất hình học đường cong bậc ba 119 Hoàng Cao Phong

Biểu diễn số nguyên dương dạng tổng số phương 125 Trịnh Đào Chiến

Một số dạng toán bất phương trình hàm 133 Trần Nam Dũng

Bài toán hay lời giải đẹp 151 Ngô Bảo Châu, Richard Fitzpatrick

Euclid Cơ sở hình học 155 Hà Huy Khoái

Lê Văn Thiêm: Con người nghiệp 161 Trần Nam Dũng

Các vấn đề cổ điển đại 173 Ban Biên tập Epsilon

(6)(7)

XẤP XỈ DIOPHANTINE TRÊN Rn- VÉC TƠ XẤP XỈ KÉM VÀ TRÒ CHƠI SIÊU PHẲNG

TUYỆT ĐỐI

Lý Ngọc Tuệ

(Đại học Brandeis, Massachusetts, Mỹ)

1 Giới thiệu

Trong phần phần loạt xấp xỉ Diophantine [16,17] chứng minh Định lý Dirichlet trênRnnhư sau:

Định lý 1 (Dirichlet). Với véc tơ vô tỉ xE Rn XQn, tồn vô số véc tơ hữu tỉ pE

q D

p1 q ; :::;

pn q

2 QnvớipE2 Znvq 2Z,q Ô0sao cho:

E

x pE q < jqj1C1n

:

Tổng quát tí, gọi hàm số liên tục không tăng WR>0 !R>0là hàm

xấp xỉ, gọi véc tơxE 2Rnlà -xấp xỉ được1nếu tn ti vụ spE2Zn,q Z,q Ô0

sao cho:

E

x pE q

jqj/ jqj :

Tập véc tơ -xấp xỉ trênRnsẽ ký hiệu làWAn / Nếu ta sử dụng ký hiệu:

˛Wk 7!k ˛, Định lý Dirichlet phát biểu lại thành:

WAn

1 n

DRn:

Hàm số gọi hàmDirichlet(trênRn) nhưWAn /DRn

Câu hỏi hàm số Dirichlet tối ưu cho Rn trả lời phần Định luật 0-1 sau Khintchine:

Định lý 2(Khintchine 1926). Ký hiệulà độ đo Lebesgue trênRn:

(i) Nếu chuỗi

X

kD1

k/nhội tụ thì.WAn //D0

(ii) Nếu chuỗi

X

kD1

k/nphân kỳ thì.RnXWAn // D0

1

(8)

Với > bất kỳ, theo Định lý 2, WA nC

D Vì vậy,

nC hàm

Dirichlet, số mũ

1C n

trong Định lý tối ưu Tuy nhiên, với hàm số tiến nhanh tí nhưk7!k 1n.logk/

1 n

hayk 7!k n1.log logk/

, Định lý cho ta biết có phải hàm Dirichlet hay không Thật hàm hàm Dirichlet được, hay tổng quát nữa, hàm số thỏa mãn:

lim

k!1k

n k/D0;

thì khơng phải hm Dirichlet trờnRn:WAn /ÔRn

iu ny cú th c chứng minh cách tồn véc tơxấp xỉ kémđược định nghĩa sau:xE 2Rnđược gọi xấp xỉ tồn sốc > 0(tùy thuộc vào

E

x) cho vi mipE2Zn,q 2Z,q Ô0:

E

x pE q > c jqj1C1n

: (1.1)

Tập véc tơ xấp xỉ trênRnsẽ ký hiệu bởiBAn

Khin D 1, số xấp xỉ tương ứng với liên phân số đơn bị chặn, BA1 khơng

rỗng Theo Định lý Lagrange, số thực˛ số đại số bậc mở rộng liên phân số của˛ tuần hoàn, số thực đại số bậc vô tỉ xấp xỉ Tuy khơng có cơng cụ liên phân số khin2, chứng minh trực tiếp mở rộng quan sát choRnnhư sau:

Định lý 3. Nếu nhưf1; ˛1; :::; ˛nglà sở trường số đại số thực2 bậc.nC1/, E

˛ D.˛1; :::; ˛n/2 BAn

Bài tập 4. Chứng minh Định lý

Ví dụ có vơ hạn đếm véc tơ xấp xỉ trênRn Mãi đến năm 1954, Davenport [5] chứng minh rằngBA2là tập không đếm được, năm sau đấy,

Cassels [4] chứng minh rằngBAnlà không đếm vớinbất kỳ

Vậy tậpBAnlớn nào? Phân tíchBAnra thành sau:

BAnD

[

c>0

RnXWA

c n

D

[

kD1

RnXWA

k 1 n

;

và áp dụng Định lý 2, ta có

.BAn/D0:

Nói cách khác, theo độ đo Lebesgue thìBAnlà tập nhỏ không đáng kể Hơn nữa,

cách phân tích cịn rằngBAnthuộc phạm trù thứ theo Baire, hội đếm

được tập không đâu trù mật

Một cơng cụ phổ biến để đo kích cỡ tập nhỏ làchiều Hausdorff, ký hiệu dim (xem thêm chi tiết ở2) Sử dụng cách biểu diễn số xấp xỉ dạng liên phân số bị chặn, Jarník [11] chứng minh tập số xấp xỉ BA1có chiều Hausdorff

bằng Đến 1966, Schmidt [21] mở rộng kết cho véc tơ xấp xỉ kém:

2

(9)

Tạp chí Epsilon, Số 08, 04/2016

Định lý 5(Schmidt 1966). dimBAnDn

Schmidt chứng minh kết dựa vào phương pháp hồn tồn mà ơng nghĩ ra: sử dụng trị chơi vơ hạn với thơng tin hồn hảo mà sau gọi trị chơi Schmidt (xem [19, Phần 4]) Áp dụng trò chơi này, Schmidt chứng minh nhưy1;E y2; :::E dãy véc tơ trênRn, giao tịnh tiến củaBAnbởiy1;E y2; :::E có chiều Hausdorff bằngn:

dimH

\

kD1

BAnC Eyk

!

Dn:

Tổng quát hơn, Schmidt chứng minh rằng:

Định lý 6(Schmidt 1966). Gọi U là tập mở trênRn,ffi WU !Vig1iD1 là họ đếm hàm3từU vào tập mởVi Rn.

dim

1

\

iD1

.fi/ 1.BAn/

!

Dn:

Dựa ý tưởng Schmidt, McMullen [20] giới thiệu biến thể trò chơi Schmidt, gọi trò chơituyệt đối4, chứng minh tậpBA1 tập thắng trò chơi

(thắng tuyệt đối5) Tuy nhiên, khin2,BAnkhông phải tập thắng tuyệt đối

Vì Broderick, Fishman, Kleinbock, Reich, Weiss [1] mở rộng ý tưởng McMullen giới thiệu trò chơi siêu phẳng tuyệt đối, tập thắng gọi thắng siêu phẳng tuyệt đối6, viết tắt HAW Áp dụng trò chơi này, BFKRW làm mạnh Định lý Schmidt sau:

Định lý 7(BFKRW 2012). Gọi U là tập mở trênRn,ffi WU !Vig1iD1 là họ đếm vi phôi7C1từU vào tập mởVi Rn.

dim

1

\

iD1

.fi/ 1.BAn/

!

Dn:

Định lý Định lý hệ Định lý sau:

Định lý 8(BFKRW 2012) BAnlà tập thắng siêu phẳng tuyệt đối.

Trong phần cịn lại này, chúng tơi giới thiệu chi tiết chiều Hausdorff, trò chơi siêu phẳng tuyệt đối, chứng minh Định lý

2 Chiều Hausdorff

Một số tài liệu tham khảo cho chiều độ đo Hausdorff: Fractal Geometry: Mathematical Foundations and Applications[6] vàThe Geometry of Fractal Sets[7] K J Falconer

3 uniformly bi-Lipschitz absolute game absolute winning

hyperplane absolute winning 7C1

(10)

Với tập khơng rỗngU Rn,đường kính củaU định nghĩa khoảng cách lớn

nhất điểm trongU:

diamU WDsup˚xE yE

W Ex;yE2U :

Nếu nhưfUiglà họ đếm tập có đường kính khơng qıvàE

[

iD1

Ui, ta gọi

fUiglà mộtı-phủ8củaE.

Với tậpE Rn, với mỗis; ı > 0,độ đo ngoài.ı; s/Hausdorff củaE định nghĩa

là:

Hs

ı E/WDinf

( 1 X

iD1

.diamUi/s W fUiglà mộtı-phủ củaE

)

:

Bài tập 9. Chứng minh rằngHıslà độ đo ngồi, nghĩa thỏa mãn tính chất sau: (i) Hıs.;/D0

(ii) AB H)Hıs.A/ H s ı B/

(iii) Hıs

1

[

iD1 Ai

!

X

iD1

H s

ı Ai/

Khi ıgiảm dần 0, lớp ı-phủ củaE nhỏ đi, nênHıs.E/tăng dần giới hạn

H s

ı E/khiı ! 0tồn (có thể làC1) Ta gọi giới hạn độ đo Hausdorff với chiềus9củaE:

H s.E/

WD lim

ı&0H s ı E/:

Theo lý thuyết độ đo tổng quát, ta giới hạn vào tậpHs-đo được,H strở thành độ đo Hơn nữa, tập Borel trênRnđều làH s-đo với mọis >

Độ đo Hausdorff có số tính chất sau:

Bổ đề 10. ChoE Rn.

(i) Khis Dn, độ đo Hausdorff với chiềuntương đương với độ đo Lebesgue trênRn: Tồn tại

một sốc > 0sao cho với tập BorelE,

H n

.E/Dc.E/: (ii) Với˛ > 0, ký hiệu˛E D˚

˛xE W Ex 2E Độ đo Hausdorff với chiềuscủa˛E thỏa mãn:

H s

.˛E/D˛sH s.E/:

(iii) Tổng quát hơn, nhưf WE !Rmlà hàm cho tồn sốc; ˛ > 0để với

mọix;E yE2E:

f x/E f y/E c

xE yE

˛ ; thì với mọis > 0:

H s=˛

.f E//cs=˛Hs.E/:

ı-cover 9s

(11)

Tạp chí Epsilon, Số 08, 04/2016

(iv) Nếu nhưHs.E/ < 1, với mọit > s,H t.E/D0 (v) Nếu nhưHs.E/ > 0, với mọi0 < t < s,H t.E/D

Bài tập 11. Chứng minh Bổ đề 10

Tính chất (iv) (v) Bổ đề 10 cho thấy có thời điểms D s0 mà H s.E/ nhảy từ1 xuống 0, gọi làchiều Hausdorff củaE:

dimH.E/WDsupfs > 0WH s.E/D 1g Dinffs > WHs.E/D0g:

Một số tính chất chiều Hausdorff sau:

Bổ đề 12. ChoE Rn.

(i) Nếu0 <H s.E/ <1, thìdimH.E/Ds (ii) NếuE là tập mở củaRnthìdimH.E/Dn (iii) NếuE F thìdimH.E/dimH.F /

(iv) NếuE là đa tạpm-chiều trongRnthìdimH.E/Dm (v) Với dãyfEig:

dimH

[

iD1 Ei

!

Dsup

i

dimH.Ei/:

Bài tập 13. Chứng minh Bổ đề 12

Chúng ta thấy chiều Hausdorff công cụ quan trọng để mô tả tập có độ đo Lebesgue khơng đáng kể thơng qua ví dụ tiếng tập Cantor

Ví dụ 14. Tập Cantor10 C định nghĩa theo bước sau ĐặtC0 D Œ0; 1 Ở bước thứk1, tậpCk D[

i

Ik;i hợp đoạn không giao cặp,

CkC1sẽ cách bỏ đoạn mở cácIk;i có độ dài dúng 1/3 độ dài Ik;i Cụ thể hơn, ta có được:

C0DŒ0; 1 C1D 0;1 [ 3;

C2D 0;1 [ 2 9; [ 2 3; [ 8

9;

:: :

Tập CantorC giao tất tậpCk

C D

1

\

kD0 Ck: 10

(12)

0

0 1=3 2=3

0 1=9 2=9 1=3 2=3 7=9 8=9

C0 C1 C2

Có thể thấy rằngCk bao gồm2k đoạn thẳng có độ dài3 k,

C0C1 C2 C:

Với mọik 0:

.C/.Ck/D

2

k

:

Từ ta suy rằngC có độ dài (độ đo Lebesgue trênR)

VìCk tập compact,C tập compact khơng rỗng Ta mô tả phần tử củaC sau Với số thực0x 1, viếtxbằng hệ số 3:

x D.0:a1a2:::/3 D

X

iD1

ai3 i; a1; a2; :::2 f0; 1; 2g:

Khi đấy:

x 2C () a1; a2; :::2 f0; 2g:

Chúng ta chứng minh vớis D log2

log3,H s

.C/D1 Vì tậpC có chiều Hausdorff log2

log3

ĐặtCk D 2k

[

iD1

Ik;i, đấyIk;i đoạn đóng có độ dài3 k, với mỗiı > 0, chọnkđủ lớn

sao cho3 k ı Khi đấyfIk;iglà mộtı-phủ củaC, ta có được:

H s

ı C/ 2k

X

iD1

.diam.Ik;i//sD 2k

X

iD1

3 k log2 log3

D1:

Lấy giới hạn khiı&0:

Hs

.C/D lim

ı&0H s

ı C/1:

Để chứng minh chiều ngược lại, gọifU˛glà phủ củaC Không tính tổng qt, giả sử rằngU˛là đoạn thẳng đóng VìC tập compact, ta tìm số hữu hạn đoạn˚Uj 1jmphủC

Gọiklà số nguyên dương nhỏ cho với mọi1i 2k với mọi1j m, phần củaIk;i giao vớiUj thìIk;i Uj GọiIj tập đoạnIk;i nằm trongUj:

Ij WD

˚

(13)

Tạp chí Epsilon, Số 08, 04/2016

vàUj0 đoạn đóng nhỏ chứa đoạnIk;i trongIj

Ta dễ dàng kiểm tra rằng˚Uj0 1

jmcũng phủ củaC, và: m

X

jD1

diamUjs m

X

jD1

diamUj0

s

:

Nếu nhưUj0 chứa đoạnIk;i hiển nhiên:Uj0 DIk;i Cịn khi: 2l <#Ij 2lC1;

ta tìm đoạn đóngK Uj0 cho: (i) Ko\Ck D ;,

(ii) diamK

3diamU j,

(iii) Uj0 XKobao gồm đoạn đóngJ vàJ0, đoạn chứa nhiều nhất2l đoạn conIk;i

Ij

Từ ta có được: diamUj0s

D diamJ CdiamKCdiamJ0s

3

2 diamJ CdiamJ

s

D2

1

2diamJ C

2diamJ

s

s D log2

log3

2

1

2.diamJ / s

C

2 diamJ 0s

.0 < s < 1/ D.diamJ /sC diamJ0s

Quy nạp theol, ta có được:

diamUj0s

X

Ik;i2Ij

.diamIk;i/s:

Vì˚Uj0 phủ củaC:

1

X

iD1

.diam.Ui//s m

X

jD1

diamUij

s

2k

X

iD1

.diamIk;i/sD1:

VậyH s.C/D1và dimH.C/D

log2

(14)

3 Trò chơi siêu phẳng tuyệt đối

Trong phần này, ký hiệuB.x; r/E ‘quả bóng’11 đóng trongRnvới tâmxE bán kínhr:

B.Ex; r/ WD˚

E

y 2RnWxE yE r :

Một siêu phẳng12LtrongRnlà tập hợp nghiệm hàm tuyến tínhnẩn khác Khoảng cách từ điểmxEđếnLđược định nghĩa là:

dist x;E L

WDinf˚xE yE

W Ey 2L :

Tập hợp điểm có khoảng cách đếnLkhơng qr gọi mộtr-lân cận củaL, ký hiệu là:

L.r/ WD˚

E

x Wdist x;E L

r :

Cho trước số0 < ˇ <

3 tập đối S R n

, trò chơiˇ-siêu phẳng tuyệt đối An Bình diễn sau:

1 An Bình thay phiên đi, Bình người trước

2 Đầu tiên Bình chọn bóng bất kỳB1DB.x1; r1/E với bán kínhr1 > Ở bước thứi 1, An chọnsi-lân cận siêu phẳngLi cho0 < si ˇri

4 Ở bước thứi C1, Bình chọn bóngBiC1DB.xiEC1; riC1/sao choriC1 ˇri và:

B.xiEC1; riC1/B.xiE ; ri/XL.si/i :

L.s1/1

E

x1 r1

B1

E x2 r2

B2

An thắng như:

S \

\

iD1

B.xiE ; ri/Ô ;; 11

vỡ chỳng ta dựng sup-norm, nên thật raB.x; r/E hình hộp vng trongRn 12

(15)

Tạp chí Epsilon, Số 08, 04/2016

cịn khơng Bình thắng TậpS gọi tậpˇ-thắng siêu phẳng tuyệt đối

nếu An có chiến lược để ln ln thắng trò chơi ˇ-siêu phẳng tuyệt đối (viết tắt làˇ-HAW) Bình có nữa.S gọi làthắng siêu phẳng tuyệt đối(viết tắt làHAW) nhưS ˇ-thắng siêu phẳng tuyệt mọi0 < ˇ <

3

Lưu ý 15. Khi nD 1, ‘siêu phẳng’L trênRđơn giản điểm Khi đấy, trò chơi siêu phẳng tuyệt đối gọi trò chơi tuyệt đối giới thiệu McMullen [20]

Lưu ý 16. Trò chơi siêu phẳng tuyệt đối chơi không gian metric tổng quát.X;dist/mà siêu phẳngLcó thể thay tập đóng cho trước trongX Trò chơi tổng quát gọi trò chơiH-tuyệt đối13 giới thiệu Fishman, Simmons, Urbanski [9], phát triển áp dụng [14]

Lưu ý 17. Điều kiệnˇ < 1=3là để An có chọn nữa, Bình ln có lựa chọn hợp lệ cho bước Ta chơi trị chơi siêu phẳng tuyệt đối tập conX Rnvới lựa chọn Bình có tâm nằm trongX điều kiện sau

thỏa mãn: Tồn tại; r0 > 0đủ nhỏ cho với bóngB.Ex; r/có tâmxE 2X bán kính

0 < r < r0và với mi siờu phngL,

X\B.x; r/E XL r/Ô ;:

Điều kiện đảm bảo khiˇđủ nhỏ, trò chơiˇ-siêu phẳng tuyệt đối trênX kéo dài vô hạn Những tậpX thỏa mãn điều kiện gọi là-siêu phẳng phân tán14.X gọi

siêu phẳng phân tánnếu tồn > 0sao choX là-siêu phẳng phân tán Ví dụ: tậpRnlà

1

3-siêu phẳng phân tán, đường thẳng không gian chiềuR

không phải tập siêu phẳng phân tán

Lưu ý 18. Nếu nhưX Rlà tập siêu phẳng phân tán, ta chơi trò chơi siêu phẳng tuyệt

đối trênX cách bắt Bình phải chọn bóng có tâm nằm trongX Khi tập thắng gọi làHAW trênX

Một số tính chất quan trọng tập thắng trò chơi siêu phẳng tuyệt đối sau:

Định lý 19([1]). Giả sử nhưX Rlà tập siêu phẳng phân tán.

(i) NếuS Rlà tập HAW thìdimH.S /Dn

(ii) NếuS là tập HAW trênX, vàY X là tập siêu phẳng phân tán, thìSHAW trên Y.

(iii) NếuS1; S2; :::là tập HAW trênX thì

\

iD1

Si cũng tập HAW trênX

(iv) Giả sử nhưf WRn !Rnlà vi phôiC1, vàS là tập HAW, thìf S /cũng một

tập HAW. 13H

-absolute game 14

(16)

Ý tưởng chứng minh phần (i) Định lý 19 xây dựng trongS tập giống tập Cantor15như sau: bước, ta chia nhỏ ‘quả bóng’B.xiE ; ri/thành bóng có phần đơi khơng giao với bán kínhˇri bỏ bóng giao với.ˇri/-lân cận siêu phẳngLi chiến lược thắng An Tập giống Cantor nằm tập

HAWS cho chặn chiều Hausdorff củaS, chặn tiến

nkhiˇtiến Bạn đọc xem thêm chứng minh đầy đủ [3, Định lý 2.2]

Bài tập 20. (a) Chứng minh tập Cantor trênRlà

9-siêu phẳng phân tán

(b) Có thể thay số

9 số khác lớn hay k0? 4 Véc tơ xấp xỉ kém

Áp dụng Định lý 19, ta dễ dàng có Định lý suy Định lý Để chứng minh Định lý 8, dùngBổ đề Đơn hình16

Bổ đề Đơn hình mở rộng quan sát sau trênR: Chok > 1, p

q p0

q0 số hữu tỉ

khác với mẫu sốki q; q0< kiC1, thì:

ˇ ˇ ˇ ˇ p q p0 q0 ˇ ˇ ˇ ˇD ˇ ˇ ˇ ˇ

pq0 p0q qq0 ˇ ˇ ˇ ˇ qq0 > k

2i :

Như đoạn thẳng trênRcó bán kính0 < r < 2k

2i

có chứa nhiều số hữu tỉ

p q vớik

i

q; q0< kiC1

Cho.nC1/điểmx0;E x1;E x2; :::;E xnE , chonvéc tơ.x1E x0/; E x2E x0/; :::; E xnE x0/E độc lập tuyến tính.Đơn hình với đỉnh˚x0;E x1; :::;E xnE định nghĩa là:

.x0; :::;E xn/E WD˚

E

x0Ca1.x1E x0/E C:::Can.xnE x0/E Wa1; :::; an0; a1C:::Can :

KhinD1,.x0; x1/là đoạn thẳng nốix0vàx1 KhinD2,.x0;E x1;E x2/E hình tam giác với đỉnhx0;E x1;E x2E KhinD3,.Ex0; :::;x3/E tứ diện với đỉnhx0; :::;E x3E

Thể tích đơn hình tính đơn giản sau:

.x0; :::;E xn/E

D nŠ

ˇ

ˇdet x1E x0;E x2E x0; :::;E xnE x0E ˇ

ˇ: Bổ đề 21(Bổ đề Đơn hình [15, Bổ đề 4]). Cho0 < ˇ <

3 Với mỗik2 N, gọiUk là tập các véc tơ hữu tỉ có mẫu nằm giữaˇnCn1.k 1/ˇ

n nC1k:

Uk WD

E p

q W Ep 2Z

n; ˇnCn1.k 1/

q < ˇnCn1k

(17)

Tạp chí Epsilon, Số 08, 04/2016

ĐặtVnlà thể tích bóng đơn vị trongRn Với mọi:

0 < r < ˇ.nŠVn/ n1 và với mọixE 2Rn, tồn siêu phẳngLk sao cho:

Uk\Bx; ˇE k 1rLk:

Bài tập 22. Gọi p0E

q0; E p1 q1; :::;

E pn

qn là.nC1/điểm hữu tỉ trongUk chonvéc tơ

E p1 q1 E p0 q0 ; E p2 q2 E p0 q0 ; , E p2 q2 E p0 q0

độc lập tuyến tính Tìm cận

 E p0 q0; E p1 q1; :::;

E pn qn

Bài tập 23. Chứng minh Bổ đề Đơn hình 21

Chứng minh Định lý 8. Cho0 < ˇ <

3 cố định Xét trò chơiˇ-siêu phẳng tuyệt đối

RvớiBAnlà tập đối tượng An Lưu ý rằngBAntrù mật, nên bán kính

lựa chọn Bình khơng hội tụ 0, An thắng An bán kính bóng Bình chọn nhỏ hơnˇ.nŠVn/ n1 Vì ta giả sử rằngB1 D B.x1; r1/E với r1 < ˇ.nŠVn/ 1n, lim

i!1ri D0

Đặtc Dˇ2r1 Với mỗik2N, gọiik lượt mà:

ˇk 1r1 rik > ˇkr1:

Với lượt không nằm dãyfikg1kD1, An Ở bước thứik, theo Bổ đề

Đơn hình 21, tồn siêu phẳngLk cho:

Uk \B.xkE ; rik/Lk:

Bước lượt thứik An

ˇkC1r1-lân cận củaLk Vì BikC1 Bij XL

ˇkC1r1/ k ;

với mọiyE2BikC1và với E p q 2Uk:

E

y pE q

ˇkC1r1 Dcˇk c q1C1n

:

Vì:

1

[

kD1

Uk DQn;

theo định nghĩa củaBAn(1.1),

\

iD1

B.xiE ; ri/D

\

kD1

B.xiEk; rik/2BAn:

Vì vậy, An có chiến lược kết

1

\

iD1

(18)

Lưu ý 24. Một hệ thú vị kết tập nhỏ tập CantorC tập số xấp xỉ kémBA1vẫn giao nhau, phần giao không nhỏ:

dimH.BA1\C/DdimH.C/D

log2

log3:

Không thế, với dãy sốa1; a2; :::, dịch chuyển củaBA1bởi giao với tập

Cantor:

dimH C \

\

iD1

.BA1Cai/

!

D log2

log3:

Kết chứng minh Fishman [8] sử dụng trò chơi Schmidt

Tài liệu tham khảo

[1] R Broderick, L Fishman, D Kleinbock, A Reich, B Weiss,The set of badly approx-imable vectors is stronglyC1-incompressible, Math Proc Cambridge Philos Soc.153, no (2012), pp 211–253

[2] R Broderick, L Fishman, D Simmons,Badly approximable systems of affine forms and incompressibiblity on fractals, J Number Theory133(2013), pp 2186–2205

[3] R Broderick D Kleinbock,Dimension estimates for sets of uniformly badly approx-imable systems of linear forms, preprint (2013), arXiv:1311.5474

[4] J W S Cassels, Simultaneous Diophantine approximation II, Proc Lon Math Soc.3

(1955), pp.435–448

[5] H Davenport,Simultaneous Diophantine approximation, Mathematika1(1954), pp 51–72 [6] K J Falconer,Fractal Geometry: Mathematical Foundations and Applications (1990),

John Wiley & Sons

[7] K J Falconer,The geometry of fractal sets, Cambridge Tracts in Math.85(1986), Cam-bridge Univ Press

[8] L Fishman,Schmidt’s game on fractals, Israel J Math.171(2009), pp 77–92

[9] L Fishman, D Simmons, M Urbanski,Diophantine approximation and the geometry of limit sets in Gromov hyperbolic metric spaces, đăng Mem Amer Math Soc., arXiv:1301.5630

[10] D Gale F M Stewart,Infinite games with perfect information, in: Contribution to the theory of games, Vol II, Annals of Math Studies28(1953), pp 245–266

[11] V Jarník,Diophantische approximationen und hausdorffsches mass, Recueil Math Moscow

36(1929), pp 371–382

(19)

Tạp chí Epsilon, Số 08, 04/2016

[13] A Y Khintchine,Zur metrischen Theorie der Diophantischen Approximationen, Math Zeitschrift24(1926), pp 706–713

[14] D Kleinbock T Ly,Badly approximableS-numbers and absolute Schmidt games, J Number Theory164(2016), pp 13–42

[15] S Kristensen, R Thorn, S Velani,Diophantine approximation and badly approximable sets, Advances in Math.203(2006), pp.132–169

[16] Lý Ngọc Tuệ,Xấp xỉ Diophantine trênRvà Liên phân số, Epsilon4(2015)

[17] Lý Ngọc Tuệ,Xấp xỉ Diophantine trênRn- Quy tắc Dirichlet Hình học số, Epsilon

5(2015)

[18] Lý Ngọc Tuệ,Xấp xỉ Diophantine với độ đo - Định lý Khintchine, Epsilon6(2015) [19] Lý Ngọc Tuệ,Trị chơi vơ hạn với thơng tin hoàn hảo, Epsilon7(2016)

[20] C McMullen,Winning sets, quasiconformal maps and Diophantine approximation, Geom Funct Anal.20, no 3, (2010), pp 726–740

[21] W M Schmidt,On badly approximable numbers and certain games, Trans Amer Math Soc.123(1966), pp 178–199

(20)(21)

C S GR ăOBNER L GÌ?

Bernd Sturmfels - Đại Học California, Berkeley, Mỹ

Bài viết Nguyễn Vũ Duy Linh dịch từ báo “What is Groebner basis” giáo sư Bernd Sturmfels đăng tạp chí Notice of AMS, volume 52, number

10; 2005:

Mt c s Grăobner l tập hợp đa thức nhiều biến có tính chất mong muốn giải thuật Mỗi tập hợp đa thức biến đổi thành s Grăobner Quỏ trỡnh bin i ny tng quỏt húa ba kỹ thuật quen thuộc: Phép khử Gauss để giải hệ phương trình tuyến tính, thuật tốn Euclide để tính ước chung lớn hai đa thức biến thuật tốn đơn hình qui hoạch tuyến tính (xemŒ3) Chẳng hạn, đầu vào phép khử Gauss tập dạng tuyến tính sau

D f2xC3yC4z 5; 3xC4yC5z 2g;

Và thuật toán biến đổi thành c s Grăobner

%D fx z C14; yC2z 11g:

Gọi K trường bất kỳ, chẳng hạn trường số thựcK D R; trường số phứcK D C;

trường số hữu tỉK DQ;hay trường hữu hạnK DFp:Ta ký hiệuKŒx1; : : : ; xnlà vành

đa thứcnbiếnxi với hệ số trườngK:NếuF tập hợp đa thức ideal sinh bởilà tập hợphibao gồm tổ hợp tuyến tính đa thức

hi D fp1f1C Cprfr Wf1; : : : ; fr vàp1; : : : ; pr 2KŒx1; : : : ; xng:

Trong vớ d ca chỳng ta, v c s Grăobner%ca sinh idealh%i D hi:

Theo định lý Hilbert sở, ideal trongKŒx1; x2; : : : ; xncó dạngI D hi;nghĩa sinh tập hữu hạncác đa thức

Mộtthứ tự đơn thứctrênKŒx1; x2; : : : ; xnlà thứ tự toàn phầntrên tập hợp tất đơn thứcxa Dxa1

1 x a2 x

an

n với hai tính chất sau

.1/ Nó nhân tính, nghĩa làxa xb kéo theoxaCc xbCc với mọia; b; c 2Nn:

.2/ Đơn thức nhỏ nhất, có nghĩa là1xa với mọia2Nnf0g:

Một ví dụ thứ tự đơn thức (vớinD2/làthứ tự từ điển phân bậc 1x1 x2 x12 x1x2 x22 x32 x12x2

Nếu cố định thứ tự đơn thức;khi đa thức có số hạng khởi đầu trong.f /D xa:Đó đơn thức cực đạixa xuất khai triễn củaf với hệ số khác không Chúng ta viết số hạng củaf theo thứ tự giảm dần, thông thường gạch số hạng khởi đầu Chẳng hạn, đa thức bậc hai viết sau:

(22)

Giả sửI ideal củaKŒx1; : : : ; xn:Khi ideal khởi đầu trong.I /là ideal sinh số hạng khởi đầu tất đa thức trongI W

in.I /D hin.f /Wf 2Ii:

Một tập hữu hạn%củaI l mt c s Grăobner tng ng vi th t theo số hạngnếu số hạng khởi đầu phần tử trong%đủ để sinh ideal khởi đầu:

in.I /D hin.g/Wg 2%i:

Khơng có u cầu tính tối tiểu để trở thành sở Grăobner Nu%l mt c s Grăobner caI thỡ mt hữu hạn củaI chứa%cũng s Grăobner khc phc tớnh phi ti tiu ú, chỳng ta gi%l mt c s Grăobner rỳt gn nu:

.1/ Với mỗig 2%hệ số in.g/trongglà1:

.2/ Tập hợpfin.g/Wg2 %glà tập nhỏ sinh in I /;và

.3/ khơng có số hạng theo sau với mọig 2%nằm in.I /:

Với định nghĩa này, ta có định lý sau đây:Nếu cố định thứ tự đơn thứcthì idealI trongKŒx1; : : : ; xncú mt c s Grăobner rỳt gn nht.

C s Grăobner rỳt gn%cú th c tớnh t mi tập sinh củaI theo phương pháp giới thiệu luận án Bruno Buchberger năm1965:Buchberger gọi phương pháp mỡnh theo tờn ca ngi hng dn Wolfgang Grăobner Sau người ta nhận rằng, ý tưởng s Grăobner ó cú trc ú chng hn mt viết Paul Gordan, nhà nghiên cứu bất biến Tuy nhiên Buchberger người đề mt gii thut tớnh c s Grăobner C s Grăobner rt tin dng gii h phng trỡnh đa thức Giả sửK C;vàlà tập hữu

hạn đa thức trongKŒx1; : : : ; xn:Đa tập củalà tập tất không điểm phức chung

./D f.z1; : : : ; zn/2Cn Wf z1; : : : ; zn/D0với mọif 2g:

Đa tạp không thay đổi ta thay tập đa thức sinh ideal

KŒx1; : : : ; xn:Núi riờng, c s Grăobner gii hn%ca idealhicó đa tạp:

./D.hi/D.h%i/D.%/:

Ưu điểm của%là cho biết đặc tính hình học đa tạp, đặc tính khơng hiển lộ từ:Câu hỏi đặt liệu đa tạp./có thể rỗng hay không.Định lý không điểm Hilbertngụ ý đa tạp./rỗng khi%bằngf1g:

Làm để đếm số không điểm hệ thống phương trình cho? Để trả lời cho câu hỏi này, cần thêm định nghĩa Cho ideal cố địnhI trongKŒx1; : : : ; xnvà thứ tự đơn thức;một đơn thứcxa Dxa1

1 x a2 x

an

n gọi làchuẩnnếu trong

ideal khởi đầu in.I /:Số lượng đơn thức chuẩn hữu hạn biếnxi xuất lũy thừa ideal khởi đầu Chẳng hạn, in.I /D˝

x13; x24; x35˛

thì có sáu mươi đơn thức chuẩn, in I / D˝

x13; x 2; x1x

4

˛

tập hợp đơn thức chuẩn vơ hạn

(23)

Tạp chí Epsilon, Số 08, 04/2016

KhinD1đây làĐịnh lý Đại số, định lý phát biểu đa tạp.f /của đa thức biếnf 2KŒxvới bậcd bao gồmd số phức Ở tập hợp có phần tử

ffgl c s Grăobner, v cỏc n thc chun là1; x; x2; : : : ; xd 1:

Tiêu chuẩn việc định liệu đa tạp có hữu hạn hay khơng tổng qt hóa cơng thức sau cho số chiều đa tạp Xét tập conS tập biếnfx1; : : : ; xng

thế cho khơng có đơn thức S xuất in .I /;và giả sử rằngS có lực lượng lớn số tập có tính chất Khi lực lượng tối đạijSjbằng với số chiều của.I /:

Tập hợp đơn thức chuẩn sở không gian vector trường sốK cho vành cácthặng dư KŒx1; : : : ; xn=I: Ảnh đa thứcp moduloI biểu diễn cách mộtK tổ hợp tuyến tính đơn thức chuẩn Biểu thức làdạng chuẩncủap:Quá trình tính dạng chuẩn thuật tốn chia Trong trường hợp quen thuộc với biếnx;khi màI D hfivàf có bậcd;thuật toán chia biểu diễn đa thức tùy ýp KŒx

dưới dạng mộtK tổ hợp tuyến tính của1; x; x2; : : : ; xd 1:Tuy nhiên thuật toỏn chia ỏp dng c cho mt c s Grăobner tùy ý%với số lượng biến tùy ý

Làm thử nghiệm liệu tập cỏc a thc cho trc l mt c s Grăobner hay không? Xét hai đa thức tùy ýgvàg0 trong%:Thành lậpS đa thứcm0g mg0:Ở đâym

vàm0 hai đa thức bậc nhỏ chom0 in.g/Dmin.g0/: S đa thức

m0g mg0 nằm idealh%i:Chúng ta áp dng thut toỏn chia i vi c s Grăobner tm thời%chom0g mg0:Dạng chuẩn thu nhận mộtK tổ hợp tuyến tính đơn thức khơng có đơn thức chia hết cho đơn thức khởi u t%:iu kin cn %l mt c s Grăobner l

normalform%.m0g mg0/D0với mọig; g0 %:

Tiêu chuẩn Buchberger phát biểu điều kiện cần điều kiên đủ: Một tập hợp

%các đa thc l mt c s Grăobner nu v ch nu mọiS đa thức có dạng chuẩn zero Từ tiêu chuẩn này, người ta đề thuật toán BuchbergerŒ1để tớnh c s Grăobner rỳt gn %t mt hp đầu

Tóm lại, sở Grăobner v thut toỏn Buchberger tỡm chỳng l nhng khái niệm đại số Chúng cung cấp cách tính tốn tiên tiến hình học đại số, chẳng hạn lý thuyết khử, tính đối đồng điều, giải tính kỳ dị, Cũng mơ hình đa thức có mặt khắp nơi từ khoa hc n k thut, cỏc c s Grăobner c cỏc nhà nghiên cứu sử dụng tối ưu hóa, lập trình, robotics, lý thuyết điều khiển, thống kê, sinh học phân tử nhiều ngành khác Chúng mời bạn đọc thử dùng qua cài đặt thuật toán Buchberger (chẳng hạn CoCoA, Macaulay2, Magma, Maple, Mathematica, hay Singular)

Tài liệu tham khảo

[1] DAVID COX, JOHN LITTLE, and DONAL O’ SHEA,Ideals, Varieties and Algorithms An Introduction to Computational Algebraic Geometry and Commutative Algebra, second ed Undergraduate Texts in Mathematics, Springer-Verlag, New York,1997:

(24)(25)

TẬP TRÁNH TỔNG TRONG NHÓM

Terence Tao - UCLA, California, Mỹ

Trong số này, Epsilon trân trọng giới thiệu với độc giả mộtbài viếtcủa nhà toán học Terence Tao, qua dịch Trần Nam Dũng

Tôi Vũ Hà Văn vừa đăng lên arXiv báo “tập tránh tổng nhóm” [1] (gửi đăng đến tạp chí Discrete Analysis), với báo tổng quan đính kèm [2] (gửi đăng đến J Comb.) Cho tập A nhóm cộng tínhG D.G;C/, ta định nghĩa.A/là lực lượng (số phần tử) tập conBlớn củaAtránh tổng trongAtheo nghĩa tổngb1Cb2 vớib1; b2là phần tử phân biệt củaB nằm ngoàiA Ví dụ, nếuAlà nhóm thì.A/ D 1, khơng có hai phần tử củaAlại có tổng nằm ngồiA Một cách tổng qt, nếuAlà hợp củaknhóm, thì.A/tối đa bằngAtheo ngun lý chuồng thỏ

NếuG tập hợp số ngun, ta khơng có nhóm thực nào, ta dự đốn rằng.A/sẽ tăng vớiA Ví dụ ta có kết đơn giản sau:

Mệnh đề 1.NếuAlà tập hợp gồm2k số tự nhiên thì.A/ > k

Chứng minh: Ta sử dụng lý luận Ruzsa [3] vốn dựa lý luận cũ Choi [4] Gọix1là phần tử lớn củaA, sau cách đệ quy, khix1; x2; : : : ; xi chọn, gọixiC1là phần tử lớn trongAkhông bằngx1; x2; : : : ; xi choxiC1Cxj …Avới j D 1; : : : ; i, kết thúc trình xây dựng khơng tìm mộtxiC1 Q

trình cho dãyx1 > x2 > > xmcác phần tử trongA, tránh tổng trongAvà có tính chất với mọiy 2A, hoặcy với cácxi, hoặcyCxi Avớii với

xi > y Lặp lại q trình này, ta thấy mọiy 2Ađều có dạngxi1 xi2 xij vớij 1nào

đó và1i1 i2 < ij m Số biểu thứcxi1 xi2 xij tối đa là2m–1,

2k 2m–1, từ đóm kC1 Vì.A/mnên ta suy điều phải chứng minh

Đặc biệt ta có.A/ >> logjAjđối với tập conAcủa tập hợp số nguyên Ta cải thiện cận đơn giản này, tốt công sức Cận tốt thời điểm

.A/ logjAj.loglogjAj/1=2 o.1/

một kết Shao [5] (xây dựng dựa cơng trình trước Sudakov, Szemeredi Vu [6] Dousse [4]) Trong chiều ngược lại Ruzsa [3] xây dựng ví dụ tập hợpAlớn với

.A/exp.O.plogjAj/)

(26)

cộng tính tùy ý, tập hợp:AWDŒ1 mod 7; mod 7; mod 7H Z=7Z/H có

.A/D3nhưng khơng có hai phần tửx; y 2Anào có tổng bằng0; dạng ví dụ thay7bằng số ngun tố Mersenne lớn hơn, ta có phản ví dụ trongZ=2nZvớinlớn tùy ý Tuy nhiên, theo hướng khẳng định, ta chứng minh câu trả lời cho cõu hi ca Erdăos l khng nh nujGjkhụng cú c nguyên tố nhỏ Rõ

Mệnh đề 2.Với mọik 1tồn tạiC 1sao cho nếuGlà nhóm abel hữu hạn có bậc khơng có ước ngun tố nhỏ hay bằngC vàAlà tập củaGcó bậc lần làC và.A/k

thì tồn tạix; y 2Asao choxCy D0

Có hai cơng cụ sử dụng để chứng minh kết Một “bổ đề loại bỏ số học” chứng minh mởi Kral, Serra Vena [8] Chú ý điều kiện.A/kcó nghĩa với phần tử phân biệtx1; : : : ; xkC1 2A, tồn phần tửxi Cxj; 1i < j kC1,

cũng nằm trongA Một cách nôm na, bổ đề loại bỏ số học cho phép ta “gần như” bỏ yêu cầux1; : : : ; xkC1phân biệt, điều có nghĩa từx 2Agần suy ra2x 2A

Phép đối xứng gần-vị tự kết hợp với giả thiết rằngjGjkhơng có ước ngun tố nhỏ cho nhiều “phân tán” hệ số Fourier của1Amà khai thác để chứng minh định lý

Công cụ thứ hai định lý cấu trúc đây, kết báo chúng tơi, đường theo hướng phân loại tập conAvới.A/nhỏ:

Mệnh đề 3.Cho Alà tập hữu hạn nhóm cộng tínhG với.A/ k Khi tìm nhóm hữu hạnH1; H2; : : : ; Hmvớim ksao chojA\Hij>>k jHi

jAn.H1[ [Hm/j<<k Hơn nữa, nếum Dkthì tập ngoại lệAn.H1[ [Hm/bằng rỗng

Một cách nôm na, định lý ví dụ hợp củaknhóm nói trước dường ví dụ “duy nhất” tập hợpAvới.A/ktheo mô-đu-lô bổ sung vài tập ngoại lệ nhỏ vài làm tốt nhóm cho tập dày

Định lý có màu sắc định lý đảo khác tổ hợp cộng tính định lý Freiman thật ta sử dụng định lý Freiman (và công cụ liên quan định lý Balog-Szemeredi) để thu cách dễ dàng phương án yếu định lý Thật vậy, khơng có tập tránh tổng củaAbậckC1thì tỷ lệ>>k 1của tất cặpa; b A phải có tổng nằm trongA(ngược lại ta lấykC1phần tử ngẫu nhiên củaAvà chúng tránh tổng trongAvới xác suất dương) Từ điều định lý Balog-Szemeredi định lý Freiman (trong nhóm abel, chứng minh Green Ruzsa [10]), ta thấy rằngAphải “cân xứng” với “đối tập cấp số” (“coset progression”)H CP rank chặn Sau ta khử thành phần không xoắnP đối tập cấp số số phương pháp (ví dụ sử dụng phương án lý luận mệnh đề 1), với kết cuối mà người ta xác định vị trí nhóm H1 hữu hạn có phần giao lớn vớiA

(27)

Tạp chí Epsilon, Số 08, 04/2016

có thể lớn so với tập thặng dưAnH1, số hạng sai số khơng chấp nhận Ta vượt qua khó khăn trước tiên ta thực bước tiền “chuẩn hóa” nhómH1, tập thặng dư không giao với đối tập củaH1quá nhiều Các lý luận trở nên phức tạp ta bắt đầu loại bỏ nhiều nhómH1; : : : ; Hi từAvà phân tích tập thặng dưAn.H1[ [Hm/; trình “quản lý epsilon” liên quan trở nên phức tạp mà buộc phải sử dụng cách phát biểu phân tích khơng tiêu chuẩn tốn để giữ độ phức tạp lý luận mức chấp nhận (xem viết blog trước chủ đề [11]) Một nhược điểm cách làm khơng có giới hạn hiệu cho số định lý chúng tơi; thú vị để có chứng minh trực tiếp cho định lý chúng tơi mà dẫn đến giới hạn hiệu

Tài liệu tham khảo

[1] http://arxiv.org/abs/1603.03068 [2] http://arxiv.org/abs/1603.03071

[3] Imre Z Ruzsa, Sum-Avoiding Subsets, The Ramanujan Journal, March 2005, Volume 9, Issue 1, pp 77-82

[4] Choi, S L G On a combinatorial problem in number theory Proc London Math Soc (3) 23 (1971), 629–642

[5] Shao, Xuancheng Finding linear patterns of complexity one Int Math Res Not IMRN 2015, no 9, 2311–2327

[6] Sudakov, B.; Szemerédi, E.; Vu, V H On a question of Erdos and Moser Duke Math J 129˝

(2005), no 1, 129–155

[7] Erdos, P Extremal problems in number theory 1965 Proc Sympos Pure Math., Vol VIII˝

pp 181–189 Amer Math Soc., Providence, R.I

[8] Kráˇl, Daniel; Serra, Oriol; Vena, Lluís On the removal lemma for linear systems over abelian groups European J Combin 34 (2013), no 2, 248–259

[9]Freiman theorem

[10] Green, Ben; Ruzsa, Imre Z Freiman’s theorem in an arbitrary abelian group J Lond Math Soc (2) 75 (2007), no 1, 163–175

(28)(29)

TỐN HỌC CỦA HƠN NHÂN

Matt Baker – Viện công nghệ Georgia, Mỹ

Nguyễn Vũ Anh dịch từblogcủa Matt Baker – Trần Nam Dũng hiệu đính

Thân tặng Võ Quốc Bá Cẩn, biên tập viên Epsilon nhân ngày cưới của anh chị Nguyễn Thị Kim Anh vào ngày 27 / / 2016.

Ngày25tháng6năm2014

Cũng lâu kể từ blog gần – lý gần tơi kết Để kỷ niệm cho kiện này, đánh dấu trở lại tơi blog tốn học tơi đăng định lý Hall hôn nhân

Hãy xét trò chơi solitaire (game xếp Windows): Bạn có chia làm13cọc, cọc4lá, bạn phải chọn1lá từ cọc cho giá trị (từAđếnK) lặp lại Đây kiện tốn học vơ đẹp đẽ trị chơi ln hồn thành, bất chấp qn chia

Chúng ta suy luận điều từ kết tổng quát Philip Hall, thường gọi định lý Hall hôn nhân Cho hữu hạn tập hợpA1; A2; : : : ; An bạn phải tìm phần tử riêng biệtx1 A1; x2 A2; : : : ; xn An:(Trong ví dụ trị solitaire, choAj chứa giá trị nằm cọc thứ j:) Sự lựa chọn gọi SDR (system of distict representatives-hệ đại diện riêng biệt) Dưới điều kiện điều khả thi? Để hệ đại diện riêng biệt tồn điều kiện cần thiết với tập J 1; 2; : : : ; n/; tập hợp

AJ DUj2JAj chứa nhấtjJjphần tử Định lý Hall khẳng định điều kiện

cũng điều kiện đủ

Trong ví dụ trị bói bài, ta chọnkcọc bất kì, chứa4k Bởi có4lá cọc với giá trị, kết hợp cọc chọn, có nhấtkgiá trị khác Do đó, điều kiện định lý Hall thỏa định lý nhân khẳng định có cách thắng

Một ví dụ khác, xét tốn sau có lời giải đơn giản định lý Hall hân:

[Kì thi Putnam, tốn B-3]Một giải đấu vịng trịn2nđội kéo dài2n 1ngày Mỗi ngày, tất đội đấu với đội khác, có1đội thắng và1đội thua mỗintrận Xuyên suốt giải đấu,2đội gặp đúng1lần Liệu có ln chọn ngày đội thắng mà khơng có đội chọn nhiều hơn1lần?

Kết Hall biết tới định lý nhân phát biểu lại theo cách sau Giả sử tập hợp hữu hạn chàng trai quen với tập hợp hữu hạn cô gái Điều kiện để chàng trai kết với người họ quen biết? Điều kiện cần đủ tập hợpkchàng trai, chọn cách ngẫu nhiên, quen với nhấtkcơ gái Rất dễ dàng nhận thấy tương đương hai cách phát biểu (choAj tập hợp cô gái mà chàng trai thứ

(30)

Cách chứng minh đơn giản cho định lý Hall mà biết xuất báo

2trang Halmos Vaughan, thực phép quy nạp theo số chàng train:Ta trích dẫn trực tiếp chứng minh Halmos Vaughan:

Vớin D1;kết hiển nhiên Vớin > 1;nếu với tập hợpkbạn nam1 6k n; ít nhấtk C1 người quen,1 bạn nam cưới người quen anh ta và đưa người lại vào giả thuyết quy nạp Nếu ngược lại, số nhómkchàng trai, 16k6n;có xáckngười quen, tập hợp kết hôn theo giả thiết quy nạp, n kchàng trai lại thỏa mãn điều kiện cần với cô gái chưa kết hôn Thật vậy, nếu16h6n k;và số tập hợphchàng trai chưa có vợ hơnhcơ gái chưa có chồng, thì tập hợphchàng trai chưa có vợ vớikchàng trai kết quen hơnkChcô gái. Áp dụng giả thiết quy nạp vàon kchàng trai chưa có vợ, ta thu kết luận định lý với nC1chàng trai.

Có chứng minh đẹp cho định lý Hall, đề Jack Edmonds, dựa đại số tuyến tính Đây ví dụ sử dụng kỹ thuật đại số để giải vấn đề tổ hợp Trước xem xét chứng minh Edmonds, cần2định nghĩa:

Định nghĩa ChoB là ma trậnmnvới số hạng trườngF đặc số0:Chúng ta gọiBgenericnếu số hạng khác0của độc lập đại số (chúng khơng thỏa mãn đa thức không đồng nhất0;với hệ số trongQ/:

Định nghĩa ChoB là ma trậnmncó hạngr < n:Ta nóiB là dạng Edmonds bình thường (Edmonds Normal Form, hay ENF) nếuB có thể viết dạng ma trận có ma trận B1là ma trậnr r C1/ở phía bên trái mà:

.a/ r C1cột củaB tạo thành tập hợp phụ thuộc tuyến tính nhỏ nhất.

.b/ Mỗi dịng ma trậnB2phía bên trái tổ hợp tuyến tính dịng củaB1:

Dễ nhận thấy, ma trậnBbậcmncó hạngr < ncó thể chuyển dạng ENF cách hốn vị cột dịng Thật vậy, nhờ vào việc hốn vị cột, giả sửr C1cột tạo thành tập hợp phụ thuộc tuyến tính nhỏ Ma trận hốn vịB0 xác định

r C1cột có hạngr;nên nhờ vào hốn vị dịng giả sửr dịng tạo thành sở cho không gian dòng củaB0:Ma trận kết ENF

Bây xem phép chứng minh Edmonds cho định lý Hall:

Chứng minh Chonlà số lượng chàng trai vàmlà số lượng cô gái Cho ma trận genericmn

(31)

Tạp chí Epsilon, Số 08, 04/2016

một số hạng củav bằng0:Và dựa vào tính chất.b/;mỗi dịng củaB2đều trực giao vớiv:

VìB2 làgeniric, nênB2D0:Nhưng tập hợp chàng trai tương ứng vớir C1cột quen với gái tương ứng vớir dịng đầu tiên, mâu thuẫn

Các chứng minh đại số tuyến tính xem mức cần thiết so với phương pháp tổ hợp đơn giản Nhưng lại có vài tính hấp dẫn: Ví dụ, việc làm sáng tỏ cách rõ ràng dẫn tới thuật toán xác suất hiệu để kiểm tra liệu có hệ đại diện riêng biệt (SDR) (Sẽ thuận tiện xem số hạng củaB ẩn số) Chứng minh đưa toán việc xét xem đa thức có khác hay khơng Về mặt tính tốn khơng khả thi để tính định thức củaB đa thức, thay giá trị cụ thể vào ẩn số, xác định cách hiệu giá trị tương ứng có khác0theo mơ-đun ngun tố đủ lớn hay khơng Nếu dùng thuật tốn nhân nhanh ma trận để tính định thức ta có thuật tốn tiệm cận nhanh để xác định tốn nhân có nghiệm hay khơng (trong thực tế, có thuật tốn tất định chạy nhanh hơn, cịn tìm cặp ghép)

Cuối cùng, chúng tơi muốn đề cập đến định lý Hall mở rộng cho trường hợp có vơ số chàng trai (nhưng số cô gái quen chàng trai hữu hạn) Sự tổng quát Everett Whaples đề xuất Một lần ta có cách tốt trích dẫn “Chứng minh từ Sách” (Proof from the Book) Halmos Vaughn, sử dụng cấu trúc điểm - tập hợp topo để giảm bớt trường hợp hữu hạn: Nếu tập hợpB chàng trai vô hạn, xét mỗib2 Bvà tậpG.b/là người

bquen, topo hóa tập topo rời rạc,G.b/là khơng gian compact Hausdorff ViếtGtheo dạng tích Đề-các topo cho mọiG;theo định lý Tikhonov,G compact Nếufb1; b2; : : : ; bnglà tập hữu hạn chàng trai bất kì, xét tập hợpH chứa phần chứa tất phần tg Dg.b/ca

gsao cho gi Ô gj vibi Ô bj; j D 1; ; 2; : : : ; n: H tập đóng G;và dựa vào kết trường hợp hữu hạn,H khác rỗng Vì hợp hữu hạn tập hợp hữu hạn nên lớp tất tập hợp nhưH có tính chất giao hữu hạn, có giao khác rỗng Vì phần tửgDg.b/

trong phần giao thỏa mãn tính chtg.b/Ôg.b0/vi mib Ôb0;phộp chng minh hon tt

Ghi chỳ

1: Bài báo công bố năm1950của Halmos Vaughan, có tên “The Marriage Problem”, đăng lại sách “Classic Papers in Combinatorics” (biên soạn by Gessel and Rota), với báo ban đầu Hall Bài báo Edmonds năm1967có tên “Systems of Distinct Representatives and Linear Algebra” Đoạn chứng minh Edmonds mà tơi nói đến lấy từ báo nàyŒ1:Các bình luận định thức khía cạnh thuật tốn tốn nhân trích từ buốn sách “Thirty - three miniatures” Jiri Matousek Cuốn sách chứa số ví dụ thú vị chứng minh kết tổ hợp đại số tuyến tính

2: Lời giải tốn Putnam xem ởŒ2:Nếu bạn thích tốn này, có tốn vui khác dựa định lý Hall: NếuGlà nhóm hữu hạn vàH nhóm sốn:Chứng minh tồn phần tửg1; g2; : : : ; gnthuộcGsao cho tồn mộtgi lớp kề trái củaH mộtgi trông lớp kề phải

3: Richard Rado chứng minh mở rộng quan trọng sau định lý Hall cho matroid: Giả sửAi họ tập tập hữu hạnS đánh số tậpI vàM matroid trênS:

(32)

con khơng gian véc-tơV cónchiều cho dim spanAJ >jJjvới tập

J f1; 2; : : : ; ng;thìV có sởfa1; a2; : : : ; angvớiai 2Ai với mọii:

4: Có phiên định lý Hall hôn nhân cho siêu đồ thị phát biểu Aharoni Haxell có đưa chứng minh định lý Hall dựa bổ đề Sperner (bổ đề lại tương đương với định lý điểm bất động Brouwer tô-pô) Xem blog

Œ3của Gil Kalai để biết rõ

5: Có số kết tiếng tổ hợp mà “tương đương” nhau, theo nghĩa chúng suy từ cách đơn giản Đó định lý Hall hôn nhân, định lý Dilworth, định lý luồng cực đạt lát cắt cực tiểu định lý Menger Một đặc điểm chung tất định lý vài điều kiện cần hiển nhiên thấy lại điều kiện đủ Có thể xem đâyŒ4:

6: Một số toán yêu thích tơi liên quan đến tốn nhân tốn nhân ổn định tốn thư ký

7: Định lý Hall đóng vai trị chủ đạo báo sauŒ5của Zur Izhakian Louis Rowen supertropical matrix algebra

8: Định lý Hall nhân hiểu điều kiện cần đủ để tồn cặp ghép đầy đủ cho đồ thị hai phe Tutte chứng minh mở rộng sau cho đồ thị bất kỳ, khơng thiết phải hai phe: Một đồ thịGcó cặp ghép đầy đủ với tập conU đỉnh, đồ thị cảm sinh phần bù củaU có tốt đajUjthành phần liên thơng với số đỉnh lẻ Từ định lý Tutte ta nghĩ đến định nghĩa thống “hơn nhân

Tài liệu tham khảo

[1] https://www.ima.umn.edu/preprints/Feb92Series/918.pdf

[2] http://kskedlaya.org/putnam-archive/2012s.pdf

[3] https://gilkalai.wordpress.com/2012/11/25/ happy-birthday-ron-aharoni/

[4] http://robertborgersen.info/Presentations/GS-05R-1.pdf

(33)

VỀ HẰNG SỐ LIÊN THÔNG TRÊN LƯỚI TỔ ONG

Huỳnh Công Bằng - Ecole Normale Supérieure de Lyon

Hằng số liên thông mạng lưới đại lượng liên quan đến số lượng đường không tự cắt lưới (lưới vuông, hay lục giác (lưới tổ ông), ) Vào năm 1982, lập luận dựa gas coulomb Nienhuis dự đoán mạng lưới lục giác, số liên thông

q

2Cp2 Điều chứng minh chi tiết Duminil-Copin Smirnov cách sử dụng phương pháp"parafermionic".

1 Giới thiệu

Ký hiệucnlà số đường không tự cắt (SAW) mạng lưới lục giácH với độ dàinvà bắt đầu từO Ta có:

cn.p2/n

Điều có cách đếm số đường lên phía xuống phía bước thứ

2kC1và số đường ngang bước thứ2kC2vớik 2N

Ta cắt đường SAW có độ dàimCnthành hai phần SAW có độ dàinvàm Do

cmCncmcn:

Theo bổ đề subadditivity, ta có limc

1 n

n D2

p

2; 2andcnn;8n

vì Dlim

n c n n Dinf

n.c n n/

Định lý 1. Đối với mạng lưới lục giác, ta cóD

q

(34)

Một vài ký hiệu: Ta làm việc trung điểm cạnh củaH Tập hợp tất điểm làH˘

Ta viếta2 H˘; Wa!E nghĩa bắt đầu tạiavà kết thúc điểm trongE H˘

l /D#fa2H Wa2 glà độ dài (Vì số đỉnh thuộc) Ta định nghĩa hàm số sau đây:

z.x/D X Wa!H˘

xl /forx > 0:

Ta co

z.x/D X Wa!H˘

xl / DX n

cnxl / DX n

cnxnX n

.x/n:

Do đó,

Ifx <

thenz.x/ <C1: Ifx >

thenz.x/D C1

Khi đó, ta cần chứng minh

z.x/ <C1ifx < p 2Cp2

va

z.x/D C1ifx D p

2Cp2 :

ta datxc D p

2Cp2

voij De2i .

2 Cầu

Trong mục này, ta định nghĩa lớp SAW cầu ta chứng minh số cầu tăng tỉ lệ với số SAW (n) từ ta chứng minh

b.H /D

q

2Cp2:

Định nghĩa 1. Một cầun-bước SAW cón-bước cho

1.0/ < 1.i /1.n/8i D1; 2; 3; : : : ; n:

(35)

Tạp chí Epsilon, Số 08, 04/2016

Ký hiệubnà cầun-bước với.0/D0 Ta đặtb0D1:

Ta cóbmCnbmbn,

b D lim

n!C1bn

n Dsup n

b n n:

Hơn nữa,bnnnb n

Định nghĩa 2. Một nửa mặt phẳngn-bước SAW cónbước với1.0/ < 1.i /;8i:

Ta đặthnlà số lượng nửa mặt phẳngnbước với.0/D0:

Định nghĩa 3. Độ dày nửa mặt phẳngnbước max

0in1.i / 0minin1.i /

vớibn;Alà số n-bước cầu với độ dàiA:

Ta cóbn D n

X

AD1 bn;A

Định lý 2. (Hardy-Ramanujan): Chon2N, gọiPD.n/là số cách để viếtnDn1Cn2C Cnk

trong đón1 > n2 > > nk 1cho bất kỳk, đó, ta có lnPD.n/n

3

12

asn! C1:

Mệnh đề 1. hnPD.n/bnvới mọin1 Đặtb0 D0, ta định nghĩa

AiC1 Dmax j >ni 1/

i

.1.j / 1.ni//

va

niC1 Dmax

n

j > ni 1/i.1.j / 1.ni// DAiC1

o

:

Nghĩa là:n1là giá trị cực đại của1.j /Wj > n0vàn2là giá trị cực tiểu của1.j /; j > n1 Ta đặthn.a1; a2; : : : ; ak/là số nửa mặt phẳngnbước với

K Dk; Ai Dai:

Ta có

hn.a1; a2; : : : ; ak/hn.a1Ca2; a3; : : : ; ak/ : : :

hn.a1Ca2C Cak/Dbn;a1Ca2CCak:

Do đó,

hnDX k1

X

1a1<a2<<ak

hn.a1; a2; : : : ; ak/

X

k1

X

1a1<a2<:::<ak

bn;a1Ca2CCak

n

X

AD1

PD.A/ bn;A PD.n/ n

X

AD1 bn;A

„ ƒ‚ …

(36)

Chúng ta có đượchnPD.n/bn

Chứng minh cung cấp phương pháp để phân tích thành cầu có độ dài giảm dần Chúng ta chứng minh định lý sau Hammersley-Welsh cho mạng lục giác

Định lý 3. Cố địnhB >

2

3

12

, đó, tồn sốn0.B/sau cho

8n > n0.B/WcnbnC3eB p

nC3 : Chứng minh. Ta chứng minh

cn n

X

mD0

hn mhmC4:

Đặt x1 D max

0in1.i /; m D maxfi W.i /Dx1g Ta xóa m/ thêm vào điểm a1; a2; a3; a4; a5của lục giác chứa.m/:

Đường 0/; 1/; : : : ; m 1/; a1; a2; a3; a4/là nửa mặt phẳng có.mC3/bước và.a5; mC1/; : : : ; n//là nửa mặt phẳng có.n m/bước Do đó,

cn n

X

mD0

hn mhmC3

Sử dụng mệnh đề:

cn n

X

mD0

hn mhmC3 n

X

mD0

PD.n m/PD.mC3/bn mbmC3

n

X

mD0

PD.n m/PD.mC3/bnC3

Theo định lý Hardy thì:PD.n/ n

12

khin! C1, đó9˛ WPD.n/ ˛eB0.A2/

với

B > B0>

2

3

12

có được:

PD.n m/PD.mC3/˛2eB0

hp

n m C

pmC3

i

˛2eB0 p

nC3 ;

Do đó:

cn.nC1/˛2eB0pnC3bnC3

9B0.B/;8nB0.B/W cn eB p

nC3 bnC3:

(37)

Tạp chí Epsilon, Số 08, 04/2016 Ta có:

cneBpnC3bnC3 )c n n eB

p

nC3 n b

1 nC3

nC3 n

nC3 )b

nên Db

Ghi chú.Chúng ta có kết cho mạng hình vngZ2bằng cách thay thếnC3bởinC1:

Cố định B >

2

3

12

khi có giá trị n0 D n0.B/sao cho cn bnC1eB p

n

với

nn0

Chúng ta định nghĩaB.x/ DXbnxn,

Nếux >

:B.x/D C1 Nếux <

:B.x/ <C1 3 Parafermionic

Một miềnlà hợp tất trung điểm từ việc đưa tập hợp đỉnhV / Một trung điểm z thuộc miềnnếu đầu mút cạnh liên kết trung điểm thuộc vềV / Trung điểm thuộc về@nếu có đầu mút nằm

V /

Định nghĩa 4. Số gốc quayW.a; b/của SAW giữaavàb số lần rẽ sang trái trừ số lần rẽ sang phải của, lấy kết nhân với

3 từađếnb

Định nghĩa 5. Choa2@; z ;chúng ta đặt

F z/DF a; z; x; / D X Wa!z

e i W.a;z/xl /:

Bổ đề 1. NếuxDxc; D

8, đóF thỏa mãn:

8v 2V /W.p v/F p/C.q v/F q/C.r v/F r/D0

trong đóp; q; r cạnh liên kết đếnv: Chứng minh. Ta viết

.p v/F p/C.q v/F q/C.r v/F r/ D.p v/ X

Wa!p

e i W.a;p/xl /C.q v/ X Wa!q

e i W.a;q/xl /C

.r v/ X Wa!r

e i W.a;r/xl /

(38)

Ta kí hiệu:

Cp D f W Wa!pg Cq D f W Wa!qg Cr D f W Wa!rg

Cp3 D

˚

2Cp W quaqvàr Cp2 D˚

2Cp W quaqvàr Cp1 D

˚

2Cp W không quaq khơng quar

Ta có3trường hợp:

1 If 2Cn3, ta liên kết đếne, định nghĩa bởi:e qua trung điểm mà

đi qua thêm vào đườngp !r vàoe Đặc biệt,e 2Cp3

2 Cp1, ta liên kết đến Q vàQQ qua trung điểm (p; q orp; r ) cách mở

rộng đường thêm bước

3 2Cp2, trường hợp 2C p orC

1 r ,

Ta định nghĩa: Nếu đường kết thúc trung điểmz,C /D.z v/e i w.a;z/xl / Trong lập luận tiếp theo, xem xét vài trường hợp:

1 Trường hợp đầu tiên, chứng minh

xcl /.p v/e i w.a;p/C.q v/e i w.a;q/xcl /Q D0

We havel /Dl /; qQ v/De2i .p v/và

w.a; p/Dw.a; r/Cw.r; p/Dw.a; r/C

4

3

wQ.a; p/DwQ.a; r/CwQ.r; p/DwQ.a; r/C

3 Dw.a; r/C

3

Do

xcl /.p v/e

i w.a;p/

C.q v/e i w.a;q/xcl /Q D.p v/e i Œw.a;r/ 43 Ce

2i

3 .p v/e i Œw.a;r/C

3  Dh.p v/e5i Ce

2i

3 p v/e 5i

6

i

e 5i8w.q;r/

D.p v/e 5i8w.a;r/

0

@e

5i Ce

i

„ ƒ‚ …

0

1

AD0

2 Trường hợp thứ 2, chứng minh

c /Cc /Q Cc /QQ D0

Nói cách khác,

xcl /.p v/e i w.a;p/C.q v/e i wQ.a;q/xl /Q

c C.r v/e

i wQQ.a;r/

(39)

Tạp chí Epsilon, Số 08, 04/2016 Ta có

l /Q Dl /QQ Dl /C1

w.a; q/Q DwQ.a; p/CwQ.p; q/Dw.a; p/C

3

; wQQ.a; r/ DwQQ.a; p/CwQ.p; r/Dw.a; p/C

q v D.p v/e2i ; r v D.p v/e

2i

Ta viết lại sau:

xcl /.p v/e

i w.a;p/

C.q v/e i wQ.a;q/xl /Q

c C.r v/e

i wQQ.a;r/

xcl /QQ D0 ,1Cxce2i e

i

5

8 Cxce 2i

3 e i

24 D0 ,xce7i Ce

7i

C1D0 , 2cos

8xC C1D0 ,cos

8 D 2xC D

p

2Cp2

Điều

q

2Cp2D2cos

8 Bổ đề chứng minh

Ghi chú1) Chúng ta thấy.1/giống tích phân rời rạc theo đường tam giác mạng lưới theo nghĩa sau GọiHlà mạng lưới "dual" củaH Một đường:

W f0; 1; 2; : : : ; ng !H

(v 2Hnếu nếuvlà tâm mặt củaH)

F WH˘ !C hàm trênH:Chúng ta định nghĩa:

I

F z/dz WD n

X

iD0 F

i

CiC1

.iC1 i/

If.0/Da; 1/Db; 2/Dc; 3/DavàF giống định nghĩa 5, ta có

I

F z/dz D.b a/F

a

Cb

C.c b/F

b

Cc

C.a c/F

a

Cc

D2e2.p v/F p/C2e i

2 q v/F q/C2e i

2 r v/F r/ D0

Tổng quát hơn, W f0; 1; 2; : : : ; ng !Hsao cho.0/D.n/ Chúng ta phân tích thành đường tam giác Ta có:

I

F z/dzD0:

2) Những mối quan hệt khơng thể xác định hàmF định nghĩa, số biến lớn số phương trình Chúng ta đưa hai hàm mà thỏa mãn mối quan hệt này:

(40)

4 Chứng minh định lý

Chúng ta cố địnha2H˘ giống gốc tọa độ mặt phẳng phức.Chúng ta xem xét miền dọcDST tạo thành từT dãy lục giác, phiên hữ hạnST;Lcắt độ cao˙L

và góc˙

3 Chúng ta xác địnhV ST/vàV ST;L/

Định nghĩa˛; ˇtương ứng biên bên trái, bên phảiST and; biên phía phía củaST;L

Chúng ta có

8z 2ˇ;Rez D 3T C1 :

Biên phía trênthuộc đường thẳng có phương trìnhp3y xD3LC

Biên phía dướithuộc đường thẳng có phương trìnhp3y x D 3L

Do

V ST/D

z V H /W0Rez 3T C1

V ST;L/Dnz 2V H /W

ˇ ˇ ˇ

p

3Im.z/ Re.z/

ˇ ˇ ˇ3L

o

Chúng ta định nghĩa

AT;L.x/D X ST;LWa!˛nfag

xl /

BT;L.x/D X ST;LWa!ˇ

xl /

ET;L.x/D X ST;LWa![

xl /

Bổ đề 2. ForxDxC; D 8, Ta có

c˛AT;L.xc/CBT;L.xc/CcET;L.xc/D1

trong đóc˛ Dcos3

8 ; c Dcos

4

Chứng minh. Chúng ta chứng minh rằng: Nếux Dxc; D

8

8v 2V /W.pv v/F pv/C.qv v/F qv/C.rv v/F rv/D0

trong đópv; qv; rv trung điểm cạnh liên kết đếnv: Chúng ta cộng mối quan hệ tất cách đỉnhv 2V ST;L/khi

X

z2ˇ

F z/CX z2

e2i F z/CX z2˛

ei F z/CX z2

e 2i F z/D0

)X

z2ˇ

F z/CX z2

e2i F z/C 1/X z2˛

F z/CX z2

e 2i F z/D0

(41)

Tạp chí Epsilon, Số 08, 04/2016

1/X z2˛

F z/ D e

i Ce i

2 AT;L.xc/ D cos

8 AT;L.xc/ D C

cos3

8 AT;L.xc/

X

z2ˇ

F z/Dei 0BT;L DBT;L

e2i X z2

F z/Ce 2i X z2

F z/De2i e 2i

3 X Wa!

xl /c Ce 2i e 2i

3 X Wa!

xcl /

De i4 X Wa!

xcl /Ce i

4 X Wa!

xcl /D

e i4 Ce i

4

ET;L.xc/Dcos

4ET;L.xc/

Do đó, ta có

cos3

8 AT;L.xc/CBT;L.xc/Ccos

4ET;L.xc/D1:

Chú ý.Chúng ta nhận thấy rằng.AT;L.xc//Land.BT;L.xc//Llà dãy tăng,

lim

L!C1AT;L.xc/DAT.xc/andL!C1lim BT;L.xc/DBT.xc/:

ta có

cos3

8 AT;L.xc/CBT;L.xc/Ccos

4ET;L.xc/D1

và.AT;L.xc//L; BT;L.xc//Llà dãy tăng, đóET;L.xc/là dãy giảm, điều suy

lim

L!C1ET;L.xc/DET.xc/:

Ta có cos3

8 AT.xc/CBT.xc/Ccos

4ET.xc/D1

Bây giờ, trình bày phần chứng minh định lý

Trong phần 2, b.H / D H / Do đó, cần chứng minh

z.xc/D C1vàB.x/ <C1;8x < xc Chứng minh rằngz.xc/D C1:

Chúng ta xem xét trường hợp:

If9T0 WET0.xc/ > 0do đóz.xc/ C1

X

LD1

ET;L.xc/ C1

X

LD1

ET0.xc/D C1 Bởi

(42)

If8T W ET.xc/ D 0, then we have cos3

8 AT.xc/CBT.xc/ D 1với mọiT: Dễ

dàng chứng minh

ATC1.xc/ AT.xc/zcBTC1.xc/2

do

cos3

8 ATC1.xc/CBTC1.xc/ cos

8 AT.xc/ BT.xc/D0 ,0Dcos3

8 ŒATC1.xc/ AT.xc/CBTC1.xc/ BT.xc/ cos3

8 xcBTC1.xc/

CBTC1.xc/ BT.xc/

Kí hiệu cos3

8 Dc˛, ta có

c˛xcBTC1.xc/2CBTC1.xc/BT.xc/:

Chúng ta chứng minh

BT.xc/mi n

B1.xc/; c˛xc

1

T :

Giả sử tồn tạiT0sau choBT0.xc/ <

B1.xc/; c˛xc

1

T0, có c˛xcBT0.xc/2CBT0.xc/BT0 1.xc/

)BT0 1.xc/c˛xcBT0.xc/2CBT0.xc/

<min

B1.xc/; c˛xc

1

T0 Cc˛xcmin

B1.xc/; c˛xc

2 1

T02

B1.xc/; c˛xc

1

T0 Cmin

B1.xc/; c˛xc

1

T02

min B1.xc/; c˛xc B B B B B @ T0 C T02

„ ƒ‚ … < T0 1 C C C C C A

Bằng cách tương tự, ta có

BT0 2.xc/ <min

B1.xc/; c˛xc

1

T0 : : :

B1.xc/ <min

B1.xc/; c˛xc

(43)

Tạp chí Epsilon, Số 08, 04/2016

bất đảng thức cuối mâu thuẫn,

BT.xc/min

B1.xc/; c˛xc

1

T for allT;

then

z.xc/ C1

X

TD1

BT.xc/min

B1.xc/; c˛xc

C1 X

TD1

T D C1

2 Chứng minh củaB.x/ <C1;8x < xc

Giả sử rằngx < xc VìBT.x/là cầu có độ dài nhấtT nên

BT.x/DXxl /X.x xc/

l

/xl / X.x xc/

T

xl /

x xc T BT.xc/ x xC T

bởi vìBT.xc/1 Như vậy,

B.x/D C1

X

TD1

BT.x/ C1

X

TD1

x xc

T

<C1

bởi vìx < xc

5 Mơ hình vịng O.n/ trên mạng lưới lục giác:

Cholà đồ thị mạng lưới lục giác, xem xét cấu hình vịng đơn giản khơng tự cắt Chon0; x > 0: Chúng ta định nghĩa độ đo tập hợpX cấu hình vịng đơn khơng tự cắt:

P.!/ n#loop.!/x#edges.!/:

Kể từ đó, phần giao vùng thêm vào Trong trường hợp này, cấu hình hợp vịng khơng tự cắt giao diện tránh vòng từavàob Chúng ta mở rộng định nghĩa "paraforminoic observable" phần

Định nghĩa 6. Chou; v2 @; z 2; n2Œ0I2; x > 0, ta định nghĩa

F z/DF z; u; v; x; n; /D

P

!2.u;z/

e i w.u;z/xj!jn#loop.!/

P

!2.u;v/

e i w.u;v/xj!jn#loop.!/

trong đó.u; v/là tập hợp cấu hình với giao diện (SAW) từuđếnv:

Ghi chú.Chou; vcố định, đặt

c D X

!2.u;v/

e i w.u;v/xj!jn#loop.!/;

khi có

F z/DF z; u; v; x; n; /D c

X

!2.u;z/

e i !.u;z/xj!jn#loop.!/

(44)

Nếu #loops.!/ > 0thenP.!/D0

Nếu #loops.!/D0thenP.!/x#edges.!/ Nghĩa làX D fSAWg

Mệnh đề 2. Gọi là miền hữu hạn củaH vàa; b @ Đặtx.n/ D p

2Cp2 n

n/D1

4 arccos

n1

vàF "paraformionic observable"

.p v/F p/C.q v/F q/C.r v/F r/D0

trong đóp; q; r trung điểm cạnh liên kết đếnv:

Ghi chú:NếunD0thenx D p

2Cp2

; D

8 Điều chứng minh bổ đề Chứng minh. Chúng ta liên kết cấu hình1; và3(4; 5; 6)

Cố định trường hợp (trường hợp1; 2; 3) Chúng ta chứng minh rằng:

C.1/CC.2/CC.3/D0:

Nói cách khác,

xj!1j

n p x/e w

.u; p/n#loops.!1/Cxj!2j

n q x/e

w2.u;q/

n#loops.!2/ Cxj!3j

n r x/e

w3.u;r/

n#loops.!3/ D0

trong đój!2j D j!3j D j!1j C1và

#loops.!1/D#loops.!2/D#loops.!3/;

w2.a; q/Dw2.a; p/Cw2.p; q/Dw1.a; p/C

; w3.a; r/Dw.a; p/C

3

q vD.p v/e2i ; r v D.p v/e 2i

(45)

Tạp chí Epsilon, Số 08, 04/2016 Chúng ta viết lại

xj!1j

n p v/e

i w1.u;p/

n#loops.!1/ Cxj!2j

n q v/e

i w2.u;q/

n#loops.!2/ Cxj!3j

n r v/e

i w3.u;r/

n#loops.!3/ Dxj!1j

n p v/e

i w1.u;p/

n#loops.!1/ Cxnj!1jC1.p v/e

2i

3 e i Œw1.u;q/

3n#loops.!2/ Cxnj!1jC1.p v/e

2i e i

h

w 1.u;p/C

i

n#loops.!3/

Nó dễ để chứng minh rằng1Ce2i xne i

3 Cxne 2i

3 e i

3 D 0 Chúng ta thay thế by

1

4 arccos

n

2

andxnbởi p

2Cp2 n

, ta có

1Ce2i xne i

3 Cxne 2i

3 e i

3

D1Ce2i e i

3 i 4arccos

n

2/

p

2Cp2 nC

e 2i e i e

i 4arccos

n

2/

p

2Cp2 n D1 e 4iarccos

n

2/

p

2Cp2 n

ei4arccos n

2/

p

2Cp2 n D1 cos

4arccos n p

2Cp2 n

cos 4arccos n p

2Cp2 n D1 2cos

1 4arccos n p

2Cp2 n D

điều

6 Giả thuyết cho mô hình O(n)

Định nghĩa biến đổi pha tương ứng đến tồn củaxc 2.0I C1/sao cho

1 Chox < xc: Xác suất đỉnhavàblà vòng giảm nhanh lũy thừa theo khoảng cách giữaavàb:

2 Chox > xc: Xác suất đỉnhavàblà nghịch đảo lũy thừa theo khoảng cách giữaavàb:

Giả thuyết:Chon2Œ 2I2, đóxc.n/D p

2Cpx n Tài liệu tham khảo

(46)

[2] Hugo Duminil-Copin, Smirnov: The connective constance of the honeycomb lattice equals

q

2Cp2

[3] Hugo Duminil-Copin, R Bauerschmidt, J Goodman, G Slade, Leture on the self-avoiding-walks

(47)

RỘNG HẸP NHỎ TO VỪA VẶN CẢ (GIỚI THIỆU TÔPÔ HỌC)

Nguyễn Hữu Việt Hưng

(Trường ĐH Khoa học Tự nhiên, ĐHQG - Hà Nội)

Có vấn đề củahình học, lạikhơng phụ thuộc vào kích cỡ to nhỏ, rộng hẹp, dài ngắn đối tượng liên quan Những vấn đề thuộc lĩnh vực gọi làTôpô học(Topology1) Trong vấn đề thuộc loại này, chuyện mảnh đất rộng hay hẹp, vuông hay méo chẳng quan trọng (Thế có lạ khơng!) Vì thế, người buôn đất, buôn bất động sản nên học Tơpơ Nếu tị mị mà họ học, họ thể nhà Tôpô học kẻ điên, hâm hấp

Cao đàm khốt luận khơng khéo dễ dẫn đến tư biện, mù mờ, dễ sinh nói nhảm Để tránh chuyện đó, ta bắt đầu vài ví dụ Đơi khi, vài ví dụ thực chất đẻ lý thuyết, có đẻ ngành học

Leonhard Euler(1707 - 1783), nhà toán học vĩ đại người Thuỵ Sĩ, xem cha đẻ ngành Tơpơ học, ông người nghiên cứu hai toán sau

1 Bài toán bảy cầu

Kăonigsberg l mt thnh ph c thucVng quc Phv nước Đức 1945 Sau Đại chiến Thế giới II, thuộc Liên Xơ (cũ) Nga, gi lKaliningrad Ch cú rt ớt du tớch ca Kăonigsberg cịn sót lại ngày Kaliningrad

Ở thành ph Kăonigsberg, cú7chic cu Chỳng ni hoc l hai b sông, bờ sông hai cù lao, nối hai cù lao Xem đồ sau đây:

1

(48)

Từ xưa, c dõn Kăonigsberg ó t cõu hi: Liu cú thể lần qua tất cả7chiếc cầu mà cầu phải lặp lại hay khơng?

Khơng cần để tâm nhiều đến vị trí cụ thể của7chiếc cầu Điều quan trọng mà người ta quan sát từ toán sau:Đây vấn đề hình học, khơng phụ thuộc vào độ lớn yếu tố tham dự(dịng sơng rộng hay hẹp; cầu dài hay ngắn, to hay bé; cù lao lớn nhỏ nào) Vấn đề phụ thuộc hình dáng vị trí tương đối yếu tố

Khơng có chứng no cũn li chng t rng Euler ó ti Kăonigsberg Tuy nhiên, năm 1735 ông chứng minh mong muốn tìm cách qua cả7chiếc cầu “một lần, không lặp lại” thực

Chúng ta thử tìm hiểu lời giải Euler cho bi toỏn7cõy cu Trờn bn Kăonigsberg hóy thay mi bờ sông, cù lao điểm, gọi đỉnh, thay mối cầu đường nối đỉnh, gọi cạnh Hình thu gọi th Bi toỏn v7cỏi cu Kăonigsberg thc cht chuyện cố gắng “vẽ nét” đồ thị sau đây:

(49)

Tạp chí Epsilon, Số 08, 04/2016

Hãy bắt đầu với nhận xét đơn giản sau đây: Mỗi ta qua đỉnh, có2cạnh (2cây cầu) xuất phát từ đỉnh qua: Cạnh tới, cạnh khỏi đỉnh Như thế, lần qua đỉnh, số cạnh nối với đỉnh mà ta chưa qua giảm đi2 Cho nên, đỉnh có số cạnh nối tới số chẵn (gọi tắt đỉnh chẵn) lần tới ta ln cịn đường để ngồi Cịn đỉnh lẻ, chẳng hạn có (2kC1) đường nối với đỉnh đó, sauklần qua, tới lần (kC1) ta hết đường để khỏi đỉnh

Như vậy, đỉnh lẻ cản trở cho việc “đi qua” mà dừng lại Chiến thuật ta không xuất phát từ đỉnh chẵn (nếu cịn đỉnh lẻ), xuất phát từ đỉnh chẵn, khỏi đỉnh đó, biến đỉnh chẵn thành đỉnh lẻ phần trò chơi

Ta cần xét đồ thị liên thông, nghĩa đồ thị mà giữa2đỉnh có đường nối (Việc vẽ đồ thị không liên thông hiển nhên qui vẽ thành phần liên thông nó.) Dựa nhận xét đỉnh chẵn đỉnh lẻ nói trên, ta chứng minh: (1) Trong đồ thị, số đỉnh lẻ số chẵn,

(2) Một đồ thị liên thơng khơng có đỉnh lẻ nào, cần tối thiểu1nét vẽ (3) Một đồ thị liên thơng có2nđỉnh lẻ (n > 0), cần tối thiểu n nét vẽ

Cách vẽ sau: Xuất phát từ đỉnh lẻ (nếu có), vẽ tuỳ ý khơng vẽ Khi ta gặp đỉnh lẻ khác Nét vẽ vừa khử bớt2đỉnh lẻ (là điểm đầu điểm cuối nét vẽ) Lặp lại trình khơng cịn đỉnh lẻ Trường hợp khơng có đỉnh lẻ nào, xuất phát từ đỉnh chẵn bất kỳ, vẽ tuỳ ý khơng vẽ dược Khi ta gặp lại đỉnh xuất phát

Chúng không sâu vào chi tiết chứng minh khẳng định

Cái phong bì có5đỉnh, đó4góc là4đỉnh lẻ Vì thế, khơng thể vẽ phong bì bằng1nét Cần nhất4=2D2nét để vẽ phong bì

(50)

2 Bài tốn số mặt, số cạnh, số đỉnh đa diện

L Euler chứng minh định lý sau đây, nhìn tưởng trị chơi trẻ con:Trong đa diện lồi nào, số mặt trừ số cạnh cộng với số đỉnh bằng2

Hãy lấy vài ví dụ

Trong tứ diện, số mặtmD4, số cạnhc D6, số đỉnhd D4; Ta cóm cCd D4 6C4D2

Trong hình hộp chữ nhật, số mặtmD6, số cạnhc D12, số đỉnhd D8; Ta cóm cCd D6 12C8D2

Vì lại có chuyện lúc lấy dấu “cộng”, lúc lại lấy dấu “trừ” định lý trên? Xin thưa: Mặt yếu tố chiều, đỉnh chiều, yếu tố chẵn chiều mang cộng; cịn cạnh yếu tố chiều, tức số chiều lẻ, nên mang dấu trừ

(51)

Tạp chí Epsilon, Số 08, 04/2016

Các mặt, cạnh, đỉnh đa diện biến thành mặt (cong), cạnh (cong), đỉnh mặt cầu Như thế, định lý Euler chất định lý mặt cầu:Trong cách phân mặt cầu thành hình đa giác cong, số mặt trừ số cạnh cộng số đỉnh bằng2 Hơn nữa, hình thu từ mặt cầu phép biến đổi liên tục (tương tự co dãn màng cao su) nghiệm định lý

Chúng ta vừa đạt bước tiến quan trọng cách nghĩ: Bài toán Euler ban đầu xét nhiều đối tượng, đa diện lồi Rút cuộc, tốn đối tượng nhất, mặt cầu

Đạt bước tiến sử dụng lập luận biển đổi kiểu “co dãn cao su” Người ta gọi phép biến đổi tôpô

(52)

Tiếp theo, lấy mặt “xuyến kép”, có cách dính2chiếc săm ơtơ vào Bạn tự chọn cách chia mặt “xuyến kép” thành mặt giống hình vng (cong), cạnh (cong), đỉnh Chẳng hạn, ta chọn cách chia mơ tả hình vẽ Các đường đỏ vàng cắt ở2điểm (1điểm nhìn thấy,1điểm hình chiếu thẳng đứng điểm nhìn thấy), vậyd D2 Các đường đỏ vàng được2đỉnh chia làm4cạnh (4nửa đường tròn), cộng thêm2đường mầu xanh, vậyc D6 Sau cắt theo đường mặt xuyến kép bị chia thành ra2hình chữ nhật, vậymD2 Ta cóm c Cd D2 6C2 D Như thế, số Euler xuyến kép

Ta dính nhiều săm ơtơ với để tạo thành mặt xuyến cóglỗ Số mà Euler quan tâm mặt bằngm cCd D 2.g 1/

Trong tơpơ đại, định lý Euler tổng quát hoá sau:Nếu chia vật thể n chiều thành phần “giống đa diện”, tổng số phần với chiều chẵn trừ tổng số phần với chiều lẻ số, gọi đặc số Euler, vật thể đó

Như thế, vật thể tổng hoà nhịp nhàng hai phần âm dương, chẵn lẻ nội nó, khơng thể thay đổi Đặc số Euler, gọi làđặc số Euler – Poincaré(bởi

Poincaré(1854-1912) người ý thức chuyện trường hợp số chiều tuỳ ý), vật thể loại“bản thể”, loại“chứng minh thư”, một“ID Card”

của vật thể

Hệ là, hai vật thể có đặc số Euler khác nhau, chúng khơng thể biến thành sau phép biến đổi thuận nghịch liên tục (kiểu co dãn cao su) Người ta nói hai vật thể khơngcùng kiểu tơpơ

Như thế, mặt cầu, mặt xuyến, mặt xuyến kép khơng kiểu tơpơ, chúng có đặc số Euler khác (tương ứng bằng2,0, 2) Về mặt trực giác, chúng khơng kiểu tơpơ? Lý thật đơn giản: Mặt cầu khơng có lỗ nào; mặt xuyến có1lỗ (là chỗ người ta chui vào để biến săm thành phao bơi); mặt xuyến kép có2lỗ Các nhà tơpơ bảo mặt xuyến có1

lỗ, nên có giống (genus) bằng1; mặt xuyến kép có2lỗ, nên có giống bằng2; mặt cầu khơng có lỗ nào, nên có giống bằng0.À thế, phải có lỗ giống không bị triệt tiêu Các nhà tôpô thật giỏi ỡm

Riemanncòn chứng minh định lý thật thâm thuý: Mọi mặt chiều trơn (tức mịn màng), bị chặn (có thể giữ phịng), có hướng (tức phân biệt phía ngồi da, phía thịt) xác định mặt tôpô cách đếm số lỗ Chà chà, phải mờiPicassođến

Những chuyện kể dẫn đến gì? Sau kết luận thật khó tin

Khẳng định:Dù có nhào nặn cục bột, hình bánh mì, kỹ đến mức nào, miễn hình của cục bột lúc đầu thơi nặn bánh mì, nặn xong lại để bánh vào chỗ cũ, khi ln ln có hạt bột mì khơng thay đổi vị trí.

(53)

Tạp chí Epsilon, Số 08, 04/2016

1) Khơng có phép biến đổi biếnB thànhS giữ nguyên điểm trênS (Ý chứng minh: Giả sử tồn phép biến đổi Trước phép biến đổi,S biên củaB, sau phép biến đổiS phải biên chínhS Điều vơ lý.)

2) Giả sử phản chứng, sau nhào nặn hạt bột mì giữ ngun vị trí Giả sử hạt bột mìx biến thànhf x/sau nhào nặn Nửa đường thẳng nốif x/vớix (kéo dài) cắt mặt cầuS điểm nhất, ký hiệug.x/ Phép biến đổix thànhg.x/chính phép biến hình liên tục, biếnB thànhS giữ nguyên điểm trênS (Nếux nằm trênS, nửa đường thẳng nốif x/vớixcắtS chínhx.) Điều mâu thuẫn với điểm1) Mâu thuẫn bác bỏ giả thiết phản chứng

Hai toán Euler nghiên cứu nói trênlà ví dụ đơn giản vấn đề hình học kích cỡ khơng quan trọng, có hình dáng vị trí tương đối đóng vai trị định Ngành tốn học nghiên cứu vấn đề ngày gọi Tôpô học (Topology) Ngẫm cho kỹ chuyện kích cỡ khơng quan trọng tạo hố trì ngun lý hàng đầu, đóng vai trị“đảm bảo an ninh”khơng cho xã hội lồi người, mà cho toàn giống loài tự nhiên Nếu người mua nhầm đôi giầy, chật hay rộng quá, tức người gặp vấn đề kích cỡ, đem đổi Thế nhưng, người lấy vợ, gặp vấn đề kích cỡ, địi đổi, nguy hiểm vô Và nhiều người sau lấy vợ gặp vấn đề kích cỡ thế, cần phải đổi, xã hội chắn sinh loạn

Bà chúa thơ nômHồ Xuân Hương(1772–1822) nhà Tơpơ học Việt Nam, người trực cảm tuyệt vời phát biểu tường minh ý tưởng táo bạo tôpô từ 200 năm trước Khơng nghiên cứu tốn cầu hay toán số mặt số cạnh số đỉnh đa diện, tiếp cận đầy mẫn cảm, bà nhận chuyện từ xưa Bà viết thật nhân văn:

“Rộng hẹp nhỏ to vừa vặn cả Ngắn dài khuôn khổ nhau”.

Hai câu thơ trích “Dệt cửi” bà:

“Thắp đèn lên thấy trắng phau Con cò* mấp máy suốt canh thâu Hai chân đạp xuống năng nhắc,

(54)

Rộng hẹp nhỏ to vừa vặn cả Ngắn dài khuôn khổ nhau

Cô muốn tốt ngâm cho kỹ Chờ đến ba thu dãi màu.”

Như thế, Hồ Xuân Hương (1772–1822) độc lập gần đồng thời với L Euler (1707-1783), phát biểu tường minh quan điểm tôpô học Nữ sĩ họ Hồ khởi đầu đầy sinh khí cho đám hậu sinh làm tôpô Việt Nam, có kẻ học trị viết này:

“Mát mặt anh hùng tắt gió Che đầu quân tử lúc sa mưa”.

Theo Hồ nữ sĩ khó May sao,

"Mỏi gối chồn chân muốn trèo".

Vậy mà lại bảo nhà Tôpô hâm nghe quái được, hở giời

Vĩ thanh:

(55)

CÁC BÀI TOÁN ĐOÁN BÀI

Đặng Nguyễn Đức Tiến (Đại học Trento, Italy)

Sân khấu bừng sáng tiếng hò reo vang dội Và nhà ảo thuật ra, sang sảng nói

“Các bạn, bạn tơi ơi, trị ảo thuật hay đêm chờ đợi Đây, hãy chọn ngẫu nhiên Chọn nào, bạn ơi, ngẫu nhiên hay suy tư cẩn trọng chẳng sao.

Chọn nào, thật kín đáo khỏi mắt gian.

Lựa chọn xong giao cho người trợ lý bạn Rồi sẽ đưa lại bốn số cho tơi.

Xem nào, một:

Bảy bích, đầm cơ, tám chuồng, cuối ba rô.

Giờ tay người trợ lý lại một, mà Một mà chỉ anh ta người lựa chọn biết gì.

Nhưng mắt nhà ảo thuật, đọc thấu tâm can. Bạn tơi ơi, già bích!”

1 Năm Fitch Cheney

Tiếp nối số trước, chuyên mục giải trí kỳ trân trọng giới thiệu với độc giả loạt toán đố, kỳ toán đoán Như thường lệ, bắt đầu chuyên mục toán kinh điển toán khởi đầu lần toán năm Fitch Cheney Bài toán lần in Math Miracle tác giả Wallace Lee vào năm 1950 Trong này, tác giả ghi nhận tác giả toán William Fitch Cheney (1894 - 1974), gọi cách thân mật Fitch - nhà ảo thuật Theo tác giả, trò ảo thuật Fitch sáng tạo vào khoảng năm 1920 mượn chi tiết để viết lại thành lời tựa cho chuyên mục kỳ Để đơn giản cho việc tiếp cận toán, chúng tơi phát biểu lại tốn dạng tốn đố sau:

(56)

Thoạt nhìn, điều khơng thể với bài, tổ hợp thành4ŠD24trường hợp khác nhau, chuẩn có 52 Tuy nhiên, với chút mày mò kiên nhẫn, hẵn bạn đọc nhanh chóng tìm cách thức để khám phá bí mật đốn nhà ảo thuật Ở đây, giới thiệu cách giải đơn giản sau:

- Vì có bài, nên chắn tồn đồng chất (cùng cơ, rô, chuồn bích) Gọi làM vàN

- Vì chất có13lá khác nhau, nên ta ln tìm cách ‘đếm tới’ để khoảng cách của2

lá tối đa là6(và tối thiểu doM vàN chất, khơng thể nhau) Ví dụ nếuM D7

vàN DBồi khoảng cách là4vì sau7là8; 9; 10và bồi,4lá NếuM Dbồi vàN bằng3thì khoảng cách là5vì sau bồi đầm, già, át, hai, ba,5lá Khơng tính tổng qt, ta gánM cho vàN ‘đếm tới’

- Chiến thuật giữ lại láN đặt láM vị trí số 1.3lá cịn lại, ln có thứ tự lớn nhỏ ngầm định trước (ví dụ thứ tự chơi tiến lên lớn rơ, rơ lớn chuồn ) nên tạo thành

6hoán vị khác nhau, hoán vị ứng với số cho biết khoảng cách củaM vàN

- Như vậy, với cách hốn vị3lá cịn lại vị trí2; 3; 4và láM, ta biết chất củaN khoảng cách từM đếnN, nên xác định đượcN

Liệu ta làm tốt hơn, nghĩa tìm bằng4lá cịn lại lớn hơn, ví dụ100lá thay chỉ52lá chẳng hạn?

Hãy quay lại trị ảo thuật với nhìn tốn học chút: xem nhận từ trợ lý thông điệp, có52515049thơng điệp khác Nhà ảo thuật thấy phải đốn cịn lại, điều nghĩa nhà ảo thuật cần phải đoán tổ hợp chập 52 trường hợp Vì 52

5

!

D5251504948=5Šnên số lượng thông điệp mà nhà ảo thuật nhận dư so với số trường hợp, gấp5Š=48D2:5lần Và thật ta giải tốn với124lá khơng phải với 52

Chiến thuật cụ thể để với tìm cịn lại số 124 xem tập dành cho độc giả Gợi ý: dựa số dư chia cho

Tổng quát hóa tốn, chomlá lấy từ n, chọn giữ lạim 1lá để từ xác định đượcn lại khinmŠCm

Tiếp tục mở rộng, với ba.m; n; k/,n > m > k, vớimlá lấy từ bàinlá, chọn giữ lạiklá cho từ xác định đượcm klá lại khi:

n m

!

n.n 1/.n 2/ : : : n kC1/:

Với toán Fitch Cheney tương đương 3.52; 5; 4/:

(57)

Tạp chí Epsilon, Số 08, 04/2016

độ lớn cực đại cặp ghép đồ thị hai phía kích thước tập đỉnh phủ nhỏ (có thể dẫn tốn luồng cực đại - lát cắt cực tiểu), điều kiện tối ưu đạt

jSj D jTj

2 Bài toán 55 bài

Trong loạt tốn tiếp theo, chúng tơi trích chọn giới thiệu từ tác giả quen thuộc chuyên mục toán học giải trí, Stan Wagon qua tốn số 922, 1219, 1223

Và đây, toán 922, tốn 55 bài:

Có 55 bài, gán số khác từ đến 55 đặt ngẫu nhiên, úp mặt, thành vòng tròn Cứ lần ta lật xem số Tìm chiến thuật lật cho tìm chắn có giá trị lớn bên trái bên phải

Bài tốn Stan Wagon giới thiệu năm 2000, theo ơng bắt nguồn từ toán

Sharyginđăng trang cut the knot

Đáp án toán cần tối đa 10 lần mở bài, ta ln tìm có giá trị lớn hai hai bên

Chúng giới thiệu lời giải sau:

- Gọif x/là giá trị vị tríx Khi đó, xét vị trí.m; n; k/bất kỳ, ta ln tìm lớn cịn lại Khơng tính tổng qt, giả sử làn, bên trái

mvà bên phải làk(do xếp thành vịng trịn nên ta ln tìm thỏa mãn điều kiện này) Khi đóf m/ < f n/vàf n/ > f k/ Ta gọi có tính chất tính chấtS, nghĩa có giá trị lớn bên trái phải

- Giả sửjn mj> 1, nghĩa lámvànkhông nằm kế Chọn lát giữamvà

n Có hai trường hợp: Nếuf t / > f n/, ta có 3.m; t; n/có tính chấtS Nếuf t / < f n/, ta có 3.t; n; k/có tính chấtS Như vậy, dù trường hợp tạo vớit có số gần so với 3.m; n; k/ Và làm vậy, cuối ta thu bộ3mà khoảng cách bằng0, hay nói cách khác, là3lá liên tục thỏa mãn tính chấtS, yêu cầu đề

- Tiếp tục đơn giản hóa, với bộ.m; n; k/ta có khoảng cách làfn m 1; k n 1g, tức số Như mục tiêu tốn tìm có khoảng cáchf0; 0g

- Để rút đượcf0; 0gtrong tình xấu trước làf0; 1g (chúng ta xét tình xấu nhất, không quan tâm đến may rủi) Lần ngược tiếp tục, ta có lớn tình xấu đưa vềf0; 1glàf1; 2g Lưu ý, đây, khơng tính tổng qt, ta thấy khoảng cách trái hay phải nhau, nghĩa làf1; 2gtương ứng vớif2; 1g

(58)

lá ta mở thêm tối đa6lá chắn đưa được0; Nếuf c/lớn nhất, ta tốn thêm lần mở để đưa20; 20về12; 20 Trường hợp ta cần tối đa7lần mở

- Vậy tình xấu nhất, ta tốn tối đa 10 lần mở

Và, với cách làm trên, để ý ta thấy số tối đa vớinlần lật ta chắn tìm có giá trị lớn hai bên cạnh số Fibonacci thứn

3 Bài toán 1219

Bài toán Stan Wagon phát biểu toán cai ngục, chúng tơi phát biểu lại với dạng trị chơi đoán bài, sau:

A B tham gia trò chơi với luật sau:

A B mời vào phòng riêng để chờ gọi tham gia trò chơi

Đầu tiên, người dẫn trò mời A cho A xem chuẩn52lá xếp ngửa mặt thành hàng Sau xem xong, A có quyền chọn2lá bất kỳ, muốn đổi chỗ2lá Sau đó, người dẫn trị lật úp lại tồn (giữ nguyên thứ tự) lại mời B Người dẫn trị nói tên bàiT tùy ý, cho phép B lật lên để tìm bàiT Nếu sau tối đa26lần lật B tìm bàiT, họ chiến thắng trị chơi Ngược lại, sau26lần lật mà B không tìm láT, họ thất bại

A B q trình chơi khơng có trao đổi với nhau, họ trao đổi chiến thuật với trước chơi Hãy chiến thuật cho A B để họ ln thắng trò chơi

Số 26 đề gợi ý quan trọng việc tìm chiến thuật Trong phần này, chúng tơi giới thiệu chiến thuật dựa chu trình sau:

- Đầu tiên A B ngầm định đánh số từ1đến52 Khi B người dẫn trị cho biết cần tìm bàiT, B mở vị tríT Nếu láT, B tìm dừng Ngược lại, nu lỏ ú cú giỏ trf T /ÔT, B s mở tiếp vị tríf T /tương ứng tiếp tục gặpT

- Một chuỗi (bắt đầu từ vị tríx, sau mở tiếp vị tríf x/trên tạix

và tiếp tục gặp láx), gọi chu trình Và tồn bộ có 52 lá, nên tồn nhiều chu trình có độ dài hơn52=2D26 Do vậy, chiến thuật A tìm chu trình dài 26, có, đổi chỗ để cắt ngắn chu trình thành chu trình khơng vượt q26

- Việc "cắt" chu trình A thực đơn giản sau: khơng tính tổng qt, giả sử chu trình dài 26là1 N (nghĩa f 1/ D 2; f 2/ D 3; : : : ; f N / D 1), lúc ta đơn giản hốn đổi vị trí thứ26và thứN ta có chu trình ngắn

1 N và27 28 26

Một câu hỏi khó chúng tơi xem tập dành cho độc giả với cách làm chiến thuật trên, kỳ vọng để B tìm bàiT ngẫu nhiên số52lá bàiE.52/là bao nhiêu? Và tổng quát vớinlá kỳ vọng E.n/là bao nhiêu? Ví dụ ta cóE.2/ D 1,

(59)

Tạp chí Epsilon, Số 08, 04/2016

4 Bài toán 1223

Bài toán cuối mà giới thiệu chuyên mục lần phát triển toán trước, phát biểu sau:

A B tham gia trò chơi với luật sau:

A B mời vào phòng riêng để chờ gọi tham gia trò chơi

Đầu tiên, người dẫn trò mời A cho A xem5lá bài, đánh số từ đến xếp ngửa mặt thành hàng Sau xem xong, A có quyền chọn2lá bất kỳ, muốn đổi chỗ2lá Sau đó, người dẫn trị lật úp lại tồn (giữ nguyên thứ tự) lại mời B Người dẫn trị nói ngẫu nhiên số nguyênT tùy ý khoảng từ đến u cầu B tìm có số tương ứng số lật úp Nếu sau lần lật B tìm bàiT, họ chiến thắng trò chơi Ngược lại, họ thất bại

A B q trình chơi khơng có trao đổi với nhau, họ trao đổi chiến thuật với trước chơi Hãy chiến thuật cho A B để khả chiến thắng trò chơi cao

Ta thấy B chọn ngẫu nhiên lá, không quan tâm đến A, khả họ thắng trị chơi

1=5 Nếu A ln đổi chỗ cho nằm vị trí chiến thuật B

T D1thì mở vị trí 1, ngược lại mở ngẫu nhiên vị trí từ đến Lúc này, xác suất chiến thắng là2=5D40%

Nếu A B sử dụng chiến thuật cắt chu trình tốn trước, xác suất chiến thắng họ bao nhiêu? Ta thấy, với chiến thuật ngày B mở vị tríT người dẫn trị cho biết cần tìm làT Stan Wagon, vậy, ký hiệu cho chiến thuật (12345) ứng với vị trí mở tương ứng Chiến thuật A sau: A gặp cặp bị hoán đổi vị trí, A hốn đổi vị trí cặp lại cho Ngược lại (nghĩa khơng có cặp bị hốn đổi vị trí), A tìm chu trình có độ dàik 3và cắt thành chu trình này, có, thành chu trình ngắn có độ dài

k 1và rời Dễ thấy, khơng có chu trìnhk, nằm vị trí từ đến Với cách làm có tất 284 trường hợp B tìm láT số5:5Škhả năng, xác suất để họ thắng trò chơi 284

5:5ŠƯ D47

1

3%, lớn hơn7

3% so với cách làm

Stan Wagon đưa cách tìm cơng thức tổng qt để tính số trường hợp thành cơngf n/vớinlá với chiến thuật là:

f n/D2:nŠ 1CT n/

vớiT n/là số lượng hoán vị củansố ngun có tồn cặp bị hốn đổi vị trí Một vài giá trịT n/đầu tiên 0, 1, 3, 9, 45, 285, 1995 Trong trường hợp đầu bàin D5, ta cóf 5/ D2:5Š 1CT 5/ D 2:120 1C45 D284 Chứng minh chi tiết T(n) f(n) xem thêm trang nhà Stan Wagon

(60)

Tổng số khả thành công theo chiến thuật là286và xác suất thắng lợi là472 3%,

cao

3% so với chiến thuật (12345) Chiến thuật Stan gọi double-door (cửa đôi)

Một câu hỏi mở chưa giải đáp với chuẩn52lá, chiến thuật tốt nhất? Và vớintổng quát? Hiện tại, kết hợp khảo sát máy tính, Stan Wagon, tác giả tốn chứng minh đề xuất cách làm tối ưu chontrong phạm vi từ đến 10

Tham khảo trích dẫn

Độc giả tham khảo chi tiết cho toán giới thiệu chuyên mục lần địa tài liệu sau:

1 Các toán Stan Wagon tạihttp://mathforum.org/wagon/

2 Kleber, M., The Best Card Trick Mathematical Intelligencer, 24 (2002)

(61)

XUNG QUANH ĐỊNH LÝ BROKARD

Nguyễn Trần Hữu Thịnh

(Trường THPT Chuyên Lý Tự Trọng, Cần Thơ)

Bài viết nói vấn đề xoay quanh định lý Brokard quen thuộc trình bày cơng cụ hình học phẳng túy

1 Mở đầu

Định lý Brokard [1] nói tam giác có định giao điểm cặp đường thẳng tạo tứ giác nội tiếp nhận tâm ngoại tiếp tứ giác trực tâm Bản thân Brokard viên ngọc quý có nhiều ứng dụng thi học sinh giỏi nước Sau xin trình bày lại cách chứng minh định lý thơng qua bổ đề có nhiều ứng dụng sau:

Định lý Brokard. Cho tứ giác lồi ABCD khơng hình thang nội tiếp đường trịn tâm O GọiE; F; G giao điểm củaAB vàCD,AD vàBC,AC vàBD Khi đóO trực tâm tam giácEF G Lời giải sau rút từ ý tưởng thầyĐỗ Thanh Sơnvề vấn đề giao điểm tiếp tuyến đường tròn

Ta có bổ đề sau:

Bổ đề 1. Gọi H; I; J; K; L; M theo thứ tự giao điểm cặp đường thẳng dAIdB/; dAIdC/; dAIdD/; dBIdC/; dBIdD/; dCIdD/1 Khi điểm.IIEILIF /,.FIMIGIH /,

.EIKIGIJ /thẳng hàng

Chứng minh. Do vai trò củaE vàF bình đẳng đường thẳngILnên ta giả sửE

nằm đoạn thẳngIL GọiE giao điểm củaABvàIL Áp dụng định lý Menelaus cho tam giácIHLvới cát tuyếnABE được:

EI EL:

BL BH:

AH

AI D1hay EI EL D

AI BL

GọiE0là giao điểm củaCDvàIL Tương tự ta được:

E0I E0L D

CI DL D

AI BL

Điều chứng tỏE vàE0chia đoạnILtheo tỉ số Do đóE trùngE0 Nên vớiE giao điểmABvàCD thìI; E; Lthẳng hàng

Tương tựI; L; F thẳng hàng Vậy điểm.IIEILIF /thẳng hàng

1

(62)

Với cách chứng minh ta suy điểm.FIMIGIH /,.EIKIGIJ /

thẳng hàng

Một cách khác, ta áp dụng định lý Pascal cho lục giác suy biến thành tứ giácABCDđể suy điểm nêu thằng hàng

Quay lại tốn,

Chứng minh. Ta cóBOCK vàAODJ tứ giác nội tiếp Mặt khác tứ giácABCD

nội tiếp nên:

F C :FB DFD:FA

NênF nằm trục đẳng phương của.BOCK/và.AODJ /nênOF ?KJ Áp dụng bổ dề đượcEG ?OF

Tương tựF G ?OE Do đóO trực tâm tam giácEF G Vậy ta có điều phải chứng minh

Nhận xét.Việc chứng minh điểm thẳng hàng giúp hướng giải sáng sủa đưa tốn trở với tính chất quen thuộc vng góc đường nối tâm trục đẳng phương hai đường trịn Với tốn thú vị ta xét đến mở rộng ứng dụng quan trọng toán quen thuộc đề thi học sinh giỏi số nước

2 Khai thác định lý

(63)

Tạp chí Epsilon, Số 08, 04/2016

Bài toán (China MO 1997) Cho tứ giácABCD với cạnh đôi không song song nội tiếp GọiP; Qlần lượt giao điểm củaAB CD,AD BC GọiQE; QF lần lượt tiếp tuyến tạiE; F của đường tròn ngoại tiếp tứ giácABCD Chứng minh rằngP; E; F thẳng hàng.

Ta thấy nếuO tâm đường tròn ngoại tiếp tứ giácABCDthìOQ?EF Mặt khác theo định lý Brokard thìP G ?OQvớiGlà giao điểm củaAC vàBD Do nếuE; G; F thẳng hàng thìP; E; F thẳng hàng, ta xét bổ đề sau

Bổ đề 2.Cho tứ giácABCDkhông hình thang nội tiếp.O/.M; T giao điểm

AB vàCD,AC vàBD VẽME; MF tiếp tuyến đường tròn.O/vớiE; F tiếp điểm Chứng minh rằngE; F; T thẳng hàng

Chứng minh. GọiH giao điểm củaOM vàEF Ta có:

MA:MB DME2 DMH :MO

Do tứ giácABHO nội tiếp Tương tự tứ giácCDOH nội tiếp Suy ra:

\

BH C DBHM\ CMH C\ DBAO[ CODC\ )BH C\ DBT C\

Nên tứ giácBH T C nội tiếp Tương tự tứ giácAH TDnội tiếp Ta có:

\

MH C DODC\ DODH\ vàTH C\DTBC[ DTAD[ DTHD\

Do đó:

\

MH T D

\

MH C CTH C\CTHD\COHD\D90ı

NênTH ?OM Do đóTH; EF trùng nhau, tức ba điểmE; F; T thẳng hàng Ta có điều phải chứng minh

(64)

Chứng minh. Từ bổ đề trên,ETF ?OQvà theo định lý Brokard ta cóP T ?OQ, ta có đượcE; F; P thẳng hàng

Bài toán giải xong

Nhận xét.Lời giải sử dụng định lý Brokard ”chiếc cầu” để liên kết điểm với Hơn sau thông qua lời giải, rõ ràng tốn vận dụng khéo léo tính chất tiếp tuyến trình bày ngắn gọn thơng qua hình học phẳng túy

Nói tứ giác hẳn vấn đề quan tâm khơng tứ giác toàn phần với điểm Miquel tứ giác này, chúng có liên hệ với định lý Brokard nào, ta xét hai toán sau:

Bài toán (IMO 1985) Cho tam giácABC Một đường tròn tâmO đi quaA; C và cắt các đoạnAB; BC theo thứ tự hai điểmK; N phân biệt Giả sử M là giao điểm thứ hai của ABC /.KBN / Chứng minhOMB\ D90ı.

Ta để ý thấyM điểm Miquel tứ giác toàn phầnACPNFB Chứng minhOMB\ D 90ı

tức chứng minhM chân đường cao kẻ từO xuốngBP Mặt khác theo định lý Brokard

OG đường cao tam giácGBP vớiGlà giao điểm củaAN vàKC Vậy ta cần chứng minhO; G; M thẳng hàng xong

Chứng minh. GọiM0là chân đường cao kẻ từO xuốngBP

Gọi D; E giao điểm dCIdK/; dAIdN/ Theo bổ đề định lý Brokard

E; B; M0; D; P thẳng hàng

Từ ta dễ dàng suy ngũ giácAEM0NO vàKM0DCO nội tiếp Nên:

\

AM0C DAM\0O COM\0C DANO\COKC\D180ı \KAC NCA[ D\ABC

Do tứ giácABM0C nội tiếp Tương tựKBM0N nội tiếp NênM0là giao điểm thứ hai

(65)

Tạp chí Epsilon, Số 08, 04/2016

Nhận xét.Một tốn khơng q khó tiếp tục áp dụng bổ đề quen thuộc Song ta tiếp tục phát tính chất hay đây,.AKNC /; AONM /; KOCM /đều nhận

Glà giao điểm hai đường chéo Điều dễ dàng thu thông qua khái niệm tâm đẳng phương ba đường trịn Vậy ta chứng minh định lý Brokard kết hay không? Ta quay lại với định lý thú vị này:

Chứng minh. GọiN; P chân đường cao kẻ từO; E xuốngEF; FO

Khi ngũ giácOBNKDvàAOCNH nội tiếp Mặt khác ta có tứ giácABCDnội tiếp Áp dụng kết ta đượcAC; BD; ON đồng quy tạiGhayO; G; N thẳng hàng

(66)

ngũ giácAMDPO nội tiếp Do đó:

[

ACE DACB[ CBCE[ DACB[ CBAD[ D90ıCADO\D90ıCAPO[ DAPE[

Suy tứ giácAP CE nội tiếp Theo ta đượcEBPD nội tiếp Áp dụng kết ta

AC; BD; EP đồng quy tạiGhayE; G; P thẳng hàng Vậy ta có điều phải chứng minh

Nhận xét.Lại thêm phát kiến thú vị định lý thông qua tâm đẳng phươngGcủa sáu đường trịn Thậm chí, quaBài tốn 2, ta cịn thấy điểm Miquel tứ giác tồn phần chân đường cao kẻ từ tâmOxuống cạnhEF Không vậy, ta thu tính chất với nhiều ứng dụng sau:

Tính chất 1. Chân ba đường cao tam giác EF G ba điểm Miquel tứ giác tồn phầnABECDF

Tính chất 2.TâmO đường tròn ngoại tiếp tứ giácABCD nằm đường trịn Miquel tứ giác tồn phầnABECDF

Ta lại nghĩ đến tính chất đường thẳng Steiner tứ giác qua trực tâm, điểmG có quan hệ với đường khơng? Ta có tốn sau:

Bài toán Cho tam giác nhọnABC Đường tròn tâmO đi quaB C cắtAB; AC lần lượt tạiM; N Gọi P là giao điểm củaBN CM GọiH; K lần lượt trực tâm tam giác ABC và tam giácAMN Chứng minh rằngP; K; H thẳng hàng.

Ta nhận thấyK; H nằm đường thẳng Steiner nênP phải nằm đường Nhìn vào giả thiết đề ta khó định hướng lối đi, ta để ý đến tính chất đường thẳng Steiner tứ giác tồn phần thơng qua bổ đề sau:

Bổ đề 3. Cho tứ giác toàn phần ABF CDE Chứng minh đường thẳng Steiner tứ giác trục đẳng phương chung của.AC /; BD/và.EF /

Chứng minh. GọiH1; H2là trực tâm củaCDE; ABE

GọiH; I hình chiếu củaH1lênED; CD Ta thấyH 2.AC /; I EF / Ta có tứ giácHH1ID vàCHIEnội tiếp nên:

H1H :H1C DID:I C DHD:HE DH1I :H1E

Như vậyH1 nằm trục đẳng phương của.AC /và.EF / Tương tựH2cũng nằm trục đẳng phương của.AC / và.EF / Hoàn toàn theo cách ta suy raH1H2 trục đẳng phương của.BD/và.EF /

Ta kết luận đường thẳng Steiner tứ giác toàn phầnABF CDElà trục đẳng phương chung của.AC /; BD/và.EF /

(67)

Tạp chí Epsilon, Số 08, 04/2016

Với bổ đề quan trọng này, toán trở nên nhẹ nhàng hơn, ta trở lại:

Chứng minh. VìP giao củaBN vàCM nên:

PM :P C DPN :PB

Suy raP nằm trục đẳng phương của.BN /và.CM / Áp dụng bổ đề ta thu đượcP nằm đường thẳng Steiner tứ giác toàn phầnAMBPNC Như vậyP; K; H thẳng hàng Vậy ta có điều phải chứng minh

Nhận xét.Bài toán khéo léo vận dụng ”viên ngọc q” tứ giác tồn phần, tứ đưa ta đến tính chất khác định lý Brokard:

(68)

ABECDF

Do định lý Brokard cho ta hệ thống tam giác trực tâm nên thơng qua tốn sau ta lại có thêm tính chất đẹp sau

Bài tốn (Romanian Master of Mathematics 2013) Cho tứ giácABCDkhơng hình thang nội tiếp đường tròn! GọiP; Q; Rlần lượt giao điểm củaAB CD,ADBC,AC BD GọiM là trung điểm củaPQ.MRcắt! tạiK Chứng minh đường tròn ngoại tiếp tam giácKPQ!tiếp xúc nhau.

Đây toán hay khó, ta sử dụng hai bổ đề, trước hết đường thẳng vng góc với

ORtạiR cắt!tạiP1; Q1 hình vẽ

Bổ đề 4.AP1cắtCQ1tại điểm thuộcPQ

Chứng minh. GọiX; Y giao điểm củaAP1vớiCQ1,AQ1vớiCP1

Áp dụng định lý Pascal cho lục giác suy biến thành tứ giácAP1CQ1 ta dễ dàng suy đường thẳngX Y chứaP vàQnênAP1vàCQ1cắt điểm thuộcPQ

Ta có điều phải chứng minh

Bổ đề 5.PP1; QQ1cắt tạiK

Chứng minh. PP1cắt! tạiP2 Ta chứng minhP2Q1đi quaQ

(69)

Tạp chí Epsilon, Số 08, 04/2016

vàAD,Q1C vàAP1cùng với giao điểm củaADvàP1P2 làP thẳng hàng Mặt khác theo bổ đề ta cóQ1C cắtAP1tại điểm thuộcPQ, giao điểm củaP2Q1vàAD nằm trênPQ, tức điểmQ VậyP2Q1đi quaQ

Ta có điều phải chứng minh Quay trở lại toán,

Chứng minh. Áp dụng bổ đề bổ đề 5, ta có tứ giácPQP1Q1là hình thang,M; Rlần lượt trung điểm củaPQ; P1Q1vàK giao điểm củaPP1; QQ1nên ta dễ dàng suy đường tròn! ngoại tiếp tam giácKP1Q1vàKPQtiếp xúc

Vậy toán giải xong

Nhận xét.Việc sử dụng khéo léo định lý Pascal giúp ta giải gọn gàng toán Như ta biết, tư giác nội tiếp khơng hình thang tạo tam giác trực tâm, ta tạm gọi theo ánh xạ tam giác trực tâm ảnh tứ giác nội tiếp thông qua ánh xạ Brokard Nhưng điều ngược lại chưa đúng, tức tam giác trực tâm có nhiều tạo ảnh Cụ thể, ta xét tam giác nhọnOEF, trực tâmG, đường caoEK Bằng cơng cụ phương tích, ta dựng đường trịn tâmO bán kínhpOE2 GE:KE, đường trịn ngoại tiếp các

tạo ảnh mà ta đề cập đến, với cát tuyến từP cắt.O/tại hai điểm cho ta tạo ảnh tam giác trực tâmOEF

Từ nhận xét này, ta có cách phát biểu rộng cho toán sau

Bài toán Cho tam giácABC trực tâmH Đường trịn đường kínhAB; AC cắt tạiX. Gọi!là đường trịn tâmAbán kínhAX GọiM là trung điểm củaBC Đường thằngMH cắt !tạiE; F Chứng minh đường tròn ngoại tiếp tam giácBCE BCF tiếp xúc!

Bài tốn cịn có thêm tính chất sau

(70)

Các toán trên, đặc biệt bổ đề nêu sử dụng định lý Pascal để giải vấn đề Như thế, việc phát triển từ tứ giác nội tiếp đường trịn sang tứ giác có bốn điểm nằm conic hồn tồn Phần tác giả xin giới thiệu việc vận dụng định lý Pascal cho bốn điểm nằm conic, từ cho thấy chất định lý Brokard tứ giác nội tiếp đường trịn Ta có bổ đề sau phát triển rộng bổ đề nêu đầu tài liệu

Bổ đề 6.Cho bốn điểmA; B; C; Dnằm conicC tạo thành tứ giác lồi khơng hình thang.M; N giao điểm củaAD vàBC,AB vàCD.P; Qlà giao điểm tiếp tuyến củaC tạiAvàC,B vàD Khi bốn điểmM; N; P; Qthẳng hàng

Chứng minh. Áp dụng định lý Pascal cho lục giác suy biến thành tứ giácAABC CD ta có

M; N; P thẳng hàng Áp dụng định lý Pascal cho lục giác suy biến thành tứ giácABBCDDta cóM; N; Q thẳng hàng Tóm lại ta cóM; N; P; Qthẳng hàng

Ta có điều phải chứng minh

Từ bổ đề ta có tiếp hệ sau

Hệ quả.Cho conicC tiếp xúc cạnhAB; BC; CD; DAcủa tứ giác ABCD theo thứ tự

M; N; P; Q Khi đóAC; BD; MP; NQđồng quy

Rõ ràng bổ đề mở rộng đẹp bổ đề nêu đầu ellipse, parabol hay hyperbol Trong trường hợp ellipse, cần thực phép co thành hình trịn ta lại có ứng dụng mẻ Bổ đề mở rộng củaBổ đề 2:

GọiA1A2A3A4là tứ giác không hình thang nội tiếp conicCvàM1; M2; M3lần lượt giao điểm củaA1A2 vàA3A4,A2A3 vàA4A1,A3A1 vàA2A4 GọiN1; N2; N3; P1; P2; P3

là giao điểm tiếp tuyến conic A1 A2,A1 A4,A1 A3, A3 A4, A2

A3,A2 A4 Gọi U1; U2 tiếp điểm hai tiếp tuyến conic qua M1 Định nghĩa tương tự V1; V2 M2 W1; W2 M3 Bổ đề 1 cho ta ba điểm thẳng hàng là.M2IM3IN1IP1/; M3IM1IN2IP2/,.M1IM2IN3IP3/ Lần lượt gọi ba đường thẳng

(71)

Tạp chí Epsilon, Số 08, 04/2016

Bổ đề 7.U1; U2 nằm trênm1,V1; V2 nằm trênm2vàW1; W2 nằm trênm3

Chứng minh. Trong mặt phẳng xạ ảnh tồn phép chiếu' biến conicC thành đường trònC0 Như ta biết, phép chiếu' bảo toàn thẳng hàng điểm nên thông qua phép biến hình ta có ngayBổ đề 2và theo điểm điểm.M2IM3IU1IU2/; M1IM3IV1IV2/; M1IM2IW1IW2/thẳng hàng

Ta có điều phải chứng minh

Một kết đẹp lại chứng minh GọiO tâm conicC, từ kết ta xét tam giác

(72)

Khi conic C đường trịn theo tính chất tiếp tuyến ta có m1 ? OM1

m2 ?OM2 ta hiểu định lý Brokard lại có phát biểu Cuối cùng, tác giả xin đề xuất số tập để bạn đọc rèn luyện thêm tư khác định lý Brokard

3 Ứng dụng

Bài toán (Trường hè Bắc Trung Bộ 2015 - Ngày 2) Cho tam giác nhọnABC có đường caoAH, trực tâmK Đường thẳngBK cắt đường trịn đường kínhAC tạiD; E(BD < BE). Đường thẳngCK cắt đường trịn đường kínhAB tạiF; G(CF < C G) Đường trịn ngoại tiếp tam giácDHF cắtBC tại điểm thứ hai làP.

a) Chứng minh điểmG; H; P; E thuộc đường tròn b) Chứng minh đường thẳngBF; CD; PK đồng quy

Bài toán Cho tứ giác lồiABCDkhơng hình thang nội tiếp đường tròn tâmO GọiE; F; G lần lượt giao điểm củaAB CD,AD BC,AC BD GọiX; Y theo thứ tự trung điểm củaAC; BD Kéo dàiX Y cắtEF tạiZ Kéo dàiGZ cắt đường tròn ngoại tiếp tam giác OFE tạiQ Đường trịn đường kínhGQcắt đường trịn ngoại tiếp tam giácOFE tạiK khác Q GọiH là điểm Miquel tứ giác tồn phần ABCDEF Chứng minh đường trịn ngoại tiếp tam giácKQG KZH tiếp xúc nhau.

Bài toán (Lê Bá Khánh Trình) Cho tam giácABC nội tiếp.O/B; C cố định,Achạy trên cung nhỏ BC. M; N là trung điểm AB; AC Lấy P trên đường thẳng MN thỏa mãn

\

BP C C\ABC CACB[ D BAC[ Đường tròn ABP / cắt AC tại E, ACP /cắt AB tại F. Chứng minh rằng.AFE/đi qua điểm cố định.

Bài tốn 10 (Lê Bá Khánh Trình) Cho tam giácABC nội tiếp.O/, trung tuyếnAI cắt.O/ tạiD.AB cắtCDtạiE.AC cắtBDtạiF..ABF /cắt.ACE/tạiK O1/; O2/là đường tròn quaB tiếp xúc vớiAC tạiAvà quaC tiếp xuc vớiAB tạiA Chứng minh ba đường trịn.O1/; O2/.OK/có điểm chung.

Bài toán 11 Cho tam giácABC nội tiếp.O/, điểmP nằm tam giác choAP vng gócBC Đường trịn đường kínhAP cắt đường thẳngAC; AB lần lượt tạiE; F và cắt.O/tại điểmG khácA Chứng minh rằngBE; CF; GP đồng quy.

Bài tốn 11 xem [6]

Bài toán 12 (USA TST 2012) Cho tam giácABC nhọn, đường caoAA1; BB1,C C1 GọiA2 là giao điểm củaBC B1C1 Định nghĩaB2; C2 tương tự GọiD; E; F theo thứ tự trung điểm củaBC; CA; AB Chứng minh đường thẳng vng góc kẻ từDtớiAA2,E tớiBB2 F tớiC C2đồng quy.

(73)

Tạp chí Epsilon, Số 08, 04/2016

Bài toán 14 (Trần Quang Hùng) Cho tam giác ABC Đường tròn tâm K đi qua B; C cắt CA; AB lần lượt tạiE; F.BE; CF cắt tạiH.Dlà hình chiếu củaK xuốngAH.M; N theo thứ tự di chuyển trênDE; DF sao choBM ?BE; CN ?CF Chứng minh đường đối trung xuất phát từ đỉnhAcủa tam giácABC chia đơi cạnhMN.

Bài tốn có tham khảo [7] Hơn ta cịn có thêm hai mở rộng sau

Bài toán 15 (Trần Quang Hùng) Cho tam giác ABC và điểmP.AP; BP; CP lần lượt cắt BC; CA; ABtạiD; E; F TrênDE; DF lần lượt lấyM; N sao choBM kAC; CN kAB Gọi S; T theo thứ tự trung điểm củaEF; MN Chứng minh rằngA; S; T thẳng hàng.

Bài toán 16 (Trần Quang Hùng) Cho tam giácABC và điểmD nằm cạnhBC..DAB/; DAC /cắtCA; AB lần lượt tạiE; F.M; N nằm trênDE; DF sao choBM kAC; CN kAB GọiS; T lần lượt trung điểm củaEF; MN Chứng minh rằngS T song song với đường đối trung xuất phát từ đỉnhAcủa tam giácABC.

Bài toán 17 (Chọn đội tuyển quốc gia Khoa học tự nhiên 2014) Cho tứ giácABCDnội tiếp. M; N lần lượt trung điểm củaCD; AB.P nằm trênCDsao cho PD

P C D BD2

AC2.AC cắtBD tạiE GọiH là hình chiếu củaE lênPN Chứng minh đường trịn ngoại tiếp tam giác HMP EDC tiếp xúc nhau.

Trên tổng hợp tác giả định lý Brokard, xin cảm ơn anhNgô Quang Dương- lớp 12A2 trường THPT chuyên KHTN có trao đổi quý báu giúp viết hoàn thiện

Tài liệu tham khảo

[1] Định lý Brokard

http://www.imomath.com/index.php?options=334&lmm=0

[2] Đỗ Thanh Sơn,Một số chuyên đề Hình học phẳng bồi dưỡng học sinh giỏi Trung học phổ thông, NXB Giáo dục, 2013

[3] Nguyễn Văn Nho,Những định lí chọn lọc hình học phẳng qua kì thi Olympic, NXB Giáo dục, 2007

[4] Romanian Master of Mathematics 2013 - Problem

http://www.artofproblemsolving.com/community/c6h523072p3558896 [5] IMO 2015 - Problem

http://www.artofproblemsolving.com/community/c6h1112748p5079655 [6] Concurrent problem

http://www.artofproblemsolving.com/community/c6h1161051p5527875 [7] Symmedian bisects segment

(74)(75)

TỨ GIÁC NGOẠI TIẾP ĐƯỜNG TRÒN

Đỗ Xuân Anh

(Trường THPT Chuyên KHTN, Hà Nội)

Tứ giác ngoại tiếp chủ đề không đam mê với mơn tốn đặc biệt mơn hình học có khơng nhiều tài liệu viết chủ đề Vậy nên viết xin đề cập đến vấn đề với kiến thức ứng dụng tứ giác ngoại tiếp

1 Một số tính chất tứ giác ngoại tiếp đường tròn

Khi nhắc tới tứ giác ngoại tiếp đường tròn, nên để ý đến tính chất hay sử dụng sau:

Cho tứ giácABCDngoại tiếp đường tròn.I / I /tiếp xúcAB,BC,CD,DAlần lượt tạiM,

N,P,Q

ĐặtAM DAQDa,BM DBN Db,CN DCP Dc,DQDDP Dd

Định lý 1.(Định lý Pithot)ABCCDDBC CDA

Định lý 2.

1 (Định lý Newton)AC,BD,MP,NQđồng quy tạiT

2 AT

C T D a c ,

BT DT D

b d :

T

D Q

P

N M

I A

B

C

(76)

Ta chứng minhT1 T2 T Thật vậy, áp dụng định lí sin, ta có:

T1A T1C D

AM

sin∠AT1M

sin∠C T1M

CP

sin∠AMP

sin∠CPM D AM

CP

Tương tự, T2A

T2C D a

c nênT1,T2,T trùng Tính chất chứng minh

Định lý 3.AC,MN,PQđồng quy tạiV AV

C V D a c,

BV DV D

b

d Từ suy ra.AC; T V /D

V

T

D

Q

P

N M

I

A

B

C

Chứng minh. LấyV trênAC cho AV

C V D a

c Áp dụng định lí Menelaus cho4ABC ta có M,N,V thẳng hàng Tương tự suy raP,Q,V thẳng hàng Vậy tính chất chứng minh

Định lý 4.Đường thẳng quaAvuông góc vớiABcắtBI tạiX, đường thẳng quaAvng góc vớiADcắtDI tạiY thìX Y vng góc vớiAC

F

E Y

X

D

I

A

B

(77)

Tạp chí Epsilon, Số 08, 04/2016

Chứng minh. GọiF hình chiếu củaX lênBC;E hình chiếu củaY lênCD Ta cóAX2 XC2 DAX2 XF2 F C2 D F C2

AY2 Y C2DAY2 YE2 EC2 D EC2:

MàF C DBC AB DDC AD DEC nênAX2 XC2 DAY2 Y C2

XA2CC Y2 AY2 CX2DXA2CAC2 CX2 AY2CAC2 C Y2/ D2AX!AC! 2AY!AC!

D2AC!YX!

Do đóAC ?X Y Vậy ta chứng minh xong định lí

Ngoài ra, nên biết số cơng thức tính yếu tố tứ giác ngoại tiếp đường tròn Việc chứng minh xin dành cho bạn đọc

Trong tính chất đây, ta đặtr bán kính đường trịn nội tiếp tứ giácABCDvà

∠DAB D˛; ∠ABC Dˇ; ∠BCDD; ∠CDADı:

Định lý 5.AB D NQ :

IA:IB r2 :

B' A'

D Q

P

N M

I A

B

C

Chứng minh. GọiIA,IB cắtQM,MN tạiA0,B0Ta cóIA:IA0DIB:IB0Dr2nên

4IA0B0 4IBAsuy A

0B0 AB D

IA0

IB Từ đó,AB D

IB:A0B0 IA0 D

IB:IA r2 :

NQ

2 Vậy định lí

được chứng minh

Định lý 6. IB

2 AB:BC D

(78)

D Q P N M B A I C

Chứng minh. Từ định lí 5, ta có CD D I C:ID r2 :

NQ

2 suy AB CD D

IA:IB

I C:ID Tương tự BC

AD D

IB:I C

IA:ID suy AB CD

BC AD D

IB2 ID2 hay

IB2 AB:BC D

ID2 CD:DA

Định lý 7.IA:I C CIB:IDDpAB BC CDDA

P' M' D Q P N M I A B C

Chứng minh. Gọi M0, P0 đối xứng M, P qua I Ta có4MIB 4NM0M suy M

0N MI D

MM0

IB hay M

0N D MM0:MI

IB D

2r2

IB Tương tự M

0Q D 2r IA, P

0Q D 2r ID, P0N D 2r

2

I C Áp dụng định lí Ptoleme cho tứ giác P

0NM0Qvà định lí 5, suy ra IA:I C C IB:ID DpABBC CDDA Vậy định lý chứng minh

Định lý 8.IA:I C D aCc/:r

(79)

Tạp chí Epsilon, Số 08, 04/2016

D Q

P

N M

I A

B

C U

Chứng minh. LấyU tiaMB choM U DNC suy raI U DI C nên

SAI U D

2.aCc/:r D

2IA:I U:sin∠AI U D

2IA:I C:sin ˛C

2

hayIA:I C D aCc/r

sin˛C2 Vậy định lí chứng minh

Định lý 9.SABCD DpABBC CDDA:sin˛C

2 :

D Q

P

N M

I A

B

C

Chứng minh. Theo định lí 8, ta cóIA:I CCIB:IDD aCbCcCd /r

sin˛C2 D

p

AB:BC:CD:DA

suy SABCD D a Cb CcCd /r D pABBC CDDA:sin˛C

2 Ta có điều phải

chứng minh

2 Một số tập áp dụng

(80)

D E

H

G F

I

B C

A

Chứng minh. Gọi FH cắt BC G Dễ thấy E; I; G thẳng hàng F; I; D thẳng hàng.Ta có ∠BEG D ∠BEI D ∠IEF D ∠GEF D ∠EGB nên tam giác BEG cân B, suy

BE DBG Do đóBEH DBGH (c.g.c) vậy∠BEH D∠BGH D90ı hayEH vng gócEB E Áp dụng định lý 4, suy DH vng góc với EC Vậy tốn chứng minh

Nhận xét.Chúng ta sử dụng tính chất để chứng minh toán Mathley [5] thầyTrần Quang Hùngnhư sau

Bài toán. 4ABC, đường tròn nội tiếp I /tiếp xúcCA,AB tạiE,F LấyP di chuyển

EF.BP cắt CAtại M,MI cắt đường thẳng quaC vng gócAC tạiN Chứng minh đường thẳng quaN vng góc vớiP C ln qua điểm cố định khiP di chuyển

Bài toán Cho tứ giácABCD ngoại tiếp đường tròn.I /.AB giaoCDtạiE,AD giaoBC tạiF Chứng minh đường tròn nội tiếp4AEF,4CEF tiếp xúc điểm trênEF.

X=Y

B

C

A Q

S P

R I

E

F

(81)

Tạp chí Epsilon, Số 08, 04/2016

Chứng minh. I /lần lượt tiếp xúcAB,BC,CD,DAtạiP,Q,R,S; đường tròn.I1/và.I2/

lần lượt đường tròn nội tiếp4AEF và4CEF, hai đường trịn tiếp xúcEF tạiX,Y Ta có

F C CE DFQCQC CR RE DF S EP DFACAS AP EADFA EA

Mặt khác

8 ˆ <

ˆ :

FX D

2.FECFA EA/ F Y D

2.FECF C CE/

nên,FX DF Y suy raX Y Vậy toán chứng minh

Nhận xét.Ngoài ra, để ý kĩ ta thấy hình vẽ có hai tam giác có đường trịn nội tiếp tiếp xúc là4ABC,4ADC (Tương tự cặp tam giác4ABD,4CBD)

Trong toán, ta để ý rằngF C CE DFA EAchính định lý Pithot mở rộng cho tứ giác lừm ngoại tiếp đường trịn Áp dụng định lý ta chứng minh toán sau

Bài toán.Tứ giácABCDngoại tiếp đường tròn.I /.AB,ADlần lượt cắtCD,BC tạiE,F GọiEP,FM cắtBC,CDtạiQ,R;EP cắtFM tạiN Chứng minh tứ giácAMNP

ngoại tiếp thìQNRC ngoại tiếp

Bài tốn đề nghị A.Golovanov diễn đàn AoPS [1]

(82)

F E

W U

P

D Y

X

V A

C B

Chứng minh. AD cắtBC tạiP, đường tròn.E/ đường tròn nội tiếp4PAB, tiếp xúc với

AB tạiF.U V cắt đường thẳng quaC vng gócCD tạiW0 Ta có4PAB 4P CD(g.g) vàX,F đối xứng với qua trung điểm củaAB Nên U V

W0V D DY Y C D

BF FA D

AX

XB, suy AU kX V kBW0 Từ đóW0W Vậy tốn chứng minh

Tiếp theo kết tiếng tứ giác ngoại tiếp

(83)

Tạp chí Epsilon, Số 08, 04/2016

N M

B

A

D F

E

I

C G

H

Chứng minh. GọiM,N trung điểm củaAC,BD Ta cóABCCD DADCBC

nênSIAB CSICD DSIADCSIBC D

2SABCD Ta xét trường hợp:

Trường hợp 1.AB kCD dễ thấy trung điểmAC,BD vàI cách đềuAB,CD nên chúng thẳng hảng

Trường hợp Giả sửABcắtCDtạiE LấyGtrên tiaEA, lấyH tiaEDsao choEG D AB,EH DCD Ta cóSIE G CSIEH DSIAB CSICD D

2SABCD nênSIHG D

2SABCD SEHG Dễ thấy điểmM,N có tính chất tương tự nên

8 ˆ <

ˆ :

SNAB CSNCD D

2SABCD SMAB CSM CD D

2SABCD

Suy

ra

8 ˆ <

ˆ :

SNHG D

2SABCD SEHG SMHG D

2SABCD SEHG

Do đóSIHG DSMHG DSNHG nênI,M,N thẳng hàng Vậy toán chứng minh

Tơi đề xuất thêm tốn sau để bạn đọc ứng dụng định lý

Bài toán.Tứ giácABCD ngoại tiếp đường tròn.I / Đường tròn.I /tiếp xúcAB,BC,CD,

DAtạiE,F,G,H Chứng minh nếuI thuộcFH thìI thuộcEG Bài toán sau tham khảo [2]

(84)

L K

N M

I2

C B

I1

D A

Q P

S

T

Lời giải theo Luis González. Ta sử dụng bổ đề

Bổ đề 1. 4ABC ngoại tiếp đường tròn I / Đường tròn bàng tiếp Ia/ ứng với góc A

4ABC tiếp xúcBC tạiD GọiM trung điểm củaBC Chứng minhAD kIM (IM gọi đường thẳng Nagel)

Bổ đề 2.4ABC ngoại tiếp đường trịn.I / Đường trịn tiếp xúcBC tạiD GọiIa tâm đường trịn bàng tiếp góc A 4ABC Gọi M trung điểm BC Chứng minh

AD kIaM (IaM gọi đường thẳng Gergonne)

Áp dụng bổ đề vào toán, ta có I N k SL kIM k SK Lại cóS, K, Lthẳng hàng đóI,M,N thẳng hàng Ta thấyP,K đối xứng với quaM (M trung điểm củaAD) Q, L đối xứng với qua N (N trung điểm BC) suy PQ k MN k KL Gọi AD cắt BC T Do 4TMN4TPQ ( g.g ) Mặt khác 4TMN cân T nên

∠T NI D90ı ∠ATB

2 D∠AIB ( doI tâm đường trịn bàng tiếp gócT của4TAB ) Suy

ra4IAB4NIB ( g.g ) IB

IA D NB NI D

NC

NI Tương tự, ta có4I NC4DI C ( g.g )

suy I C

ID D NC

NI Từ đó, ta có I C ID D

IB

IA nên IA:I C D IB:ID Vậy toán chứng

minh

(85)

Tạp chí Epsilon, Số 08, 04/2016

j K

F E

X

D

Q

P

N M

B

A

I

C

Chứng minh. Gọi ME giao NF K Ta có AC,BD,MP,NQ đồng quy tạiX Áp dụng định lý Pascal cho lục giácMEQNFP suy raK,B,Xthẳng hàng đóME,NF,BDđồng quy Vậy toán chứng minh

Bài toán (Romania TST 2012) Cho tứ giácABCDngoại tiếp đường tròn cho∠ABCC

∠ADC < 180ı ∠ABDC∠ACB D ∠ACDC∠ADB Chứng minh hai đường chéo tứ giácABCDđi qua trung điểm đường chéo lại.

E

D A

B

C

Chứng minh. Đường tròn ngoại tiếp 4ABC cắt BD E Dễ dàng nhận thấy ∠DAE D

∠DCE Áp dụng định lý sin, ta có DA sin∠AED D

DE

sin∠DAE D

DE

sin∠DCE D

DC

sin∠DEC

Suy DA

DC D

sin∠AED

sin∠DEC D

sin∠ACB

sin∠BAC D AB

BC Lại có tứ giácABCDngoại tiếp đường trịn

do đóAB DBC CDA CD Nên DA

DC D

BC CDA CD

BC

Hay.BC CD/.DA CD/D0suy raBC DCDhoặcCD DDA

(86)

Bài toán (IMO Shortlist 2009) Cho tứ giácABCDngoại tiếp đường tròn Một đường thẳng d / bất kỳ qua A cắt BC tại M, cắt CD tại N Gọi I1/; I2/; I3/ là đường tròn nội tiếp 4ABM,4MNC,4NDA Chứng minh trực tâm4I1I2I3nằm trên.d /

F E

X H

I3

I2 I1

N A

D

C

B

M

Chứng minh. Gọi X giao điểm d / với tiếp tuyến kẻ từ C với đường tròn I1/ nên

ABCX tứ giác ngoại tiếp hay CX D BC CAX AB Lại có ABCD ngoại tiếp nên

AD DABCCD BC Suy raCXCAD DAXCCDhay tứ giácAXCDngoại tiếp đường tròn.I3/ Gọi E; F đối xứng vớiC qua I2I3,I2I1; H trực tâm 4I1I2I3 Dễ thấy

E,F thuộc.d / Lại có F I2HI1,I3HEI2 nội tiếp, đó∠FHI2 D ∠F I1I2 D ∠CI1I2 D

∠CI3I2 D∠EI3I2D∠EHI2hayH thuộc.d / Ta có điều phải chứng minh

Nhận xét.Trong q trình tìm hiểu, tơi nhận thấy toán số đề thi VN TST 2015 coi mở rộng từ toán

Bài tốn.4ABC nhọn có điểmP nằm tam giác cho∠APB D ∠AP C D ˛và

˛ > 180ı ∠BAC Đường tròn ngoại tiếp4APBcắtAC ởE, đường tròn ngoại tiếp4AP C

cắtAB ởF.Qlà điểm nằm trong4AEF cho∠AQE D∠AQF GọiD điểm đối xứng vớiQquaEF, phân giác∠EDF cắtAP tạiT

a) Chứng minh rằng∠DET D∠ABC ;∠DF T D∠ACB

b) Đường thẳngPAcắt đường thẳngDE,DF tạiM,N GọiI,J tâm đường tròn nội tiếp các4PEM,4PF N vàK tâm đường tròn ngoại tiếp4DIJ Đường thẳngDT cắt.K/tạiH Chứng minh rằngHK qua tâm đường tròn nội tiếp của4DMN

(87)

Tạp chí Epsilon, Số 08, 04/2016

F

E

N

M

I

L K

I2 I1

D

A B

C

P

Chứng minh. GọiM,N giao điểm hai tiếp tuyến chung ngồi của.I /,.I1/và.I /,.I2/

Ta có ADCDP AP

2 D

DP CDC P C

2

CP CBC BP

2 D

DC CCP DP

2 ,

nên ADCCP DAP CCD

BP CCD DDP CBC hay tứ giácAP CD,BPDC ngoại tiếp đường tròn

Áp dụng định lý Monge & d’Alembert cho ba đường tròn.I /,.I1/,.AP CD/và.I /,.I2/,

.BPDC /thìM; A; C vàN; B; D Tương tự, áp dụng định lý Monge & d’Alembert cho ba đường tròn.I /,.I1/,.I2/suy raKL,AB,MN đồng quy Ta tiếp tục áp dụng định lý Desargues cho4KAM và4LBN; suy raE,I,F thẳng hàng Vậy ta có điều phải chứng minh

(88)

E1

F1

D A

M

C B

N

G

H E

F

I

P

R

Chứng minh. Đường tròn I / tiếp xúc với AB CD G H Gọi P điểm đồng quy củaAB,EF,CD suy raBC,AD,GH đồng quy tạiR LấyEE1giaoAD tạiM Ta có

(

MF2 DME:ME1

ME:ME1DMA:MD (do M tâm đẳng phương I /; AED/; AID/ ) Suy

raMF2 D MA:MD Áp dụng định lý Ceva cho 4PAD với AC, BD,PF đồng quy, ta có

FA FD

CD CP

BP

BA D Ta lại áp dụng định lí Menelaus cho4PADvớiR,B,C thẳng hàng, ta

có RD

RA BA BP

CP

CD D1 Từ suy RD RA D

FD

FA vậy.AD; RF /D

Từ kết hợp hệ thức Newton, suy raM trung điểm củaRF Tương tự,FF1giaoBC

N, suy raN trung điểm củaRE MàRE2DRF2DRG:RH, nên4REF cân tạiR Suy ra∠NFE D∠NEF Lại có tứ giácEFE1F1 nội tiếp đường trịn.I /do tứ giácEFE1F1 hình thang cân Suy raEF kE1F1 Vậy toán chứng minh

(89)

Tạp chí Epsilon, Số 08, 04/2016 J K F E D I A B C M

Chứng minh. Gọi CI cắt BD K nên CK phân giác ∠BCD suy BK

BC D DK DC Gọi

phân giác góc∠MBC giao CI J1 suy ra∠IBJ1 D

2∠ABM D ∠ABD D ∠MBD

đó∠IBK D ∠CBJ1 D ∠MBJ1 Từ ta có IK

I C J1K J1C D

BK BC/

2

Gọi phân giác∠CDM

cắtCI tạiJ2.Tương tự, ta có IK

I C J2K J2C D

DK DC/

2

Từ điều trên, suy J1K

J1C D J2K J2C

hayJ1J2 J NênCJ,FJ,EJ MJ phân giác của∠F CE,∠CFM,∠CEM,

∠FME vậyJ tâm đường tròn nội tiếp tứ giácCEMF Vậy toán chứng minh Bài toán sau tham khảo [4] ứng dụng định lý từ đến công thức liên quan đến yếu tố tứ giác ngoại tiếp

Bài toán 12 Cho tứ giácABCDngoại tiếp đường trịn.I /khơng có cặp cạnh song song. Chứng minh rằngI là trọng tâm củaABCD khi khiIA:I C DIB:ID

H G F E D P N Q M A I B C

Chứng minh. Trước tiên, ta chứng minh chiều thuận tốn nghĩa nếuI trọng tâm tứ giácABCDthìIA:I C DIB:ID đặtAM DAQDa,BM DBN Db,CN DCP Dc,

DP DDQDd suy raQH D ja dj

2 ; NF D

jb cj

Gọi E,F,G,H trung điểm củaAB,BC,CD,DA Ta cóI trọng tâm tứ giác

(90)

a d D b c nên aCc D b Cd Theo định lý 8, ta cóIA:I C D IB:ID Trường hợp 2:

a d Dc bnên aCb D cCd Tương tựa b D c d suy raa Dc,b Dd Do tứ giácABCDlà hình bình hành (do tứ giácABCDkhơng có cặp cạnh song song) Ta chứng minh xong phần thuận toán

Tiếp theo, ta chứng minh phần đảo, tức làIA:I C DIB:IDthìI trọng tâm tứ giácABCD Ta cóIA:I C DIB:IDsuy raHQ DNF,IF DIH hayI thuộc đường trung trực củaFH Tương tựI thuộc đường trung trực củaEG MàEGvàFH không song song suy raI trọng tâm củaABCD Vậy toán chứng minh xong

3 Bài tập tự luyện

Bài toán 13 (Sharygin Geometry Olympiad 2014) ChoI là tâm đường tròn nội tiếp tứ giác ABCD Tiếp tuyến tạiAC của đường tròn ngoại tiếp4AI C cắt tạiX Hai tiếp tuyến tạiB D của đường tròn ngoại tiếp4BIDcắt tạiY Chứng minh rằngX,Y,I thẳng hàng.

Bài tốn 14 Cho tứ giácABCDngoại tiếp đường trịn.I / GọiH1,H2,H3,H4lần lượt là trực tâm4IAB,4IBC,4I CD,4IDA Chứng minh rằngH1,H2,H3,H4thẳng hàng.

Bài toán 15 Cho tứ giácABCDnội tiếp Một đường tròn quaC,DgiaoAC,AD, BC,BDtạiA1,A2,B1,B2 Một đường tròn khác quaA,B giaoCA,CB,DA,DB tạiC1, C2,D1,D2 Chứng minh tồn đường trịn tiếp xúc vớiA1A2,B1B2,C1C2,D1D2

Bài tốn 16 Cho hai đường tròn.O1/.O2/tiếp xúc với.O/tạiA,B TừAkẻ tiếp tuyếnAt1,At2tới.O2/, từB kẻ tiếp tuyếnBz1,Bz2 tới.O1/ GọiAt1cắtBz1tạiX,At2cắt Bz2tạiY Chứng minh rằngAXBY ngoại tiếp đường trịn.

Bài tốn 17 Cho tứ giácABCDngoại tiếp đường tròn Đường trung trực củaDA,AB,BC, CDlần lượt cắt trung trực củaAB,BC,CD,DAtạiX,Y,Z,T Chứng minh tứ giác X Y Z T ngoại tiếp đường tròn.

Bài tốn 18 Cho4ABC, lấyD,EthuộcBC Gọi.I1/,.I2/,.I3/,.I4/lần lượt đường trịn nội tiếp4ABD,4ACE,4ABE,4ADC Chứng minh tiếp tuyến chung khácBC của.I1/,.I2/,.I3/,.I4/cắt trênBC.

4 Lời kết

Tôi xin gửi lời cảm ơn chân thành tới thầyTrần Quang Hùng giáo viên trường THPT chuyên KHTN, Hà Nội, anh Ngơ Quang Dương học sinh lớp 12A2 Tốn trường THPT chuyên KHTN, Hà Nội, đọc kỹ thảo đưa lời góp ý quý báu, xác đáng để tài liệu hoàn chỉnh Mặc dù cố gắng tài liệu cịn có nhiều thiếu sót, tơi mong nhận góp ý phê bình bạn đọc để chun đề hồn thiện

Tài liệu tham khảo

(91)

Tạp chí Epsilon, Số 08, 04/2016

[2 ] http://www.artofproblemsolving.com/community/q1h569004p3338258 [3 ] Blog hình học sơ cấp http://analgeomatica.blogspot.com/

[4 ] http://www.artofproblemsolving.com/community/c6h35309p220212 [5 ] Mathley No 1, (January 2014)

[6 ] DarijGrinberg, Circumscribed quadrilaterals revisited, 2012 [7 ] The IMO Compendium

(92)(93)

MỘT SỐ ỨNG DỤNG CỦA CỰC VÀ ĐỐI CỰC

Trần Quang Hùng (Trường THPT Chuyên KHTN, Hà Nội) Nguyễn Tiến Dũng (Đại học Ngoại thương, Hà Nội)

Bài viết tóm tắt lại số kiến thức cực đối cực đường tròn đồng thời đưa số ví dụ nâng cao cho thấy ứng dụng khó thay khái niệm thực hành giải toán

1 Mở đầu

Khái niệm điểm liên hợp xuất phát từ việc ta chia điều hòa đoạn thẳng đường tròn nghĩa cho đường tròn cắt đoạn thẳng hai điểm hai điểm liên hợp điều hịa với hai đầu mút đoạn thẳng Khái niệm cực đối cực định nghĩa thông qua khái niệm điểm liên hợp, điều có nghĩa gắn chặt với khái niệm hàng điều hòa chùm điều hòa Tuy nhiên số toán việc dùng cực đối cực thực cần thiết mà việc sử dụng khái niệm điều hòa thông thường thay được, cố muốn thay lại dẫn đến rườm rà trình bày Do chúng tơi viết với mục đích tổng kết lại ý cách sử dụng công cụ cực đối cực Mặt khác chúng tơi muốn đưa số ví dụ xác thực cho thấy cần thiết phải dùng cực đối cực không muốn sử dụng khái niệm cách hình thức theo kiểu dùng mà khơng dùng

2 Tóm tắt lý thuyết

Trong mục tơi tóm tắt lại số ý từ việc định nghĩa cực đối cực số tính chất hay dùng để giải tốn Cực đối cực có nhiều cách định nghĩa với đường trịn, định nghĩa sau theo chúng tơi hay Cách định nghĩa tham khảo [3] Các khái niệm cực đối cực vấn đề liên quan khác bạn tham khảo [1,2] Ta khái niệm điểm liên hợp

Định nghĩa Cho đường tròn.O/hai điểmA; B gọi liên hợp với.O/nếu đường trịn đường kínhAB trực giao với.O/

Khái niệm cực đối cực định nghĩa thông qua khái niệm điểm liên hợp sau

Định nghĩa Cho đường tròn.O/và điểmP Tập hợp tất điểm liên hợp vớiP là một đường thẳng vng góc vớiOP Đường thẳng gọi đường đối cực củaP đối với.O/ Điểm P gọi cực đường thẳng đối với.O/

(94)

Định lý (La Hire) Đối với đường trịn thìAnằm đối cực củaB khi khi B nằm đối cực củaA

Hệ Quả 2.0.1 Đối với đường trịn đường đối cực đồng quy cực của chúng thẳng hàng.

Hệ Quả 2.0.2 Đối với đường trịn tỷ số kép chùm đối cực tỷ số kép của hàng cực tương ứng.

Định lý cách khác định nghĩa nhận biết khái niệm điểm liên hợp

Hệ Quả 2.0.3 Cho đường tròn.O/và đoạn thẳngAB cắt.O/tạiM; N thìA; Bliên hợp với O/khi hàng điểm.AB; MN /điều hòa.

Định lý sau nhận biết số điểm đặc biệt khác đường đối cực từ kết hợp định nghĩa ta có thêm nhiều tình nhận cực đường đối cực

Định lý Cho đường tròn.O/P.

i) Nghịch đảo củaP qua.O/nằm đường đối cực củaP đối với.O/

ii) Nếu kẻ tiếp tuyếnPA; PB tới.O/vớiA; B thuộc.O/thìA; B nằm đối cực củaP đối với.O/

Các ứng dụng cực đối cực tóm gọn qua hai định nghĩa, ba định lý hai hệ Mặc dù triển khai khái niệm cịn nhiều định lý khác muốn dành lại linh hoạt cho bạn đọc giải tốn Chúng tơi khơng muốn viết lý thuyết cách hình thức rườm rà quan điểm muốn ứng dụng khái niệm định lý hình học sơ cấp trước hết khái niệm định lý phải đơn giản dễ hiểu, định lý đơn giản mà khơng tầm thường ứng dụng giải tốn hình học cao

3 Một số định lý ứng dụng

Định lý sau tham khảo [4]

Định lý (Định lý Salmon) Cho đường tròn O/ P; Q là hai điểm GọiK; L lần lượt hình chiếu của P; Qlên đường đối cực củaQ; P đối với.O/ Chứng minh

OP OQ D

PK QL.

O

P

Q K

L T

S U

(95)

Tạp chí Epsilon, Số 08, 04/2016

Chứng minh. GọiS; T nghịch đảo củaP; Qđối với.O/thìS; T nằm đường đối cực

P; Qđối với.O/.OP; OQlần lượt cắtK T; LS tạiU; V Ta thấy tứ giácS T V U nội tiếp nênOS:OU DOV:OT mặt khácOS:OP DOT:OQ Từ OU

OP D OV

OQ nênU V kPQ Ta

dễ thấy hai tam giácKP U vàLQV đồng dạng g.g nên PK

QL D P U QV D

OU OV D

OP

OQ, ta hoàn

tất chứng minh

Định lý sau quen thuộc với bạn học sinh phổ thông Việt Nam qua toán Olympic, nhiên tên gọi định lý khơng nằm tài liệu thức mà tham khảo qua [5,6]

Định lý (Định lý Brokard) Cho tứ giácABCDcó thể khơng lồi nội tiếp đường tròn.O/ GọiAB cắtCDtạiE.ADcắtBC tạiGAC cắtBDtạiG Chứng minh rằngEF; F G; GE lần lượt đường đối cực củaG; E; F đối với đường tròn.O/

O B A

D C

F

E

G Q

P

Chứng minh. TrênBC; AD lấyP; Qsao cho hàng điểm.BC; PF /và.AD; QF /điều hòa P; Q liên hợp F O/ nên PQ đối cực F Mặt khác

.FP; BC /D 1D.FQ; AD/nênAB; CD vàPQ đồng quy tạiE Đồng thời.FP; BC /D 1D.FQ; DA/nênAC; BD vàPQđồng quy tạiG Từ đóPQ quaE; G hayEG đối cực củaF Tương tựF G đối cực củaE Vậy hiển nhiênGlà cực củaEF

Bản chất lời giải việc dựng điểm liên hợp củaF đối với.O/trênBC vàAD Tuy nhiên nhiều tài liệu chứng minh định lý thường dựng tiếp tuyến

.O/đi quaF Điều không cần thiết chưa thực xác với điều kiện tứ giác khơng cần lồi chưa chắcF nằm ngoài.O/để dựng tiếp tuyến

Định lý sau phát biểu tổng quát Conic tham khảo [7], viết chứng minh cho trường hợp đường tròn

(96)

A

B C

K D

E F V

U

W

Z

Y

X H

Chứng minh. GọiY Z; ZX; X Y cắtBC; CA; AB tạiU; V; W, theo định lý Desargues ta cần chứng minhU; V; W thẳng hàng, mặt khác áp dụng định lý Menelaus cho tam giác trung bình U; V; W thẳng hàng trung điểm củaAU; BV; C W thẳng hàng Ta lại dễ thấy trực tâmH tam giácABC có phương tích với đường trịn đường kính

AU; BV; C W nên muốn trung điểm củaAU; BV; C W thẳng hàng ta cần chứng minh đường trịn đường kínhAU; BV; C W đồng trục DoY; Zlần lượt cực củaCA; AB nênA

là cực củaY Z Từ hình chiếuD củaAlênY Z nghịch đảo củaAqua.K/ Tương tự ta cóE; F Dễ thấyD; E; F nằm đường trịn đường kínhAU; BV; C W mặt khác theo tính chất nghịch đảo thìKD:KADKE:KB DKF:KC nênK có phương tích với đường trịn này, điều có nghĩaKH trục đẳng phương đường trịn đường kínhAU; BV; C W, ta hoàn thành chứng minh

Điểm đồng quy gọi tâm thấu xạ tam giácABC đường tròn.K/ Hai tam giác

ABC vàX Y Z gọi hai tam giác liên hợp đường trịn.K/ Định lý tính chất quan trọng cực đối cực ứng dụng nhiều toán tam giác Ba toán sau coi hệ trực tiếp toán

Hệ Quả 3.0.4 Cho tam giácABC và đường tròn.K/bất kỳ Chứng minh đối cực của A; B; C với.K/cắtBC; CA; AB theo ba điểm thẳng hàng.

(97)

Tạp chí Epsilon, Số 08, 04/2016

Hệ Quả 3.0.6 (Mở rộng tính chất cát tuyến trực giao) Cho tam giácABC P bất kỳ. D; E; F lần lượt nằm trênPA; PB; P C sao choPD:PADPE:PB DPF :P C Chứng minh rằng đường thẳng quaD; E; F lần lượt vng góc vớiPA; PB; P C cắtBC; CA; ABtheo ba điểm thẳng hàng.

Qua định lý Brokard ta dễ thấy tứ giác nội tiếp tam giác tạo giao điểm hai đường chéo cạnh đối tự liên hợp với đường trịn ngoại tiếp tứ giác Hệ sau coi phát biểu ngược định lý Brokard

Hệ Quả 3.0.7 Nếu tam giác tự liên hợp đường trịn tâm đường trịn đó phải trực tâm tam giác.

Tên gọi nội dung định lý quan trọng sau tham khảo [13]

Định lý (Định lý Steinbart) Cho tam giác ABC có đường trịn nội tiếp I / tiếp xúc BC; CA; AB tạiD; E; F.X; Y; Z nằm trên.I /sao choDX; EY; F Zđồng quy Chứng minh rằngAX; BY; C Zđồng quy.

A B C I X D U M G H K E F

Chứng minh. Gọi tiếp tuyến tạiX; Y; Z của.I /lần lượt cắtBC; CA; AB tạiU; V; W Dễ thấy ba đường đồng quyDX; EY; F Zlần lượt đối cực củaU; V; W đối với.I /nênU; V; W thẳng hàng GọiAX; BY; C Zlần lượt cắtBC; CA; ABtạiM; N; P GọiUXcắtCA; ABtạiG; H tứ giácBC GH ngoại tiếp Theo tính chất quen thuộc thìCH; BGvàDXđồng quy tạiK nên ta có biến đổi tỷ số kép.BC; UM /DA.BC; UM /D.HG; UX /DK.HG; UX /D.CB; UD/ Từ ta suy UB

UC W MB M C D

UC UB W

DC DB hay

MB M C D

UB2 UC2

DC

DB DoAD; BE; CF đồng quy

vàU; V; W thẳng hàng nên nhân tỷ số tương tự ta dễ thu MB

M C NC

NA PA PB D

do đóAX; BY; C Zđồng quy

Chứng minh đưa sử dụng cực đối cực theo cho định lý kinh điển Chứng minh dựa ý tưởng xuất phát bạnNgơ Quang Dươnghọc sinh lớp 12 Tốn THPT chun KHTN bạn Dương giải toán tổng quát

Một hệ đơn giản rút từ phần cách chứng minh định lý sau

Hệ Quả 3.0.8 Cho tam giácABC có đường trịn nội tiếp.I /.P thuộc.I / Tiếp tuyến tạiP của.I /cắtBC tạiU.AP cắtBC tạiM thì MB

M C D UB2 UC2

(98)

Việc phát biểu lại hệ cho chúng thấy tầm áp dụng rộng hệ

Hệ Quả 3.0.9 Cho đường tròn.K/tiếp xúc đoạnBC tạiD TừB; C vẽ tiếp tuyến khác BC cắt tạiX.P thuộc.I / Tiếp tuyến tạiP của.I /cắtBC tạiU.XP cắtBC tạiM thì MB

M C D UB2 UC2

DC DB

Mặc dù tưởng chừng cách phát biểu thứ hai khơng có ý nghĩa hệ thứ thực chất có nhiều giá trị tốn thực hành khác ý phát biểu vậy.K/có thể đường trịn bàng tiếp tam giácXBC khơng bắt buộc đường tròn nội tiếp

Định lý sau định lý kinh điển hình học xạ ảnh chứng minh đơn giản cực đối cực định lý La Hire

Định lý (Định lý Brianchon) Chứng minh đường chéo lục giác ngoại tiếp đồng quy.

O F

B

D A

C E

S

M

Q

P R

N

X

Z Y

Chứng minh. Giả sử lục giácABCDEF ngoại tiếp đường tròn O/ Các tiếp điểm của.O/

vớiAB; BC; CD; DE; DF; FAlàM; N; P; Q; R; S Thấy giao điểm cặp đường thẳngMN vàRQ,NP vàSR,PQ vàSM cực đường thẳngBE; CF

AD đường tròn.O/ Theo định lý Pascal điểm thẳng hàng nênAD; BE; CF

đồng quy

Một cách ngược lại công nhận định lý Brianchon ta hồn tồn chứng minh định lý Pascal cực đối cực cách dựng tiếp tuyến đường tròn cắt tạo lục giác ngoại tiếp

(99)

Tạp chí Epsilon, Số 08, 04/2016

O S

N

Q M

P R

F

A

D

C E

B

X

Z Y

Chứng minh. Xét lục giácABCDEF nội tiếp đường tròn.O/ta cần chứng minh giao điểm

X; Y; Z theo thứ tự cặp đường thẳngAB vàDE,BC vàFE,CDvàFAthẳng hàng Gọi tiếp tuyến tạiA; B; C; D; E; F của.O/cắt tạo thành lục giác MNPQRS

ngoại tiếp hình vẽ Dễ thấyX; Y; Z cực đường thẳngMQ; PS vàNR Theo định lý Brianchon thìMQ; PS vàNRđồng quy nênX; Y; Z thẳng hàng

Lục giác hai định lý suy biến khơng thiết phải lồi hình minh họa nên hiểu linh hoạt khái niệm "đường chéo chính" "cạnh đối" lục giác phát biểu định lý

Mặc dù đặt hai chứng minh cạnh chúng tưởng chừng khơng có ý nghĩa dùng định lý để chứng minh định lý kia, khơng phải Trong hình học sơ cấp biết nhiều cách khác chứng minh định lý Pascal Brianchon hồn tồn dùng định lý để chứng minh định lý thông qua phương pháp sử dụng công cụ cực đối cực Tuy nhiên mục đích chúng tơi trình bày hai chứng minh cạnh để làm bật đối ngẫu hai định lý kinh điển Pascal Brianchon thông qua hai khái niệm đối ngẫu quan trọng khác hình học xạ ảnh cực đối cực

(100)

4 Một số ví dụ

Các tốn ví dụ chúng tơi đề nghị mục mang hướng chủ đạo tốn thi Olympic khơng phải tốn thuộc đối tượng nghiên cứu chuyên sâu Trong số tốn chúng tơi sử dụng định lý phần trước chúng tơi khơng có tham vọng dùng hết tất định lý, định lý cịn có ứng dụng chun sâu mà viết dài chưa thể viết hết

Bài toán tham khảo [19]

Bài tốn Cho tam giácABC với đường trịn nội tiếp.I /.P là điểm bất kỳ.PA; PB; P C cắt BC; CA; AB tạiK; L; N GọiX; Y; Z lần lượt cực đường thẳngLN; NK; KLđối với.I / Chứng minh rằngAX; BY; C Zlần lượt cắtBC; CA; AB tại ba điểm thẳng hàng trên một đường thẳng đường thẳng tiếp xúc với.I /

A

B C

P

L N

K I

X

Y

Z

U

V W

F

E

D Q

S

T

Chứng minh. Gọi AX; BY; C Z cắtBC; CA; AB U; V; W Trước hết ta chứng minhYB; C Z đồng quy vớiEF, từ ta dễ thấyV W tiếp xúc I / Tương tự U W; U V

tiếp xúc I / nên U; V; W thẳng hàng đường thẳng tiếp xúc I /, GọiDE; DF

lần lượt cắtKL; KN tạiS; T doAD; BE; CF đồng quy vàAK; BL; CN đồng quy nên

D.KA; EF / D D K.DA; LN / đóA; S; T thẳng hàng VìB; Y cực

DF; NK nênT cực củaBY Tương tự S cực củaC Z Mặt khác Alà cực EF Vì

(101)

Tạp chí Epsilon, Số 08, 04/2016

Bài toán Cho tam giácABC nội tiếp đường tròn.O/ Đường tròn.Ka/tiếp xúcCA; AB tiếp xúc trong.O/ GọiD là cực củaBC đối với.Ka/ Tương tự cóE; F Chứng minh rằng AD; BE; CF đồng quy.

A

B C

O

Ka

D N

M

X

I

U Q

Chứng minh. Gọi.Ka/tiếp xúcCA; AB tạiM; N thìMN đối cực củaAđối với.Ka/ Vậy

MN cắt BC X thìX cực củaAD Ka/ Từ đóAD cắt MN; BC tạiQ; U

.MN; QX /D 1nên.BC; UX /DA.MN; QX /D Từ tương tự có giao điểmY; Z

vàU; V GọiI tâm nội tiếp thìIX; I Y; I Z theo thứ tự vng góc vớiIA; IB; I C theo tốn cát tuyến trực giao thìX; Y; Z thẳng hàng Từ cóAU; BV; C W đồng quy hay nói cách khác làAD; BE; CF đồng quy

Một sai lầm phổ biến giải sau raX cực củaAD tương tự vớiY; Zthì nhận xétX; Y; Z thẳng hàng nênAD; BE; CF đồng quy Nguyên nhân sai lầm doX đối cực củaADvới đường tròn.Ka/cònY; Ztương ứng cực củaBE; CF với đường tròn.Kb/; Kc/ Hệ định lý La Hire xét cực đối cực với đường trịn khơng phải với ba đường tròn khác

(102)

a

b

A

B C

O Kc

I

X

P

Q

N

M D

K

K

Chứng minh. Gọi Kb/; Kc/ tiếp xúc O/ P; Q Theo tính chất quen thuộc P I qua trung điểm N cung AC chứa B QI qua trung điểm M cung AB chứa C Đường thẳng quaI vng gócAI cắtPQ tạiX Dễ thấyAI ? MN nênXI kMN,

∠XIQ D∠NMI D∠IPQ Do đóXI2 D XP:XQ Ta dễ thấy tâm ngoại tiếp tam giác

BI C giao củaAI với.O/nênXI tiếp tuyến của.BI C /nênXI2 phương tích củaX

đối với.BI C /mặt khácXP:XQlà phương tích củaX đối với.O/ Do đóXnằm trục đẳng phương của.O/và.BI C /chính làBC Từ việcBC tiếp tuyến chung của.Kb/; Kc/còn

P; Qcũng tâm vị tự của.Kb/; Kc/với.O/nên theo định lý D’lambert giao điểm

X củaPQvàBC phải tâm vị tự của.Kb/; Kc/nênKbKc quaX Theo giả thiếtD

là cực củaKbKc với.Ka/màKbKc quaX nênD vàX liên hợp với.Ka/ Lại dễ thấyAvà

X liên hợp với.Ka/nênADlà đường đối cực củaX đối với.Ka/ Tương tự vớiBE; CF Theo chứng minh trước đường đối cực đồng quy

Chúng ta tiếp tới toán sau đề thi chọn đội tuyển KHTN vòng năm 2011-2012 ngày thứ tác giả đề xuất

Bài tốn Cho tam giác khơng cânABC Đường tròn nội tiếp.I /của tam giácABC tiếp xúc vớiBC; CA; ABlần lượt tạiD; E; F.ADgiaoEF tạiJ.M; N di chuyển đường tròn I /sao choM; J; N thẳng hàng vàM nằm phía nửa mặt phẳng chứaC bờAD,N nằm về phía nửa mặt phẳng chứaB bờAD Giả sửDM; DN lần lượt cắtAC; AB tạiP; Q

a) Giả sửMN giaoPQtạiT Chứng minh rằngT luôn thuộc đường thẳngd cố định.

b) Giả sử tiếp tuyến tạiM; N của.I /cắt tạiS Chứng minh rằngS thuộcd.

(103)

Tạp chí Epsilon, Số 08, 04/2016

A

B C

I F

E

D G

J N

M Q

P T

S

K

Lời giải. a) GọiEF giaoBC tạiG.MN giaoAGtạiT Ta chứng minhP; Q; T thẳng hàng từ suy raT giao củaMN vàPQthuộcAGcố định, thật vậy, xét cực đối cực với đường tròn.I / Dễ thấyGlà cực củaADnênG; J liên hợp,J; Acũng liên hợp nênJ cực củaAG

MN quaJ cắtAG tạiT suy MN; J T /D 1suy D.MN; J T /D 1D A.CB; DG/

từ suy raT; P; Qthẳng hàng

b)S cựcMN suy raS; J liên hợp nênS thuộcd đối cực củaJ

c)S cựcMN,T thuộcMN nênS; T liên hợp,J; T liên hợp nênT cựcSJ,K thuộcSJ

nênT; Kliên hợp, ta cóD; Kliên hợp nênK cực củaTDdo đóIKvng gócTD Bài tốn sau tác giả mở rộng toán [8], lời giải dựa theo ý tưởng Telv Cohl [8]

Bài toán Cho tam giácABC nội tiếp đường tròn.O/ Các điểmE; F lần lượt nằm cạnh CA; AB Đường đối trung quaAcắt đường tròn ngoại tiếp tam giácAEF tạiP.OP cắtEF tạiD và cắt đường tròn ngoại tiếp tam giácAEF tạiQ khácP.DAcắt O/tạiT khácA Tiếp tuyến tạiT của.O/cắtBC tạiS LấyRthuộcADsao choOR?AQ Chứng minh RS ?OR

A

B

C O

F

E

P D

T

Q

W S

R

V U

(104)

Chứng minh. GọiAQcắtOR; EF tạiU; V cắt.O/tạiW khácA Dễ thấy.AP; EF /D

nên chùmQ.AP; EF / D 1chiếu chùm lên đường thẳngEF thì.VD; EF / D 1nên chùmA.VD; EF /D 1, chiếu chùm lên.O/thì hàng.W T; BC / D Từ nếuT W

cắtBC tạiK thì.SK; BC /D 1nênT; Kđều liên hợp vớiSđối với.O/ Nói cách khácTK

là đối cực củaS O/nên S W tiếp xúc.O/, từ T; W thuộc đường trịn đường kínhOS Ta lại có ∠T WO D 90ı ∠TAW D ∠TRO nên tứ giácRTOW nội tiếp, đường trịn ngoại tiếp đường trịn đường kínhOS Từ đóRS ?OR

Bài tốn sau tham khảo [9] mở rộng đề IMO năm 1998

Bài toán Cho đường tròn.I /nội tiếp tam giácABC GọiK; LM là điểm tiếp xúc đường tròn nội tiếp tam giác ABC tương ứng với cạnhAB; BC CA Đường thẳngMKMLcắt đường thẳng quaBvà song song vớiKLtương ứng điểmQR Đường trịn với đường kínhQRcắt.I /tạiS; T Chứng minh rằngS T chia đôi đoạn thẳngKL

B

A C

I K

M

T Q

R D

S

L J

Lời giải. Gọi D/ đường trịn với đường kính QR Ta dễ dàng thấy 4BLR 4BQK

nên BQ:BR D BK:BL D BK2 Giờ ta có hai lần phương tích củaI với đường tròn.D/

IR2CIQ2 RQ2 D.BI2CBR2/C.BI2CBQ2/ BRCBQ/2 D2.BI2 BR:BQ/D 2.BI2 BK2/D2IK2 Điều có nghĩa đường tròn.D/và.I /là trực giao, nênDS; DT

là tiếp tuyến của.I /suy raS T đối cực củaDđối với.I / Mặt khác nếuJ trung điểmEF

thìJ cực đường thẳng quaB vng góc vớiIB đối với.I /nênJ; D liên hợp

.I / Từ đóS; T; J thẳng hàng

Bài tốn sau mở rộng đề thi vô địch Nga năm 2015 [10]

(105)

Tạp chí Epsilon, Số 08, 04/2016

A

B C

I P Q

N

M F

E

D

T

S

Chứng minh. GọiAP; AQcắtEF tạiS; T DoA.BC; PQ/D 1nên hàng.EF; S T /D VậyS; T liên hợp với.I /, dễ thấyS; Aliên hợp nênS cực củaAT, suy raS; Qliên hợp

S nằm trênDN Tương tựT nằm trênDM Từ chiếu chùmD.EF; S T /D 1lên đường trịn.I /thì hàng.EF; MN /D 1nên tứ giácEMF N điều hịa Từ đóMN quaA

(106)

Z' Z

Y

Y'

E N'

P'

X X'

D M'

P

N F

I A

B M C

Chứng minh. LấyM0; N0; P0trênBC; CA; ABsao cho.BC; MM0/D.CA; N N0/D.AB; PP0/D Chú ý rằngAM; BN; CP đồng quy ta có M

0B M0C

N0C N0A

P0A P0B D

MB M C

NC NA

PA PB D

nên M0; N0; P0 thẳng hàng D; E; F tiếp điểm I / với BC; CA; AB Kẻ tiếp tuyến M0X0; N0Y0; C0P0 I / theo toán ta thấy AX0; BY0; C Z0

AX ; BY; C Z.DX0; EY0; F Z0có cực đối với.I /tương ứng làM0; N0; P0thẳng hàng nên chúng đồng quy Áp dụng định lý Steinbart ta cóAX0; BY0; C Z0đồng quy hayAX ; BY; C Z

đồng quy

Bài toán sau bổ đề thú vị cực đối cực, tham khảo [11]

(107)

Tạp chí Epsilon, Số 08, 04/2016

A

B C

I K

Q

S L

R

Lời giải thứ nhất. Gọi hình chiếu AlênIB; I C làQ; R dễ thấy QRlà đường trung bình ứng vớiAcủa tam giácABC Gọi.I /tiếp xúcCAtạiS.BI cắtCKtạiL Dễ thấy tứ giácI SLC vàIQSAnội tiếp Từ ∠I SQ D ∠IAQD ∠AIB 90ı D ∠I CA D∠ILS VậyIQ:IL DI S2, mặt khácCK vng góc vớiIQtạiLnênCK đối cực củaQđối với

.I / VậyK vàQliên hợp đối với.I / Tương tựK vàR liên hợp đối với.I / VậyQRchính đối cực củaK đối với.I /

K

N

M F

E

D I A

B C

Lời giải thứ hai. GọiD; E; F tiếp điểm của.I /vớiBC; CA; AB.M; N giao củaBI; CI

vớiDE; DF dễ thấyMN đường trung bình ứng với đỉnhAcủa tam giácABC.DF đối cực củaBđối với.I /nênB; N liên hợp với.I / Do đóBK ?I N đối cực củaN

.I /nênK; N liên hợp với.I / Tương tựK; M liên hợp với.I / VậyMN đối cực củaK đối với.I /

Bài toán sau tác giả đề xuất [12]

(108)

A

B C

I

E F

N

M

K

P

S

T

Q R

Chứng minh. Gọi.I /tiếp xúcCA; AB tạiS; T GọiAM; AN vng góc vớiI C; IB

tạiR; Q Dễ thấy điểmS; Q; T; Rđều nằm đường trịn đường kínhAI Áp dụng định lý Pascal cho

R S I T Q A

suy raME; QRvàS T đồng quy Tương tự thìNF; QRvàS T

đồng quy.Từ theo đề bốn đường thẳngME; NF; QR; S T đồng quy tạiP Chú ýAlà cực củaEF theo thìK cực củaQRđối với.I / Ta suy raP cực củaAK đối với.I /vậyIP ?AK

(109)

Tạp chí Epsilon, Số 08, 04/2016

Ib

Ic

M

Ia

Y Z

X

K

L

F E

D H

I

C B

A

Chứng minh. Các đối cực củaD; E; F đối với.I /cắt tạo thành tam giácH; K; L.E; F

cùng liên hợpH đối với.I /nênEF đối cực củaH đối với.I / Theo tốn thìH trực tâm tam giácIBC Tương tựK; Llà trực tâm tam giácI CA; IAB GọiIa; Ib; Ic tâm bàng tiếp gócA; B; C tam giácABC Dễ thấyH; K; Lđối xứng vớiIa; Ib; IcquaD; E; F Chú ý hai tam giácIaBC; IaIbIc đồng dạng nên IaD đường đối trung tam giác

IaIbIc Từ dễ thấyDH; EK; FLđồng quy tạiM điểm Lemoine tam giácIaIbIc điểm Mittenpunkt tam giácABC.X thuộc đối cực củaD; H đối với.I /nênD; H liên hợpXđối với.I / Do đóDH đối cực củaX đối với.I /nênX liên hợpM đối với.I / Tương tựY; Z liên hợpM đối với.I / Vậy X; Y; Z thuộc đối cực củaM

.I /

(110)

Bài toán 12 Cho tam giácABC D; E; F là trung điểmBC; CA; AB LấyL; M; N trên EF ; FD; DE sao cho DL; EM; F N là tiếp tuyến đường tròn nội tiếp tam giác ABC. Chứng minh rằngL; M; N thẳng hàng.

Ta cần bổ đề sau

Bổ đề Cho tam giácABC và đường tròn.O/cắt cạnhBC; CA; AB tại cặp điểmA1; A2IB1; B2IC1; C2 Khi đóAA1; BB1; C C1đồng quy khiAA2; BB2; C C2 đồng quy.

Bổ đề quen thuộc với nhiều bạn đọc, xin khơng chứng minh Trở lại tốn

Y Z

M

W

N

V L

U X

R

Q

F E

D P

I

C B

A

Chứng minh. Gọi đường tròn.I /nội tiếp tam giácABC tiếp xúc vớiBC; CA; AB tạiX; Y; Z

DL; EM; F N tiếp xúc.I /tạiU; V; W.X U; Y V; ZW cắt tạo thành tam giácPQR Từ toán8dễ thấyP; Q; Rlà cực củaEF ; FD; DE đối với.I /và trực tâm tam giác

IBC; I CA; IAB Chú ý rằngPX; QY; RZ đồng quy tạiI, áp dụng bổ đề cho tam giác

PQRvà đường trịn.I /ta cóP U; QV; RW đồng quy Xét cực đối cực với.I /.Lthuộc đối cựcEF; DU củaP; U nênLliên hợp vớiP; U Do đóP U đối cực củaL Lập luận tương tự

(111)

Tạp chí Epsilon, Số 08, 04/2016

Tương tự với kỹ thuật sử dụng toán 11, giải tốn tổng quát Nguyễn Phạm Đạt đề xuất xem [14]

Bài tốn 13 Cho tam giácABC có đường trịn nội tiếp.I /.A1; B1; C1bất kỳ trênBC; CA; AB A2là giao điểm củaB1C1với tiếp tuyến quaA1của.I /khácBC.B2; C2 được xác định tương tự Chứng minh rằngA2; B2; C2thẳng hàng khiAA1; BB1; C C1đồng quy.

Lời giải sau dựa theoLuis González

B'

C'

A' B2

C0

A0

B0

E' C2

F' A2

D' C1

F

E

D I

C B

A

A1

B1

Chứng minh. Gọi.I / tiếp xúc vớiBC; CA; AB tạiD; E; F.A1A2; B1B2; C1C2 tiếp xúc với

.I /tạiD0; E0; F0.DD0; EE0; FF0cắt tạo thành tam giácA0B0C0.A0; B0; C0là giao điểm củaBC; B1C1ICA; C1A1IAB; A1B1 Xét cực đối cực với.I /.A0là giao đối cựcEE0; FF0

củaB1; C1nênB1; C1 liên hợpA0; đóA0là cực củaB1C1 Lập luận tương tựB0; C0

là cực củaC1A1; A1B1.A2 giao đối cựcB1C1; A1D0củaA0; D0nênA2là cực củaA0D0 Lập luận tương tự ta cóB2; C2là cực củaB0E0; C0F0;A0; B0; C0là cực củaA0D; B0E; C0F Ta có khẳng định sau tương đương

1)A2; B2; C2thẳng hàng

(112)

3)A0D; B0E; C0F đồng quy (Áp dụng bổ đề 11.1 với tam giácA0B0C0 và.I /) 4)A0; B0; C0thẳng hàng (Hệ 1.1)

5)AA1; BB1; C C1đồng quy (Áp dụng định lý Desargues cho hai tam giácABC vàA1B1C1)

Bài toán sau tham khảo [17]

Bài toán 14 Cho tam giácABC với tâm nội tiếpI, trung tuyếnAD; BE; CF đồng quy tại G TrênEF; FD; DE lấyX; Y; Z sao choIX ?IA; I Y ?IB; I Z ?I C Chứng minh rằng X; Y; Z thẳng hàng đường thẳng vng góc vớiI G

A

B C

I

F E X

H

K D

M J

G

Chứng minh. Gọi.I /là đường tròn nội tiếp tam giácABC.H trực tâm tam giácIBC Theo toán 8H cực củaEF vậyH vàX liên hợp với.I /vậy đường đối cực củaX đường thẳng quaH vng góc vớiIX GọiM tâm nội tiếp tam giácDEF vàK tâm bàng tiếp gócAcủa tam giácABC Dễ thấyIK đường kính đường tròn ngoại tiếp tam giácIBC

nênK vàH đối xứng quaD GọiJ đối xứngI quaM Dễ thấyDM k IA Từ theo tính chất đường trung bình hình thang thìHJ kIA ?IX Nói cách khác đường đối cực

X ln quaJ Tương tự đường đối cực củaY; Z quaJ VậyX; Y; Z thẳng hàng vàJ cực đường thẳng quaX; Y; Z Dễ thấyI; G; M; J thẳng hàng nên đường thẳng quaX; Y; Z vng góc vớiI G

(113)

Tạp chí Epsilon, Số 08, 04/2016

Bài tốn 15 Cho tam giácABC có đường tròn nội tiếp.I /và tâm ngoại tiếpO Đường tròn Oa/quaB; C tiếp xúc với.I / Các đường tròn.Ob/; Oc/được định nghĩa tương tự Chứng minh tâm đẳng phương của.Oa/; Ob/; Oc/thuộcOI.

Lời giải sau dựa theo Dựa theo Ilya I Bogdanov, Fedor A Ivlev Pavel A Kozhevnikov [15]

l H

M K

U

Z Y

O

X F

E

D I

A

B C

Chứng minh. Gọi.Oa/; Ob/; Oc/tiếp xúc với.I /tại X; Y; Z Tiếp tuyến tạiX I /cắt

BC tạiM Ta cóM X2DMB M C nênM thuộc trục đẳng phươngl của.O/; I / GọiK cực củal đối với.I /thìOI kIK ?l nênK thuộcOI Xét cực đối cực với.I /.M thuộc đối cựcl K nênK thuộc đối cựcDX củaM Lập luận tương tựEY; F Z quaK Các tiếp tuyến tạiY; Zcủa.I /cắt tạiU thìAU trục đẳng phương của.Ob/; Oc/.EF; Y Zcắt tạiH.H thuộc đối cựcEF; Y Z củaA; U nênA; U liên hợp vớiH, thếAU đối cực củaH Dễ thấyH thuộc đối cựclcủaK nênK thuộc đối cựcAU củaH Do đóK phương tích với.Ob/; Oc/ Lập luận tương tự ta cóK có phương tích với.Oa/; Ob/; Oc/ Vậy tâm đẳng phươngK của.Oa/; Ob/; Oc/thuộcOI

Cũng kỳ thi Romanian Masters In Mathematics năm 2013, có tốn sau

(114)

T S

K

M R

P B

A

O

Q

D C

Chứng minh. Gọi O tâm đường trịn ! Đường thẳng qua O vng góc với MR cắt

PQ; MR tạiS; T Theo định lý Brokard dễ thấyR trực tâm tam giácOPQ Chú ý M trung điểm PQ T hình chiếu O MR, theo kết quen thuộc

T thuộc đường trịn OPQ/ Dễ có PQ đối cực R ! nên S liên hợp R ! Do MR?OS đối cực S ! nên SK tiếp tuyến O/ Ta có

SK2DS T SO DSP SQnênSK tiếp tuyến của.KPQ/ Vậy đường tròn ngoại tiếp tam giácKPQtiếp xúc!tạiK

Bài toán 17 Cho tam giácABC nội tiếp.O/có đường trịn nội tiếp.I /tiếp xúc vớiBC; CA; AB tạiA0; B0; C0.A1là điểm cungBAC của.O/ LấyA2trênIA1sao choA0A2 kIA LấyA3trênB0C0sao choA2A3 ?IA1 .B3; C3được định nghĩa tương tự Chứng minh rằng A3; B3; C3cùng nằm đường thẳng vng góc vớiOI.

Chứng minh. IA1 qua tiếp điểmX đường trònA- mixtilinear tam giácABC với

.O/ Đường thẳng quaI vng góc vớiIAcắtBC tạiA4 Theo tính chất quen thuộcXA4là phân giác ngồi của∠BXC nênXA4 ?XI Xét cực đối cực với.I / Dễ thấyA0A2 ?IA4là đối cực củaA4nên A2liên hợp vớiA4 Do đóXA4 ? IA2 đối cực củaA2nên X liên hợp vớiA2 Vì thếA2A3 ? IX đối cực củaX nên X liên hợp vớiA3.A3 thuộc đối cựcB0C0

của Anên Aliên hợp vớiA3 Từ đóAX đối cực A3 đối với.I /.AX qua tâm vị tự ngoàiS của.O/; I /nênA3 liên hợp vớiS Lập luận tương tựB3; C3đều liên hợp vớiS Vậy

(115)

Tạp chí Epsilon, Số 08, 04/2016

A4 S

X

A3 A1

B0 C0

A2 O

A0 I

A

B

C

Kết thúc ví dụ, chúng tơi có đơi lời muốn chia sẻ Cực đối cực khái niệm quan trọng hình học xạ ảnh cơng cụ quan trọng việc giải tốn hình học sơ cấp Trong viết nhỏ diễn đạt hết tốn sử dụng cơng cụ Định lý phần định lý La Hire sử dụng nhiều bên cạnh định lý lớn phần định lý Salmon, định lý Brokard, định lý Conway giới thiệu qua với cách chứng minh viết chưa đề cập tới ứng dụng phong phú định lý Mặt khác ứng dụng lớn cực đối cực gắn liền với tứ giác ngoại tiếp, lục giác chưa đề cập tới viết Do viết nhìn lướt qua số toán ứng dụng hay cực đối cực chưa sâu khai thác cụ thể Mỗi toán chun đề cịn có nhiều khai thác mà viết thành chuyên đề riêng Về mặt chất cực đối cực xuất phát từ khái niệm điểm liên hợp hàng điều hịa phát triển lên giữ tính đơn giản nên có ứng dụng lớn Nếu toán lớn để tránh dùng cực đối cực, người giải hồn tồn làm thêm cơng đoạn chứng minh cách trá hình tính chất cực đối cực thân tính chất đơn giản Tuy nhiên rõ ràng điều khơng có lợi mặt sư phạm gây khó hiểu, chúng tỗi ln khuyến khích bạn, em học sinh thi Olympic nên dùng nghĩ theo hướng thấy tiện lợi

5 Một số toán luyện tập

Bài tốn 18 Cho tam giácABC có đường tròn nội tiếp.I / GọiH; K; Llần lượt trực tâm tam giácIBC; I CA; IAB Chứng minh rằngAH; BK; CLlần lượt cắtBC; CA; AB theo ba điểm thẳng hàng đường thẳng tiếp xúc.I /

(116)

rằngPAlà phân giác∠EPF.

Bài toán 20 Cho tam giácABC nhọn nội tiếp đường tròn.O/.P nằm tam giác cho AP ? BC.D; E; F là hình chiếu củaP lên BC; CA; AB Đường tròn ngoại tiếp tam giác DEF cắtBC tạiM khácD.MP cắtEF tạiN TiaND cắt.O/tạiQ Chứng minh rằngQA là phân giác∠EQF.

Bài toán 21 ChoA; B; M là cực đường thẳnga; b; mvới đường tròn.O/ Chứng minh rằng

d.M; a/ d.M; b/ W

d.A; m/ d.B; m/ D

d.O; a/ d.O; b/ D

d.B; a/ d.A; b/ Trong đód.X; d /ký hiệu khoảng cách từ điểmX tới đường thẳngd.

Bài toán 22 Cho tam giácABC nội tiếp đường tròn.O/P nằm phân giác ∠BAC. E; F là hình chiếu củaP lên CA; AB.OP cắt EF tại Q.S thuộcBC sao choSQ ? OQ GọiAQ cắt.O/tạiT khácA Chứng minh rằngS T tiếp xúc.O/

Bài toán 23 Cho tam giácABC có đường trịn nội tiếp.I /.P là điểm bất kỳ.PA; PB; P C lần lượt cắtBC; CA; AB tại D; E; F TrênEF; FD; DE lấy X; Y; Z sao choDX; EY; F Z tiếp xúc.I / Chứng minh rằngX; Y; Z thẳng hàng.

Bài toán 24 (Telv Cohl) Cho tam giácABC vớiP; Qlà hai điểm đẳng giác.H; K là trực tâm của tam giácPBC; QBC Chứng minh trung điểmHK là cực đường trung bình ứng vớiAcủa tam giácABC đối với đường tròn Pedal củaP; Q

Bài tốn 25 Cho tam giácABC đường trịn nội tiếp.I / GọiHa; Hb; Hc là trực tâm tam giácIBC; I CA; IAB ĐoạnIAcắt.I /tạiA1, tiếp tuyến tạiI củaA1cắtHbHc tạiA2 Tương tự cóB2; C2 Chứng minh rằngA2; B2; C2thẳng hàng.

Bài toán 26 Cho tam giácABC nội tiếp.O/ Đường cao tạiA; B; C cắt đường tròn điểm thứ haiA0; B0; C0 Tiếp tuyến tạiB0; C0cắt tạiA1.A2là đối xứng củaA0quaO.A1A2 cắt.O/tạiA3.A0A3cắt tiếp tuyến tạiAcủa.O/tạiA4 Tương tự cóB4; C4 Chứng minh A4; B4; C4thẳng hàng.

Bài toán 27 ChoI; K là tâm nội tiếp điểm Nagel tam giácABC.D; E; F là tiếp điểm của I /nội tiếp với BC; CA; AB.M; N; P lần lượt trung điểm BC; CA; AB Các đường thẳng qua D; E; F và vng góc vớiAK; BK; CK cắt cạnh NP; PM; MN tại X; Y; Z. Chứng minh rằngX; Y; Z thẳng hàng đường thẳng vng góc vớiILtrong đóLlà điểm Lemoine tam giác tạo ba tâm bàng tiếp.

Bài toán 28 Cho tứ giác ABCD ngoại tiếp đường tròn I / Các tiếp điểm tương ứng với AB; BC; CD; DAM; N; P; Q.MP giaoNQtạiK.ABgiaoCDtạiE,BC giaoADtại F Chứng minh rằngIKvng góc vớiEF.

(117)

Tạp chí Epsilon, Số 08, 04/2016

Bài toán 30 (Bài T12/445 - số tháng năm 2014) Cho tứ giácABCDnội tiếp đường tròn I / Các cạnhAB; BC tiếp xúc với.I /lần lượt tạiM; N GọiE là giao điểm củaAC MN, F là giao điểm củaBC DE.DM cắt.I /tạiT khácM Chứng minh rằngF T là tiếp tuyến của.I /

Bài toán 31 (VN TST 2003) Cho tam giácABC nội tiếp đường tròn.O/.A0; B0; C0là trung điểm đường caoAH; BK; CLcủa tam giácABC Đường tròn.I /nội tiếp tam giácABC tiếp xúc vớiBC; CA; ABtạiD; E; F Chứng minh rằngA0D; B0E; C0F OI đồng quy

Tài liệu tham khảo

[1] http://www.cut-the-knot.org/Curriculum/Geometry/PolePolar.shtml [2] http://mathworld.wolfram.com/Polar.html

[3] Nathan Altshiller-Court, College Geometry: An Introduction to the Modern Geometry of the Triangle and the Circle, Dover Publications; Rev Enl edition (April 19, 2007) [4] http://mathworld.wolfram.com/SalmonsTheorem.html

[5] http://www.mit.edu/ alexrem/ProjectiveGeometry.pdf [6] http://www.imomath.com/index.php?options=334&lmm=0 [7] http://www.maths.gla.ac.uk/wws/cabripages/triangle/conics.htm [8] http://artofproblemsolving.com/community/c6t48f6h1213584

[9] http://analgeomatica.blogspot.com/2015/07/nhat-ky-mot-chuyen-i.html [10] http://artofproblemsolving.com/community/c6t48f6h1211140

[11] http://www.xtec.cat/ qcastell/ttw/ttweng/resultats/r276.html

[12] http://analgeomatica.blogspot.com/2016/02/moi-tuan-mot-bai-toan-tuan-2-thang-2.html [13] http://web.mit.edu/ darij/www/geometry2.html

[14] http://www.artofproblemsolving.com/community/c80019h307278/ [15] On circles touching the incircle, Journal of Classical Geometry

http://jcgeometry.org/Articles/Volume2/JCG2013V2pp43-52.pdf [16] http://www.artofproblemsolving.com/community/q1h612004p3639577 [17] http://www.artofproblemsolving.com/community/c6h260772

(118)(119)

TÍNH CHẤT HÌNH HỌC CỦA ĐƯỜNG CONG BẬC BA

Nguyễn Tiến Lâm, Ngô Quang Dương - THPT Chuyên KHTN, Hà Nội

Bài viết trình bày số tính chất đơn giản đường cong bậc ba

1 Mở đầu

Trong hình học sơ cấp, định lý Menelaus định lý tiếng liên quan đến toán chứng minh thẳng hàng điểm Định lý chứng minh định lý Thales Dưới đây, ta đưa cách chứng minh phương pháp tọa độ

Quy ước viết, tọa độ Descartes điểmM là.xM; yM/

Định lý 1(Định lý Ménélaus). Các điểmA1,B1,C1 chia đoạn thẳngBC,CA,AB

theo tỉ số˛,ˇ,, đó˛,ˇ, 2R Ba điểmA1,B1,C1thẳng hàng khi˛ˇ D1

Chứng minh Chiều thuận. Giả sử ba điểm A1,B1, C1 thẳng hàng  đường thẳng qua ba điểm Trong hệ trục tọa độ Oxy, đường thẳng  có phương trình ax Cby C c D đặt f x/ D ax Cby C c Điểm A1 chia BC theo tỉ số ˛ nên tọa độ củaA1

A1xB ˛xC ˛ I

yB ˛yC ˛

:VìA1 2nên

axB ˛xC ˛ Cb

yB ˛yC

1 ˛ Cc D0;

dẫn đến˛ D f xB/ f xC/:

Một cách tương tự, ta cóˇ D f xC/ f xA/I D

f xA/

f xB/:Từ đó, ta có ngay˛ˇ D1:

Chiều đảo.Giả sử đường thẳng MN cắt đường thẳngAB tạiP0 giả sửP0 chia đoạnAB

theo tỉ số0:Theo chiều thuận thì˛ˇ0D 1mà ˛ˇ D1nên D 0;tức làP0 P:Suy

M; N; P thẳng hàng

Tiếp theo, ta xét định lý Carnot dạng tổng quát định lý Menelaus Nội dung định lý phát biểu đây:

Định lý 2(Định lý Carnot). Giả sử điểmAi,Bi,Ci chia cạnhBC,CA,ABtheo tỉ số˛i; ˇi; i;vớii D1; 2:Khi đó, điểmA1,A2,B1,B2,C1,C2thuộc conic

˛1˛2ˇ1ˇ212 D1

(120)

2 Một số tính chất đường cong bậc ba

2.1 Đường cong bậc ba có phương trình y D ax3 Cbx2 CcxCd

Mục đưa tính chất đường bậc ba có phương trình dạngy Dax3Cbx2CcxCd

Bổ đề 1. Khib D0, giả sửA,B,C ba điểm phân biệt thuộc đường cong bậc baK Khi đó,

A,B,C thẳng hàng khixACxBCxC D0

Chứng minh. Giả sử đường thẳng qua hai điểmA,Bcó phương trìnhW˛xCˇyC D0: Chiều thuận.Giả sửC thuộc đường thẳngAB;ta cần raxACxBCxC D0:Thật vậy, tọa độA,B,C nghiệm hệ

(

˛xCˇyC D0; ax3CcxCd Dy:

Thayy từ phương trình thứ hai vào phương trình đầu tiên, ta phương trình hồnh độ giao điểm của.C /vàlà

ax3C.˛Cb/x C Cc D0:

VìxA,xB,xC nghiệm phương trình nên theo định lý Viete, ta có ngayxACxBCxC D 0:

Chiều đảo.Giả sửxACxBCxC D0:GọiC0là giao điểm củaKvà đường thẳngAB, theo phần thuận thìxACxB CxC0 D 0;dẫn đếnxC D xC0 hayC C0:Do đó, A,B,C thẳng hàng

Từ bổ đề trên, ta thu kết đẹp liên quan tới đường cong bậc baKtrong trường hợp

b D0được trình bày mệnh đề

Mệnh đề 1. Khib D0;giả sửA,B,C ba điểm phân biệt thuộc đường congK Tiếp tuyến củaKtại điểmA,B,C cắt đường congKtại điểm thứ haiA0,B0,C0 Chứng minh

A0,B0,C0thẳng hàng khiA,B,C thẳng hàng

Chứng minh. Giả sửA,B,C ba điểm thẳng hàng thuộc đường congK Phương trình tiếp tuyến củaKtạiAlày D 3axA2 Cc/.x xA/CaxA3 CcxACd:Do vậy,xA nghiệm kép phương trình

ax3CcxCd D.3axA2 Cc/.x xA/CaxA3 CcxACd

hay

.x xA/2.x C2xA/D0:

Phương trình có hai nghiệmxA; 2xAnên hồnh độ điểmA0làxA0 D 2xA:Một cách tương tự, ta cóxB0 D 2xB; xC0 D 2xC:Suy raxA0CxB0CxC0 D 2.xACxBCxC/: Từ đây, áp dụng bổ đề ta thấy A0, B0, C0 thẳng hàng A, B, C thẳng hàng

Tiếp theo, ta xét mở rộng mệnh đề

(121)

Tạp chí Epsilon, Số 08, 04/2016

Chứng minh. GọiA3,B3,C3tương ứng giao điểm đường thẳngA1A2,B1B2,C1C2

với đường congKvà giả sửai,bi,ci hoành độ điểmAi,Bi,Ci vớii D1; 2; Theo bổ đề ta phải có

a1Ca2Ca3Db1Cb2Cb3 Dc1Cc2Cc3 D0

Suy ra†3iD1.ai Cbi Cci/ D Nhưng vìAi,Bi,Ci thuộc đường thẳng di vớii D 1; 2nên Cbi Cci D0với mọii D1; 2:Từ đó, ta phải cóa3Cb3Cc3D0:Cũng theo bổ đề ba điểmA3,B3,C3thẳng hàng Phép chứng minh hồn tất

Trong toán trên, ta cần cho hai đường thẳng d1; d2 trùng đường thẳng

A1A2; B1B2; C1C2sẽ suy biến thành tiếp tuyến đường congKvà ta thu lại kết mệnh đề Một câu hỏi đặt liệu kết cịn khơng với đường congKbất kỳ? Ta xét toán mở rộng sau đây:

Mệnh đề 3(Mở rng mnh 2). Xột ng congKWy Dax3Cbx2CcxCd, viaÔ0

và hai đường thẳngd1,d2:Giả sử rằngdi cắt đường congKtại ba điểm phân biệt Ai,Bi,Ci

vớii D1; GọiA3,B3,C3lần lượt giao điểm đường thẳngA1A2,B1B2,C1C2với đường congK Khi đó, ba điểmA3,B3,C3thẳng hàng

Hình 12.1:A3,B3,C3thẳng hàng

Chứng minh. Ta chia làm hai trường hợp A1A2,B1B2,C1C2đôi cắt

GọiA,B,C giao điểm cặp đường thẳng.B1B2; C1C2/,.C1C2; A1A2/,

.A1A2; B1B2/ Giả sử điểmAi chia đoạnBC theo tỉ số˛i; điểmBi chia đoạnCAtheo tỉ sốˇi; điểm Ci chia đoạn AB theo tỉ sối, với i D 1; 2; Áp dụng định lý Carnot cho đường congKbậc với4ABC ta có˛1˛2˛3ˇ1ˇ2ˇ3123D1:Nhưng ba điểm

Ai,Bi,Ci thẳng hàng (cùng nằm đường thẳngdi)nên áp dụng định lý Menelaus cho

4ABC với cát tuyếnd1ta có˛iˇii D1;vớii D1; 2:Suy ra˛3ˇ33 D1;nghĩa làA3,

B3,C3thẳng hàng theo định lý Ménélaus

2 Trong cặp.B1B2; C1C2/,.C1C2; A1A2/,.A1A2; B1B2/có cặp song song Lúc ta thực phép chiếu xuyên tâm biếnAi,Bi,Ci thànhAi0,Bi0,Ci0, đường

bậc baKthànhK0 Hiển nhiên tồn phép chiếu thỏa mãnA01A02,B10B20,C10C20 đôi

(122)

Ở toán trên, cho hai đường thẳngd1,d2 trùng đường thẳngA1A2,B1B2,

C1C2 suy biến thành tiếp tuyến đường congKthì kết mệnh cng ỳng trng hpb Ô0hay ỳng vi đường cong bậc ba tổng quát

2.2 Đường cong bậc ba bất kì

Ở trên, ta trình bày số tính chất đường cong bậc ba có phương trình quen thuộc Phần đề cập đến đường cong bậc ba bất kì, mà tọa độ Descartes, phương trình cho dạng hàm ẩn:

ax3x3Cay3y3Cax2yx2yCaxy2xy2Cbx2x2Cby2y2CbxyxyCcxxCcyyCd D0

Trong hệ sốax3,ay3,ax2y,axy2,bx2,by2,bxy,cx,cy,d số thực vàax3,ay3,ax2y,

axy2khơng đồng thời bằng0

Nói đến đường cong bậc ta có kết tiếng:

Định lý 3(Định lý Cayley-Bacharach). Hai đường cong bậc cắt điểm (theo định lý Bezout, hai đường cong bậcm,ncắt tạimnđiểm) Một đường cong bậc khác qua số điểm qua điểm thứ

Định lý Cayley-Bacharach [1] suy biến: thay đường cong bậc hợp đường thẳng, hay hợp conic đường thẳng, Do đó, ứng dụng định lý chứng minh điểm thẳng hàng, hay điểm thuộc conic Chẳng hạn định lý mang đến chứng minh ngắn gọn cho mệnh đề 2.1:2 đường cong bậc làK

A1; A2; A3[B1; B2; B3[C1; C2; C3 có giao điểm làAi,Bi,Ci vớii D 1; 2; Khi đường bậc suy biếnA1; B1; C1[A2; B2; C2[A3; B3đi qua số điểm nên cũng sẽ chứa điểmC3 Điều nghĩa làA3,B3,C3thẳng hàng.

Hệ quả. Các điểmA,B,C,D,P,P1,P2 thuộc đường cong bậc baK điểmA,B,C,D,P1,

P2thuộc conic giao điểm khácA,B,C,DcủaAB,CD vớiKvàP thẳng hàng

Chứng minh. AB,CDcắtKtạiM,N vàM,N,P thẳng hàng.Kvà cubic suy biếnA; B; M[ C; D; N [P; P1; P2 có giao điểm A, B, C, D, M, N, P,P1, P2 Theo định lý Cayley-Bacharach, đường bậc ba suy biếnABCDP1[M; N; P (trong đóABCDP1là conic qua điểmA,B,C,D,P1) quaP2 nênA,B,C,D,P1,P2thuộc conic

Phần đảo,A,B,C,D,P1,P2thuộc conic Xét ba đường bậc baK,A; B; M [C; D; N [ P; P1; P2,ABCDP1P2[M; N từ áp dụng định lý Cayley-Bacharach ta suy raM,N,P

thẳng hàng

(123)

Tạp chí Epsilon, Số 08, 04/2016

Hình 12.2: Một ứng dụng trực tiếp định lý Cayley-Bacharach

Tài liệu tham khảo

[1] Cayley-Bacharach theorem, Wikipedia

[2] Generalization of some triangle geometry results associated with cubics

(124)(125)

BIỂU DIỄN SỐ NGUYÊN DƯƠNG DƯỚI DẠNG TỔNG CÁC SỐ CHÍNH PHƯƠNG

Hoàng Cao Phong

Chuyên đề nghiên cứu toán Waring trường hợpkD2 Trong chuyên đề, ta chứng minhg.2/D4 Để làm điều đó, ta cần có số số ngun dương khơng thể biểu diễn đước dạng tổng hai số phương, số số biểu diễn dạng tổng ba số phương, số nguyên dương biểu diễn dạng tổng bốn số phương Ngồi ra, tìm đáp án cho câu hỏi: "Những số nguyên dương biểu diễn dạng tổng hai số phương ba số phương" Cuối cùng, xem xét toán Waring tổng quát vấn đề mở rộng cho hàmg.k/

1 Giới thiệu

Việc tìm cách biểu diễn số nguyên dương dạng tổng số phương nhiều đối tượng quan tâm, từ người yêu toán nhà toán học

Vào năm 1632, Albert Girard người đưa nhận định: số nguyên tố lẻ đồng dư với1 mod4là tổng hai số phương, điều cơng bố vào năm 1634, sau chết ông Fermat cho người đưa lời giải cho tốn, đưa vào thư ông gửi Marin Mersenne vào ngày 25 tháng 12 năm 1640

Tuy nhiên, thư, Fermat không đưa chứng minh cho khẳng định Lời giải tìm Euler vào năm 1747, ơng 40 tuổi Một cách tự nhiên, Định lí Fermat tổng hai số phương dẫn đến câu hỏi: “Tìm giá trị nhỏ củansao cho số nguyên dương biểu diễn tổng khơng qnsố phương" Đây trường hợp riêng toán Waring khik D2

Trong chuyên đề, ta chứng minhnD4và số nguyên dương biểu diễn dạng tổng hai ba số phương

Trong mục 2, ta chứng minh số nguyên tố biểu diễn dạng tổng hai số phương khơng đồng dư với mod trả lời câu hỏi: "Những số nguyên dương biểu diễn dạng tổng hai số phương?"

(126)

Trong mục 4, ta chứng minh số nguyên dương biểu diễn dạng tổng ba số phương có dạng4a.8nC7/ Mục đề cập đến hình học số học định lí Minkowski

Trong mục 5, ta đưa thêm thông tin bình luận xoay quanh tốn Waring tổng quát

2 Biểu diễn số nguyên dương dạng tổng hai số phương

Trước hết, quan tâm đến toán : “Những số nguyên tố biểu diễn dạng tổng hai số phương?”, đáp án tốn dẫn đến định lí mang tên Fermat tổng hai số phương

Bổ đề 2.1. p số nguyên tố cho trước Nếup mod 4/vàx2Cy2 chia hết cho p

xchia hết chopvày chia hết chop

Chứng minh. Giả sử.x; p/D.y; p/D1,x2 y2.mod p/dẫn đếnxp 1/p21yp 1.mod p/

suy 1/p21 mod p/, suy 11 mod p/dẫn đến điều vơ lí

Định lý 2.1 (Định lí Fermat tổng hai số phương) Số ngun tố p có thể biểu diễn được dạng tổng hai số phương khip 63 mod 4/

Chứng minh. Giả sửpD4kC3biểu diễn dạng tổng hai số phươngx; y Theo bổ đề 2,xchia hết chopvàychia hết chop Suy rapchia hết chop2 Vơ lý

Nếup D2thìp D12C12

Nếu p D 4k C1thì số phương modulo p ([5]), tồn tạia Nthỏa mãna2

1 mod p/ Đặtq Dp

p

, xét.1Cq/2 số có dạngxCay vớixD0; 1; : : : ; qvày D0; 1; : : : ; q

Do qC1/2 > p, tn ti.x1; y1/ Ô x2; y2/tha mónx1Cay1 x2Cay2 mod p/nên

.x1 x2/2 y1 y2/2.mod p/

Vì.x1 x2/ q < pp và.y1 y2/ q < pp, ta có được.x1 x2/2C.y1 y2/2 D p Định lí 2.1 chứng minh

Bây giờ, xem xét : ”Những số tự nhiên biểu diễn dạng tổng hai số phương?"

Bổ đề 2.2. Tích hai số, với số tổng số phương, số phương

Chứng minh. Giả sửm Da2Cb2vànDc2Cd2, suy ramn D.acCbd /2C.ad bc/2

(127)

Tạp chí Epsilon, Số 08, 04/2016

Chứng minh. Giả sửncó thể biểu diễn dạng hai tổng hai số phương tồn

ti lẻ:nDx2Cy2Dqtb; b; q/D1

Theo bổ đề 2,x Dqx1; y Dqy1 Điều dẫn đếnx12Cy12Dqt 2b Sau số hữu hạn bước lặp lại, ta thu được:

q.xk2Cyk2/Db;

dẫn đến điều vô lí GọiDlà tập hợp

˚

njn2N; nDx2Cy2

Giả sửti chẵn với mọii Do 22 D; pi D, bổ đề 2.2 rằngm DvớimD 2r

Y

ps1 i

Tồn x; yE N thỏa mãn x2Cy2 D m Do ti chẵn với i nên

Y

qti i D h

2

Vì vậy,

nD.xh/2C.yh/2

Định lí 2.2 chứng minh

3 Biểu diễn số nguyên dương dạng tổng của bốn số phương

Trong mục tạm thời không xét đến vấn đề biểu diễn số nguyên dương thành tổng ba số phương khơng có lời giải hồn tồn sơ cấp cho vấn đề Vả lại, lời giải cho định lí bốn số phương có phần tương tự với lời giải cho đính lí hai số phương nên ta quay lại sau chứng minh thành công định lí Lagrange tổng bốn số phương

Bổ đề 3.1. ĐặtD D ˚

nj nDx2Cy2Cz2Ct2In; x; y; z; t 2N Nếum; n2 D mn

D

Chứng minh. Giả sửmDa2Cb2Cc2Cd2vànDe2Cf2Cg2Ch2

mnD.aeCbfCcgCdh/2C.af beCch dg/2C.ag bh ceCdf /2C.ahCbg cf de/2

Suy ramn2 D

Bổ đề 3.2. Giả sửp số nguyên tố lẻ cho trước, tồn tại1k < p chokp 2D

Chứng minh. Xét hai tậpAD˚

x2 ; xD0; 1; : : : ;p

2 vàB D

˚

y2 ; y D0; 1; : : : ;p

Hiển nhiên, phần tử củaAđều phân biệt theo modulop, phần tử củaB Ta lại có:jAj C jBj DpC1 Suy tồn tạix; y2

0; 1; : : : ;p

sao chox2 y2 mod p/

(128)

Dokp Dx2Cy2C1 < p C

p2 < p

2

nênk < p ĐặtM D f1k < p; kp 2Dg Theo bổ 3.2,M Ô

Gi sml phn t nht tập hợpM,

Định lý 3.1 Ta chứng minhmD1từ suy rap 2Dvới số nguyên tốp

Chứng minh. Giả sử1 < m < p,mp 2D Ta có,mpDx2Cy2Cz2Ct2

Nếumchẵn thìmp Dx2Cy2Cz2Ct2 a20; mod 4/ Trường hợp 1:Nếux; y; z; t chẵn lẻ

xCy ;

x y ;

zCt ;

z t 2Z

.xCy/2

4 C

.x y/2

4 C

.zCt /2

4 C

.z t /2

4 D

m 2p )

m 2M

Điều mâu thuẫn bởimlà phẩn tử nhỏ trongM

Trường hợp 2:Nếux; ychẵn vàz; t lẻ

xCy ;

x y ;

zCt ;

z t 2Z

Lập luận tương tự, ta suy mâu thuẫn với định nghĩa củam Nếu m lẻ: XétS D

0;˙1;˙2; : : : ;˙m

DoS hệ thặng dư đầy đủ modm, tồn

a; b; c; d S chox a mod m/; y b mod m/; z c mod m/; t d mod m/ Vì

a2Cb2Cc2Cd2 mod m/nên tồn tạikthỏa mãna2Cb2Cc2Cd2 Dkm:

Từa2Cb2Cc2Cd2 < m2suy ra0k < m Một lần nữa, xét hai trường hợp:

Trường hợp 1: kD 0dẫn đếnaD b Dc Dd D 0suy rax y z t mod p/nên

m2 jx2Cy2Cz2Ct2 Dmpđồng nghĩa vớimchia hếtpdẫn đến mâu thuẫn

Trường hợp 2:1k < m Theo bổ đề 3.1, ta có:

mp:kmD.axCbyCczCdt /2C.bx ayCdz ct /2C.cx dy azCbt /2C.dxCcy bz at /2:

Do

X DaxCby CczCdt a2Cb2Cc2Cd2 mod m/ Y Dbx ayCdz ct ab abCdcCcd mod m/ ZDcx dy azCbt ca db acCbd mod m/ T DdxCcy bz at daCcb bc ad mod m/ X2 CY2 CZ2CT2 D m2.X12CY

2 CZ

2 CT

2 1/ D m

2

kp.kp Ddẫn đến k M Vô

(129)

Tạp chí Epsilon, Số 08, 04/2016

Chứng minh. Áp dụng bổ đề 3.1 định lí 3.1, định lí Lagrange bốn số phương trở nên hiển nhiên dẫn đếng.2/4

Cho đến giờ, ta gần hồn thiện mục đích Điều cịn lại chứng minh có số tự nhiên biểu diễn tổng ba số phương

4 Biểu diễn số nguyên dương dạng tổng ba số phương

Như nói trên, khơng có lời giải hồn tồn sơ cấp cho định lí Legendre tổng ba số phương Tuy nhiên, cách sử dụng định lí Minkowski: "bất kì tập lồi trongRn đối xứng qua gốc tọa độ tích lớn hơn2nd.L/đều chứa điểm nguyên khác không", dựa vào chứng minh Ankeny vào năm 1957, ta đưa lời giải đẹp cho toán

Định lý 4.1 Số tự nhiênmcó thể biểu diễn dạng tổng ba số phương chỉ khimkhơng có dạng4a.8nC7/

Chứng minh. Nếum số ngun dương có dạng4a.8nC7/thìm khơng thể biểu diễn tổng ba số phương:

Giả sử m biểu diễn dạng tổng ba số phương: 4a.8n C7/ D m D x2Cy2Cz2 Bởi số phương đồng dư với0; 1hoặc4theo modulo8nên

mDx2Cy2Cz2b 2.0; 1; 2; 3; 4; 5; 6/ mod 8/

Nếua > 0, ta cóm mod 4/,x2Cy2Cz2 b 2.0; 4/ mod 8/nênx2 y2 z2 mod 4/suy rax D2x1; y D2y1; z D2z1dẫn đếnx12Cy12Cz12D4a 1.8kC7/

Sau số hữu hạn lần lặp lại bước trên, ta thu được:

xa2Cy a Cz

2

a D8kC77 mod 8/;

Vô lý Sau chứng minh chiều "chỉ khi", Ta thành công việc rằngg.2/D4 Tuy nhiên, ta tiếp tục chứng minh chiều “khi" vốn phức tạp nhiều

Nếumkhơng có dạng4a.8nC7/thìmcó thể biểu diễn dạng tổng ba số phương:

Ta cần chứng minh toán trường hợp m số phương tự (square free) (nếu khơng, ta coi m ) Do tổng ba số phương đồng dư với7 mod 8/, Ta giải hai trường hợp riêng biệt:

Trường hợp 1:m3 mod 8/ Theo định lí Dirichlet cấp số cộng ([3]), tồn số nguyên tốqthỏa mãn 2q 1.mod m/vàq mod 4/ Cơng thức sau có sử dụng kí hiệu Jacobi ([5])

Ta có:

m q /D

1 q

m q

D

m q

(130)

Dq m

(Theo luật thuận nghịch bình phương)

D m q m

(Dom mod 8/)

D 2q m / D1

Tồn số nguyênb thỏa mãnb2 m mod q/hayb2 khD m.k2 Z/

Do1 k mod 4/,k D4k1Dẫn đếnb2 m mod 4q/

Xét lưới nguyên:LD˚.x; y; z/2Z3 jxy mod m/; y bz mod 2q/ Có thể tích đơn

vị là2mqtrongR3 vec-tơ.x; y; z/2Lđều thỏa mãn:

2qx2Cy2Cmz2

x2Cy2.mod m/0 mod m/ bz/2Cmz2 mod 2q/0 mod 2q/:

Hình Ellipsoid E:2qX2CY2CmZ2 4mqcó thể tích

3

.2pq m/3 p

2q m , lớn hơn2

:2q m Theo định lí Minkowski, tồn vec-tơ khác không.x; y; z/2L\E cho2qx2Cy2Cmz2 mod 2q m/nên2qx2Cy2Cmz2 D2q m

Để chứng minh m biểu diễn dạng tổng ba số phương ta chứng minh

y2Cmz2

2q biểu diễn dạng tổng hai số phương Định lí 2.2 giúp

giải điều Ta cần chứng minh: Tất số nguyên tố p thỏa mãnp > 2và

p.y2Cmz2/D2nC1([6]) đồng dư với1 mod 4/ Nếupkhông chia hếtm

y2Cmz2 mod p/dẫn đếny2 mz2.mod p/

Tồn z0sao cho zz0 mod p/nên z0y/2 m mod p/suy mlà số phương.mod p/ Hơn nữa, vìx2 m mod p/nênmlà số phương modp

1D m p D p m p D p suy

p mod 4/

Nếupchia hết m,pcũng chia hếtxvày, dẫn đếnmz2 2q m mod p2/

Domlà số phương tự nên2qlà số phương mod p và2q mod p/, ta thu được:p mod 4/

Trong hai trường hợp, ta cóp 1.mod 4/ nên y

2

Cmz2

2q biểu diễn

dạng tổng hai số phương dẫn đến m biểu diễn dạng tổng ba số phương

(131)

Tạp chí Epsilon, Số 08, 04/2016

Theo định lí Dirichlet cấp số cộng, tồn số nguyên tốq thỏa mãn:

q mod m/; q

8 ˆ <

ˆ :

1 mod 4/khim1; mod 8/ mod 8/khim mod 8/ mod 8/khim mod 8/

Dễ dàng chứng minh mlà số phương modqnên tồn tạib 2Zsao chob2 m mod p/

Lần này, ta xét lưới nguyênLD˚

.x; y; z/2Z3 jxy mod m/; y bz mod q/ hình

ellipsoidE:qX2CY2CmZ2 2mq

Sau vài bước tương tự, ta thu được: tồn vec-tơ khác không.x; y; z/2L\E:qx2Cy2C mz2 Dq m Việc chứng minh số nguyên tốp thỏa mãnp > 2vàp.y2Cmz2/D 2nC1đều đồng dư1 mod 4là tương tự Cuối cùng, ta biểu diễn y

2Cmz2 q

dạng tổng hai số phương kết thúc tốn

Trong hai trường hợp,mđều biểu diễn dạng tổng ba số phương Định lí 4.1 giải triệt để

Qua chuyên đề này, ta n=4 số nguyên nhỏ thỏa mãn số nguyên dương biểu diễn tổng n số phương mà cịn biết số biểu diễn dạng tổng hai ba số phương Cụ thể,nD2rYpsii Yqit i

vớipi mod 4/ qi mod 4/ biểu diễn tổng hai số phương khiti chẵn với mọii;m biểu diễn tổng ba số phương khimkhơng có dạng4a.8nC7/

5 Bài tốn Waring

Vào thể kỉ18, Waring, nhà toán học lỗi lạc người Anh đưa nhận xét số nguyễn dương biểu diễn tổng lập phương tổng 19 lũy thừa bậc Ơng mở rộng giả thuyết mình: Vớiklà số nguyên dương cho trước, tồn tạim(phụ thuộc vàok) cho số nguyên dương biểu diễn tổng m lũy thừa bậc k Đấy tốn Waring tổng qt

Vào năm 1906, David Hilbert, nhà toán học tiếng người Đức chứng minh thành công giả thuyết lời giải vô phức tạp

Gọig.k/là sốmnhỏ nhất, có nghĩa số nguyên dương biểu diễn dạng tổng củag.k/lũy thừa bậckvà tồn số nguyên dương biểu diễn dạng tổng (g.k/ 1) lũy thừa bậck Trong chuyên đề, ta thành công việc chứng minh

g.2/ D Gần đây, người ta chứng minh g.3/ D 9; g.4/ D 19; g.5/ D 37và

k471600000thìg.k/D

.3 2/

k

C2k

(132)

Tài liệu tham khảo

[1] Wikipedia, Albert Girard.https://en.wikipedia.org/wiki/Albert Girard

[2] Wikipedia, Pierre de Fermat.https://en.wikipedia.org/wiki/Pierre de Fermat

[3] PETE L Clark, Dirichlet’s Theorem on Primes in Arithmetic

Progressions Department of Mathematics -University of Georgia

http://math.uga.edu/ pete/4400DT.pdf

[4] H Davenport, the geometry of number, Mathematical Gazette vol 31 (1947) (206-210) [5] Titu Andreescu and Dorin Andrica, 2009 Number Theory: Structures, Examples, and

Problems (179-188)

[6] Titu Andreescu and Gabriel Dospinescu, 2008 Problem from the Books (49-67)

[7] Micheal Wong Representing integers as sum of squares University of Chicago: Department of Mathematics

[8] N c Ankeny, 1957 Sums of three squares Proceedings of the AMS (316-319) [9] Hardy, Wright, 1954 An Introduction to the Theory of Numbers Oxford

(133)

MỘT SỐ DẠNG TỐN

VỀ BẤT PHƯƠNG TRÌNH HÀM

Trịnh Đào Chiến

(Trường Cao Đẳng Sư Phạm Gia Lai)

Các toán giải bất phương trình hàm thường tốn khó Trong năm gần đây, dạng tốn loại đơi xuất đề thi chọn học sinh giỏi cấp Olympic Toán quốc tế Chẳng hạn Bài toán 3, IMO 2011:

Giả sửf WR!Rlà hàm giá trị thực xác định tập số thực thỏa mãn

f xCy/yf x/Cf f x// với số thựcxy:Chứng minh rằngf x/ D0với mọix 0:

Bài viết đề cập đến phương pháp giải lớp bất phương trình hàm dạng Đây phương pháp tham khảo để tìm tịi lời giải cho tốn bất phương trình hàm

1 Bất phương trình hàm với cặp biến tự do

Xét hàm biến số thựcf thỏa mãn tính chất sau

f xCy/f x/f y/:

Ta tìm hàmf thỏa mãn tính chất nếuf thỏa mãn thêm số điều kiện ban đầu đó, chẳng hạn (xem [1])

f x/ ax; a > 0:

Để giải toán trên, trước hết ta cần giải toán sau

Bài toán 32 Xác định hàm sốf x/thỏa mãn đồng thời điều kiện sau

i) f xCy/f x/Cf y/với mọix; y2 RI

ii) f x/0với mọix2R:

Chứng minh. Từ điều kiện toán, thayx D0ta thu đượcf 0/2f 0/vàf 0/0 Do đóf 0/D0 Vậy nên

0Df 0/Df xC x//f x/Cf x/0:

(134)

Bài toán 33 Cho trướca2R Xác định hàm sốf x/thỏa mãn đồng thời điều kiện sau

i) f xCy/f x/Cf y/với mọix; y2 R;

ii) f x/axvới mọix 2R.

Chứng minh. Xét hàm số g.x/ D ax Để ý g.xCy/ D g.x/C g.y/ Đặt f x/ D g.x/Ch.x/ Khi đó, ta thu điều kiện

i) h.xCy/h.x/Ch.y/với mọix; y 2RI

ii) h.x/0với mọix 2R:

Theo Bài tốn 1, ta cóh.x/ 0hayf x/Dax Thử lại, ta thấy hàm sốf x/Daxthỏa mãn điều kiện

Bây giờ, ta trở lại toán nêu ban đầu

Bài toán 34 Cho trướca > Xác định hàm sốf x/thỏa mãn đồng thời điều kiện sau

i) f xCy/f x/f y/với mọix; y2 R;

ii) f x/ax với mọix 2R.

Chứng minh. Nhận xét rằngf x/ > 0với mọix2 R Vậy ta logarit hóa hai vế bất

đẳng thức điều kiện cho

i) lnf xCy/lnf x/Clnf y/với mọix; y 2R;

ii) lnf x/.lna/x với mọix 2R

Đặt lnf x/ D'.x/, ta thu

i) '.xCy/'.x/C'.y/với mọix; y 2R;

ii) '.x/.lna/x với mọix 2R

Ta nhận dạng Bài toán Vậy'.x/D.lna/x Suy raf x/Dax Thử lại, ta thấy hàm sốf x/ Dax thỏa mãn điều kiện

Nhận xét rằng, toán giải tập xác địnhRcủa hàm số thay khoảng mởU chứa0sao cho với mọix; y2 U thìxCy 2U

Một câu hỏi tự nhiên đặt ra: Trong Bài tốn 3, thay hàm sốg.x/Dax hàm số để tốn có nghiệm khơng tầm thường?

Nhận xét

Với0 < a < 1thìax > 1Cx,8x < 0vàax 1Cx,8x 0;

(135)

Tạp chí Epsilon, Số 08, 04/2016

Bài toán 35 Giả sửU là khoảng mở chứa0sao cho với mọix; y 2U thìxCy 2U Xác định các hàm sốf WU !Rthỏa mãn đồng thời điều kiện sau

i) f xCy/f x/f y/với mọix; y2 U;

ii) f x/1Cxvới mọix 2U. Chứng minh. Bởi i), ta có

f x/ Dfx C

x

f2x

0; 8x 2U:

Nếuf x0/D0,

0Df x0/Dfx0 C

x0

f2x0

:

Do đófx0

D0:Quy nạp, ta cófx0 2n

D0với số nguyên dươngn Tuy nhiên, từ ii) suy rằngf x/ > 0với mọix 2U vàxgần Do điều mâu thuẫn Vậy

f x/ > 0; 8x 2U:

Tiếp theo, từ i) ii), ta thấy rằngf khả vi điểmx 2U vàf0.x/ Df x/ Thật vậy, từ i) ii), vớih > 0đủ nhỏ, ta có

f xCh/ f x/f x/f h/ f x/DŒf h/ 1f x/hf x/:

Do

f xCh/ f x/

h f x/:

Mặt khác, từ i) ii), vớih > 0đủ nhỏ, ta có

f x/ Df xCh h/f xCh/f h/.1 h/f x Ch/:

Suy

.1 h/f x/Chf x/.1 h/f xCh/:

Do

hf x/.1 h/Œf xCh/ f x/;

hay

f xCh/ f x/

h

f x/ h:

Vậy, vớih > 0đủ nhỏ, ta có

f x/ f xCh/ f x/

h

f x/ h:

Tương tự, bất đẳng thức chiều ngược lại, vớih < 0đủ nhỏ Do đó, ta có

f0.x/D lim

h!0

f xCh/ f x/

h tồn bằngf x/, với mọix U Từ đó, với mọix 2U,

ta có

f x/

ex

0

D f

0.x/ f x/ ex D0:

(136)

Như vậy, vớig.x/Dax hoặcg.x/D1Cx, Bài toán Bài toán giải Một câu hỏi đặt ra: Với lớp hàmg.x/nào tốn tổng qt giải được? Ta có kết sau

Định lý Giả sửU là khoảng mở chứa0sao cho với mọix; y 2U thìxCy 2U Nếu hàm sốf WU !Rthỏa mãn đồng thời điều kiện sau

i) f xCy/f x/f y/với mọix; y2 U;

ii) f x/ g.x/, 8x U; đó g.x/ là hàm số cho trước khả vi tại 0; g.0/ D 1, g0.0/Dk, thìf x/Dekx.

Chứng minh. Tương tự lời giải Bài toán 4, từ điều kiện cho, ta suy raf x/ > 0với

x 2U Giả sử rằngf x/là hàm số thỏa mãn điều kiện định lý Thế thì, vớih > 0đủ nhỏ, ta có

f xCh/ f x/f x/f h/ f x/ D.f h/ 1/f x/.g.h/ 1/f x/:

Do

f xCh/ f x/

h

g.h/ g.0/ h f x/:

Mặt khác, từ i) ii), vớih > 0đủ nhỏ, ta có

f x/ Df xCh h/f xCh/f h/f xCh/g h/:

Vì hàmg.x/khả vi tại0nên liên tục điểm Do đó, vớih > 0đủ nhỏ, ta cóg h/ > Khi đó, vớih > 0đủ nhỏ, ta có

f xCh/ f x/ g h/

g h/ f x/ D

g h/ g.0/ hg h/ f x/:

Vậy vớih > 0đủ nhỏ, từ kết trên, ta có

g.h/ g.0/

h f x/

f xCh/ f x/

h

g h/ g.0/ g h/ f x/:

Tương tự, bất đẳng thức chiều ngược lại, vớih < 0đủ nhỏ Do đó, ta có

f0.x/D lim

h!0

f xCh/ f x/

h tồn bằngg

0.0/f x/Dkf x/, vớix2 U.

Từ đó, vớix2 U, ta có

f x/ ekx

0

D f

0.x/ kf x/ ekx D

kf x/ kf x/ ekx D0:

(137)

Tạp chí Epsilon, Số 08, 04/2016

Rõ ràngf x/ Dekxthỏa mãn điều kiện i) Nếu giả thiết toán có thêm điều kiệng.x/ekx, vớix2 U, hàm sốf x/Dekx thỏa mãn tất điều kiện toán

Từ kết trên, ta có

Hệ Giả sửU là khoảng mở chứa vàf WU !Rthỏa mãn điều kiện i) với mọix; y 2U

sao choxCy 2U Nếuf khả vi tại0; f 0/D1f0.0/ Dkthìf x/Dekx,x2U. Chứng minh. Áp dụng Định lý 1, vớig.x/Df x/,8x 2U, ta có điều phải chứng minh

Hệ Giả sửF là hàm xác định khoảng mởU chứa thỏa mãn F x Cy/F x/CF y/

với mọix; y 2U sao choxCy 2U NếuF bị chặn hàmGkhả vi tại0và thỏa mãnG.0/D1, thìF x/Dkx,x 2U, đóklà số.

Chứng minh. Áp dụng Định lý 1, vớif x/De F x/ vàg.x/De G.x/;ta có đpcm Tương tự phương pháp chứng minh Định lý 1, ta có kết sau

Định lý Giả sửU là khoảng mở chứa0sao cho với mọix; y 2U thìxCy 2U Nếu hàm sốf WU !Rthỏa mãn điều kiện sau

f xCy/f x/g.y/; 8x; y 2U;

trong đóg.x/là hàm số cho trước khả vi 0,g.0/D 1; g0.0/ Dk;thì nghiệm bất phương trình hàm có dạngf x/ DC ekx; C là số.

Hệ Ta cóf x/Dekx g.x/Dekx là nghiệm hệ bất phương trình hàm

(

f xCy/f x/g.y/ g.xCy/g.x/f y/

với điều kiệnf 0/D1,g.x/là khả vi 0,g.0/D1g0.0/ Dk

Chứng minh. Từ bất phương trình hàm thứ nhất, áp dụng Định lý ta đượcf x/ DC ekx (C số) Vìf 0/D1, nênC D1 Do

f x/ Dekx:

Tương tự, từ bất phương trình hàm thứ hai, áp dụng Định lý ta có

g.x/Dekx:

(138)

Định nghĩa Hàmg.x/xác định khoảng mở U chứa0 được gọi hàm tựa bởil tại nếu tồn hàmk.x/xác định trên U sao cho k.0/ D g.0/,k0.0/ D l tồn và k.x/g.x/với mọix2 U.

Hệ Bất phương trình hàm

f xCy/f x/g.y/;

trong đóg là hàm cho trước xác định trênI vớig.0/ D 1và hàm tựa bởi l tại 0; nghiệm không âmf khi khielx g.x/trênI và trường hợp nghiệm khơng âm có dạngf x/DC elx, đóC 0là số.

Chứng minh. Giả sử f x/ nghiệm không âm bất phương trình hàm cho Vì

g.x/k.x/trênU;nên ta có

f xCy/f x/k.y/;

trong đók.x/thỏa mãnk0.0/Dl vàk.0/Dg.0/D1 Áp dụng Định lý vào bất phương trình hàm này, ta cóf x/DC elx, đóC 0là số Rõ ràng,f x/DC elx nghiệm không âm bất phương trình hàm cho nếuelx g.x/trênU:

Từ Hệ 4, ta sáng tác toán, chẳng hạn sau

Bài toán 36 Tìm tất hàm sốf x/, xác định khoảng mở e;1/, thỏa mãn hệ bất phương trình hàm sau

(

f xCy/f x/logf y/ f x/xCe

Chứng minh. Trên khoảng mở e;1/, nghiệm dươngf suy bất phương trình hàm thứ hai Ngồi ra, từ hệ bất phương trình hàm cho, ta cóf 0/De Áp dụng Hệ trường hợpg.x/ Dlogf x/là hàm tựa

e tại0;qua hàmk.x/D log.xCe/ Do

đó, theo chứng minh Hệ , ta cóf x/ D f 0/exe D e1C x

e Thử lại, ta thấy hàm số f x/De1Cxe thỏa mãn hệ bất phương trình hàm cho khoảng mở e;1/

Bài toán 37 Trên khoảng mở chứa0có nghiệm hệ bất phương trình hàm

(

f xCy/f x/ef y/ f x/x2

Chứng minh. Giả sử f x/ nghiệm xác định khoảng mở chứa0 Thế thì, bất phương trình hàm thứ hai,f x/là khơng âm Từ hệ bất phương trình cho suy

f 0/D0 Áp dụng Hệ trường hợpg.x/Def x/ hàm tựa bởi0tại0, qua hàm

k.x/Dex2 Hơn nữa, vìf 0/D0, nên ta cóf x/0thỏa mãn bất phương trình hàm thứ hai khoảng khơng mở chứa0

(139)

Tạp chí Epsilon, Số 08, 04/2016

Định lý Giả sửU là khoảng mở chứa0sao cho với mọix; y U thìxCy U Xét bất phương trình hàm

f xCy/f x/g.y/Cf y/g.x/; 8x; y 2U;

trong đóg.x/là hàm giới nội, khả vi 0,g.0/D 1g0.0/ Dk Thế thìf x/ 0 hàm số thỏa mãn bất phương trình cho, với điều kiện lim

x!0 f x/

x D0 Chứng minh. Giả sử rằngf x/là nghiệm bất phương trình cho, với điều kiện

lim

x!0 f x/

x D0:

Thế thì, vớih > 0đủ nhỏ, ta có

f xCh/f x/g.h/Cf h/g.x/

hay

f xCh/ f x/ g.h/ 1/f x/Cf h/g.x/:

Do

f xCh/ f x/

h

g.h/ g.0/

h f x/C f h/

h g.x/:

Mặt khác, ta có

f x/Df xCh h/f xCh/g h/Cf h/g.xCh/

hay

g h/.f x/ f xCh//g h/f x/ f x/Cf h/g.xCh/

Vì hàmg.x/khả vi tại0nên liên tục điểm Do đó, vớih > 0đủ nhỏ, ta cóg h/ > Vậy, vớih > 0đủ nhỏ, ta có

f xCh/ f x/

h

.g h/ 1/f x/Cf h/g.xCh/ g h/

D g h/ g.0/

hg h/ f x/C

f h/

hg h/g.xCh/:

Vậy vớih > 0đủ nhỏ, từ kết trên, ta có

g.h/ g.0/

h f x/C f h/

h g.x/

f xCh/ f x/ h

g h/ g.0/

hg h/ f x/C

f h/

hg h/g.xCh/:

Tương tự, bất đẳng thức chiều ngược lại, vớih < 0đủ nhỏ Do đó, ta có

f0.x/D lim

h!0

(140)

tồn bằngg0.0/f x/Dkf x/, vớix 2U, lim

x!0 f x/

x D0vàg.x/là hàm giới nội

Từ đó, vớix2 U, ta có

f x/

ekx

0

D f

0.x/ kf x/ ekx D

kf x/ kf x/ ekx D0:

Do f x/ D C ekx (C số) Hơn nữa, từ lim

x!0 f x/

x D 0;suy C D Vậy f x/0là hàm số thỏa mãn yêu cầu, với điều kiện lim

x!0 f x/

x D0 2 Bất phương trình hàm dạng cộng – nhân tính

Phần đề cập đến việc giải hệ bất phương trình hàm, với dạng sau

Dạng “cộng”:f aCx/˛Cf x/,f bCx/ˇCf x/,x 2R;

Dạng “cộng - nhân”:f aCx/˛f x/,f bCx/ˇf x/,x 2R;

Dạng “nhân - cộng”:f ax/˛Cf x/,f bx/ˇCf x/,x2I,I R;

Dạng “nhân”:f ax/ ˛f x/,f bx/ ˇf x/,x I,I R; đóa; b; ˛; ˇlà

các số thực cho trước

Chú ý rằng, nếu˛ Df a/,ˇ Df b/, hệ bất phương trình hàm dạng “cộng” thu hẹp bất phương trình hàm Cauchy cổ điển

f xCy/f x/Cf y/; 8x; y 2R:

Trước hết, ta nhắc lại rằng, tập hợpM trù mật tập số thựcRnếu lân cận điểm tùy ý tậpRđều có điểm tậpM Chẳng hạn, tậpQcác số hữu tỷ tập trù mật tậpR

Tính chất sau kết quen thuộc (Định lý Kronecker), tìm thấy chứng minh tài liệu lý thuyết bản:“Nếu ablà số thực không thông ước với nhau, tập AD fmaCnbjm; n2Zgtrù mật trongR”.

Hơn nữa, ta chứng minh kết sau

Bổ đề Giả sửa; b 2Ra < < blà số cho trước Ký hiệu

AD fmaCnbjm; n2Ng:

1) Nếu b

a … Q, tậpAtrù mật trongR.

2) Nếu b

a Q, tồn tạid > 0sao choAD fkdjk 2Zg.

(141)

Tạp chí Epsilon, Số 08, 04/2016

Bổ đề Giả sửa; b 2R0 < a < < blà số cho trước Ký hiệu

M D fambnjm; n2Ng:

1) Nếu logb

loga …Q, tậpM trù mật trong.0;1/

2) Nếu logb

loga 2Q, tồn tạid > 0sao choM D

n

dkjk 2Z

o

.

Bây giờ, ta chứng minh định lý sau

Định lý 10 (Dạng “cộng”) Giả sửa; b; ˛; ˇ là số thực cho trước thỏa mãna < < b, ˛

a D ˇ

b, giả sử hàmf WR!Rliên tục điểm.

1) Nếu b

a … Q, thìf thỏa mãn hệ bất đẳng thức hàm

f aCx/ ˛Cf x/; f bCx/ˇCf x/; 8x 2R 1/

khi khif x/ DpxCf 0/,x 2R, đóp WD ˛

a.

2) Nếu b

a 2Q, tồn nghiệm hàm liên tụcf WR!Rcủa hệ phương trình hàm tương ứng

f aCx/ D˛Cf x/; f bCx/DˇCf x/; 8x 2R 2/ sao chofˇˇŒ0;d  Df0, đód WDminfmaCnb > 0jm; n2Ngtồn tại, số dương

f0 WŒ0; d  !Rlà hàm liên tục cho trước thỏa mãn điều kiện

f0.d /D ˛

ad Cf0.0/:

Hơn nữa, nếuf0là đơn điệu nghiêm ngặt, trùng với hàmf trên đoạnŒ0; d  Chứng minh. 1)Từ (1), dễ dàng suy

f maCx/m˛Cf x/; f nbCx/nˇCf x/; 8m; n2N; x2 R:

Trong bất đẳng thức trên, thayxbởinbCx, ta có

f maCnbCx/ m˛Cf nbCx/m˛CnˇCf x/:

Do

f maCnbCx/ m˛CnˇCf x/; 8m; n2N; x 2R:

Đặtp WD ˛ a D

ˇ

b, ta viết bất đẳng thức dạng

(142)

trong đó, theo Bổ đề 1, tậpAD fmaCnbjm; n2Ngtrù mật trongR

Giả sử rằngx0là điểm mà hàmf liên tục vàx giá trị thực tùy ý Bởi tính chất trù mật củaAtrong tậpR, tồn dãy.tn/sao cho

tn 2A n2N/; xlim

!C1tnDx0 x:

Từ bất đẳng thức (3), ta có

f tnCx/ ptnCf x/; 8n2 N:

Chon! 1, tính liên tục hàmf tạix0, ta thu

f x0/p.x0 x/Cf x/; 8x2 R:

Bây giờ, để chứng minh phần đảo, thayxbởix t (3), ta dược

f x/ptCf x t /; 8t 2A; x 2R:

Chọn điểmx 2Rcố định tùy ý và, tính trù mật củaAtrongR, dãy.tn/sao cho

tn2 A n2 N/; nlim

!C1tnDx x0:

Thế thì, ta có

f x/ptnCf x tn/; 8n2 N:

Chon! 1, tính liên tục hàmf tạix0, ta thu bất đẳng thức

f x/p.x x0/Cf x0/; 8x2 R:

Do đó, ta có

f x/Dp.x/C.f x0/ px0/; 8x 2R:

Ta có điều phải chứng minh

2)Từ (2), dễ dàng suy

f maCnbCx/D ˛

a.maCnb/Cf x/; 8m; n2 N; x2R: 4/

Theo Bổ đề 1, phần 2, sốd WDminfmaCnb > 0jm; n2Nglà xác định số dương Hơn

nữafmaCnb > 0jm; n2Ng D fkd jk2Zg Dó đó, (4) có dạng

f kd Cx/D ˛

akd Cf x/; 8k2Z; x 2R:

Dễ dàng thấy hệ phương trình hàm tương đương với phương trình

f d Cx/D ˛

(143)

Tạp chí Epsilon, Số 08, 04/2016

Bây giờ, ta xác địnhf1W.d; 2d !Rbởi công thức

f1.x/ WD ˛

ad Cf0.x d /; x2 d; 2d :

Giả sử rằngfn W.nd; nC1/d !R.n2N/đã xác định Thế thì, ta xác định

fnC1 W nC1/d; nC2/d !R

bởi hệ thức truy hồi

fnC1.x/ WD ˛

ad Cfn.x d /; x nC1/d; nC2/d ; n2N:

Tương tự, giả sử

f 1.x/WD ˛

ad Cf0.xCd /; x 2Œ d; 0/:

Giả sử ta có định nghĩa

f n.x/WDŒ nd; nC1/d / !R.n2N /:

Thế thì, ta định nghĩa

f nC1/.x/WD ˛

ad Cf n.x Cd /; x 2Œ nC1/d; nd /; n2N:

Dễ dàng kiểm tra rằngf WR!R, xác định

f x/D

8 ˆ ˆ <

ˆ ˆ :

f n.x/ khix 2Œ nd; nC1/d / f0.x/ khix 2Œ0; d 

fn.x/ khix 2.nd; nC1/d 

.n2N/;

thỏa mãn hệ (2), hàm liên tục vàfˇˇŒ0; d  Df0 Định lý chứng minh hoàn toàn

Định lý 11 (Dạng “cộng – nhân”) Giả sửa; b 2R˛; ˇ > 0là số cho trước thỏa mãn a < < b, log˛

a D

logˇ

b và giả sử hàmf WR!Rliên tục điểm.

1) Nếu b

a … Q, thìf thỏa mãn hệ bất đẳng thức hàm

f aCx/˛f x/; f bCx/ˇf x/; x 2R 5/

khi khif x/ Df 0/epx,x 2R, đóp WD log˛

a .

2) Nếu b

a 2Q, tồn nghiệm hàm liên tụcf WR!Rcủa hệ phương trình hàm tương ứng

f aCx/D˛f x/; f bCx/Dˇf x/; x2 R 6/

sao chofˇˇŒ0; d  Df0, đód WDminfmaCnb > 0jm; n2Ngtồn tại, số dương

f0 WŒ0; d  !Rlà hàm liên tục cho trước thỏa mãn điều kiện

f0.d / Df0.0/eloga˛d:

(144)

Chứng minh. 1)Từ (5), dễ dàng suy

f maCx/ ˛mf x/; f nbCx/ ˇnf x/; 8m; n2 N; x2 R:

Trong bất đẳng thức trên, thayxbởinbCx, ta có

f maCnbCx/˛mf nbCx/˛mˇnf x/:

Do

f maCnbCx/˛mˇnf x/; 8m; n2 N; x2R:

Đặtp WD log˛

a D

logˇ

b , ta viết bất đẳng thức dạng

f t Cx/eptf x/; t A; x2 R 7/

trong đó, theo Bổ đề 1, tậpAD fmaCnbjm; n2Ngtrù mật trongR

Giả sử rằngx0là điểm mà hàmf liên tục vàx giá trị thực tùy ý Bởi tính chất trù mật củaAtrong tậpR, tồn dãy.tn/sao cho

tn2 A n2 N/; nlim

!C1tnDx0 x:

Từ bất đẳng thức (7), ta có

f tnCx/eptnf x/; n2N:

Chon! 1, tính liên tục hàmf tạix0, ta thu

f x0/ep.x0 x/f x/; x2 R:

Bây giờ, để chứng minh phần đảo, thayxbởix t (7), ta dược

f x/eptf x t /; t A; x2R:

Chọn điểmx 2Rcố định tùy ý và, tính trù mật củaAtrongR, dãy.tn/sao cho

tn2 A n2 N/; lim

n!C1tnDx x0:

Thế thì, ta có

f x/ eptnf x tn/; n2N:

Chon! 1, tính liên tục hàmf tạix0, ta thu bất đẳng thức

f x/ep.x x0/f x0/; x2 R:

Do đó, ta có

f x/Dep.x/.f x0/ px0/; x2 R:

Ta có điều phải chứng minh

(145)

Tạp chí Epsilon, Số 08, 04/2016

Định lý 12 (Dạng “nhân – cộng”) Giả sửa; b; ˛; ˇlà số thực cho trước thỏa mãn < a < < b; ˛

loga D ˇ

logb; và giả sử hàmf WI !Rliên tục điểm.

1) Nếu logb

loga …Q, thìf thỏa mãn hệ bất đẳng thức hàm

f ax/˛Cf x/; f bx/ˇCf x/; x 2I; 8/ thì

i) Trường hợpI D.0;1/:f x/DplogxCf 1/,x > 0,

ii) Trường hợpI D 1; 0/: f x/ D plog x/Cf 1/,x < 0, p WD ˛

loga.

2) Nếu logb

loga 2Q, tồn nghiệm hàm liên tụcf WI !Rcủa hệ phương trình hàm tương ứng

f ax/D˛Cf x/; f bx/ DˇCf x/; x 2I 9/ sao chofˇˇŒ1; d  D f0, d WD minfambn> 1jm; n2 Ngtồn tại, lớn hơn1 f0 WŒ1; d  !Rlà hàm liên tục cho trước thỏa mãn điều kiện

f0.d /D ˛

logalogd Cf0.1/:

Hơn nữa, nếuf0là đơn điệu nghiêm ngặt, trùng với hàmf trên đoạnŒ1; d  Chứng minh. 1) i)Giả sửI D.0;1/ Từ (8), chứng minh tương tự phần trên, ta có

f ambnx/m˛CnˇCf x/; m; n2N; x > 0:

Đặtp WD ˛

loga D ˇ

logb, ta viết bất đẳng thức dạng

f ambnx/ plog.ambn/Cf x/; m; n2N; x > 0;

hay

f tx/plogt Cf x/; t M; x > 0; 10/

trong đó, theo Bổ đề 2, tậpM D fambnjm; n2Ngtrù mật trongI

Giả sử rằngx0 > 0là điểm mà hàmf liên tục vàx > 0là giá trị tùy ý Bởi tính chất trù mật củaM tậpI, tồn dãy.tn/sao cho

tn2 M n2N/; nlim

!C1tn D x0

(146)

Từ bất đẳng thức (10), ta có

f tnx/plogtnCf x/; n2N:

Chon! 1, tính liên tục hàmf tạix0, ta thu

f x0/plogx0

x Cf x/; x > 0:

Bây giờ, để chứng minh phần đảo, thayxbởi x

t (10), chọn dãy.tn/sao cho tn2 M n2N/; nlim

!C1tn D x x0:

Thế thì, ta có

f x/ plog x

x0 Cf x0/; x > 0:

Do đó, ta có

f x/Df x0/ plogx0Cplogx; x > 0:

Phần i) chứng minh

ii)Giả sử rằngI D 1; 0/ Ta xét hàmgW.0; 1/!Rxác định công thức

g.x/Df x/; x < 0;

thỏa mãn hệ (8) chứng minh tương tự chứng minh phần i)

2)Phần chứng minh tương tự chứng minh Định lý 4, phần

Hệ 13 Giả sửa; b; ˛; ˇ Rthỏa mãn giả thiết Định lý 4, phần Nếu hàm

f W h 1; 0/[.0; 1/i !Rthỏa mãn hệ bất đẳng thức.8/và khoảng.1; 0/,

.0; 1/tồn điểm mà hàmf liên tục, thì f x/ D

(

plogxCf 1/; khix2 0;1/ plog x/Cf 1/; khix2 1; 0/ trong đóp WD ˛

loga.

Chú ý Giả sửa; b; ˛; ˇlà số thực cho trước thỏa mãn0 < a < < b ˛

loga D ˇ

logb. Nếu02I, không tồn hàm thỏa mãn hệ.8/:

Thật vậy, (8) đặtxD0, thì0˛,0ˇ, mâu thuẫn với giả thiết˛ˇ <

Định lý 14 (Dạng “nhân”) Giả sử a; b; ˛; ˇ là số thực cho trước thỏa mãna < < b,

log˛

loga D

logˇ

(147)

Tạp chí Epsilon, Số 08, 04/2016

1) Nếu logb

loga …Q, thìf thỏa mãn hệ bất đẳng thức hàm

f ax/ ˛f x/; f bx/ ˇf x/; x 2I; 11/ thì

i) Trường hợpI D.0;1/:f x/Df 1/xp,x > 0,

ii) Trường hợpI D 1; 0/:f x/Df 1/ x/p,x < 0, đóp WD log˛

loga.

2) Nếu logb

loga Q, tồn nghiệm hàm liên tụcf W I ! R.I D 0;1/ hoặcI D 1; 0//của hệ phương trình hàm tương ứng

f ax/D˛f x/; f bx/ Dˇf x/; x 2I; 12/ sao chofˇˇŒ1; d  D f0, đód WD minfambn> 1jm; n2Ngtồn tại, lớn hơn f0 WŒ1; d  !Rlà hàm liên tục cho trước thỏa mãn điều kiện

f0.d /Df0.1/dlog ˛ loga:

Hơn nữa, nếuf0là đơn điệu nghiêm ngặt, trùng với hàmf trên đoạnŒ1; d  Chứng minh. 1) i)Giả sửI D.0;1/ Từ (11), chứng minh tương tự phần trên, ta có

f ambnx/ ˛mˇnf x/; m; n2 N; x > 0:

Đặtp WD log˛

loga D

logˇ

logb, ta viết bất đẳng thức dạng f ambnx/.ambn/pf x/; m; n2N; x > 0;

hay

f tx/tpf x/; t 2M; x > 0; 13/

trong đó, theo Bổ đề 2, tậpM D fambnjm; n2Ngtrù mật trongI

Giả sử rằngx0 > 0là điểm mà hàmf liên tục vàx > 0là giá trị tùy ý Bởi tính chất trù mật củaM tậpI, tồn dãy.tn/sao cho

tn2 M n2N/; nlim

!C1tn D x0

x :

Từ bất đẳng thức (13), ta có

f tnx/tnpf x/; n2N:

Chon! 1, tính liên tục hàmf tạix0, ta thu

f x0/x0 x

p

(148)

Bây giờ, để chứng minh phần đảo, thayxbởi x

t (13), chọn dãy.tn/sao cho tn2 M n2N/; nlim

!C1tn D x x0:

Thế thì, ta có

f x/

x

x0

p

f x0/; x > 0:

Phần i) chứng minh

ii)Giả sử rằngI D 1; 0/ Ta xét hàmgW.0; 1/!Rxác định công thức

g.x/Df x/; x < 0;

thỏa mãn hệ (11) chứng minh tương tự chứng minh phần i) 2) Phần chứng minh tương tự chứng minh Định lý 4, phần

Chú ý Giả sửa; b; ˛; ˇlà số thực cho trước thỏa mãn0 < a < < b log˛

loga D

logˇ

logb. NếuI DRhoặcI DŒ0; 1/hoặcI D 1; 0f WI !Rthỏa mãn hệ.11/;thì

f 0/D0:

Thật vậy, hai giả thiết˛ < < ˇhoặcˇ < < ˛và, nữa,f 0/.1 ˛/0

vàf 0/.1 ˇ/0, ta suy raf 0/D0 Từ Chú ý này, ta có

Chú ý

i) Giả sửf W Œ0; 1/! Rthỏa mãn hệ.11/: Nếufˇˇ.0;1/ a; b; ˛; ˇ thỏa mãn tất cả các giả thiết Định lý 7, phần 1, thì

f x/ D

(

f 1/xp khix 2.0; 1/; khix D0; trong đóp WD log˛

loga.

ii) Giả sửf W 1; 0!Rthỏa mãn hệ.11/:Nếufˇˇ 1; 0/ a; b; ˛; ˇthỏa mãn tất cả các giả thiết Định lý 7, phần 1, thì

f x/ D

(

f 1/ x/p khix2 1; 0/;

0 khixD0;

trong đóp WD log˛

loga.

(149)

Tạp chí Epsilon, Số 08, 04/2016

i) Nếu hàmf Wh 1; 0/[.0; 1/i !Rthỏa mãn hệ bất đẳng thức.11/và mỗi

khoảng 1; 0/,.0;1/tồn điểm mà hàmf liên tục, thì

f x/D

(

f 1/xp khix 2.0; 1/ f 1/ x/p khix 1; 0/

trong đóp WD log˛

loga.

ii) Nếu hàmf WR!Rthỏa mãn hệ bất đẳng thức.11/và khoảng 1; 0/

.0; 1/tồn điểm mà hàmf liên tục, thì

f x/D

8 ˆ ˆ <

ˆ ˆ :

f 1/xp khix 2.0;1/;

0 khix D0;

f 1/ x/p khix 1; 0/

trong đóp WD log˛

loga.

Chú ý Ta ln có định lý tương tự Định lý - Định lý 7, với hàmf thỏa mãn các bất đẳng thức có dấu ngược lại.

Tài liệu tham khảo

[1] Nguyễn Văn Mậu,Bất đẳng thức, định lý áp dụng, Nhà xuất Giáo dục, 2006 [2] Trịnh Đào Chiến,Một số dạng bất phương trình hàm dạng bản, Kỷ yếu Hội nghị khoa

học chuyên đề chuyên Toán bồi dưỡng học sinh giỏi Trung học phổ thông, Hà Nội -Nam Định, 26-28/11/2010

[3] Th M Rassias,Functional equations, inequalities and applications, 73 - 89, Kluwer Aca-demic Publishers, 2003

(150)(151)

BÀI TOÁN HAY LỜI GIẢI ĐẸP

Trần Nam Dũng - Đại học Khoa học Tự nhiên - ĐHQG TP.HCM

LỜI GIỚI THIỆU

Chuyên mục lấy cảm hứng từ viết thầy Nguyễn Duy Liên toán số6trong kỳ thi IMO2001với5cách giải khác Mục để dành viết toán hay, lời giải đẹp câu chuyện thú vị xung quanh tốn lời giải

Tên chun mục mượn từ tên nhóm người yêu toán Facebook anh Nguyễn Văn Lợi sáng lập “Bài toán hay – Lời giải đẹp – Đam mê toán học” Chuyên mục ghi nhận đề cử bạn đọc chọn đăng kỳ1;

bài toán

Như biết, Euler giới thiệu công thức tiếng tam giác, thể mối liên hệ bán kính đường tròn ngoại tiếp nội tiếp khoảng cách hai tâm đường tròn tam giác Đó

d2DR2 2Rr

với.O; R/; I; r/lần lượt đường tròn ngoại tiếp, nội tiếp tam giác vàd DOI:

Cịn Nicolaus Fuss, học trò bạn L.Euler giới thiệu cơng thức khác, tương tự hình thức đẹp không tứ giác lưỡng tâm, tức tứ giác vừa nội tiếp đường tròn ngoại tiếp đường tròn khác

Bài tốn 38 Trong tứ giácABCDngoại tiếp đường trịn.I; r/và nội tiếp đường trịn O; R/thì

1

.R d /2 C

.RCd /2 D r2:

(152)

GọiK; Llà tiếp điểm I; r/lênAB; BC:Do tứ giácABCDnội tiếp nênA; C hai góc bù nhau, suy

∠BAI C∠I CB D90ı:

Ta ý rằngIK DILDr nên hai tam giácAIKvàCILcó thể ghép lại thành tam giác vng có hai cạnh góc vng làI C; IAnhư hình vẽ

Từ đó, ta thấy tổng diện tích hai tam giác tính theo hai cách:

r.AKCCL/DAI CI:

Ngoài ra, theo định lý Pythagorean tam giác thì:

.AKCCL/2 DAI2CC I2:

Từ ta thấy

r2.AI2CC I2/DAI2C I2 , r2 D

1 AI2 C

1 C I2:

Giả sửAI; CI cắt đường tròn.O; R/theo thứ tự tạiF; E:Khi đóEF đường kính của.O; R/

vìE; F trung điểm hai cung chung dâyBD

Theo công thức đường trung tuyến tam giácEF I

EI2CF I2 D2I O2CEF

2 D2.d

CR2/:

Theo tính chất phương tích củaI với.O; R/thì

AI F I DCI EI DR2 d2:

Do

1 r2 D

1 AI2 C

1 C I2 D

F I2CEI2 R2 d2/2 D

2.R2Cd2/ R2 d2/2 D RCd /

2

C.R d /2 R2 d2/2 D

1

(153)

Tạp chí Epsilon, Số 08, 04/2016 Định lý chứng minh

Từ đây, ta chứng minh tứ giác lưỡng tâm, ta ln cóR rp2 Thật vậy,

1 r2 D

1

.RCd /2 C R d /2

2

q

.RCd /2.R d /2

D

R2 d2 R2

nênR rp2 Đẳng thức xảy khid D0và ta dễ dàng chứng minh lúc đó,ABCDphải hình vng

(154)(155)

EUCLID VÀ CƠ SỞ CỦA HÌNH HỌC

Ngơ Bảo Châu, Richard Fitzpatrickn

Trong số này, chuyên mục điểm sách Epsilon trân trọng giới thiệu với độc giả điểm sách GS Ngô Bảo Châu giáo sư Richard Fitzpatrick cho sách vừa đời "Euclide Cơ sở hình học"

A Lời giới thiệu Giáo sư Ngô Bảo Châu cho dịch tiếng Việt

Euclid viết sáchCơ sở hình họcở Alexandria khoảng 300 năm trước Công nguyên Đây thời kỳ Hellenistic triết học cổ đại, thời kỳ mà triết học cổ đại lan toả tới vùng đất chịu ảnh hưởng văn hoá Hy Lạp mà tiêu biểu thành Alexandria bên bờ Phi Địa Trung Hải Nét chung triết học thời kỳ Hellenistic, phần thể sáchCơ sở hình học, tư đạt đến mức tinh tuý, có lẽ tính bay bổng thời kỳ trước Socrates sức mạnh tư Plato, Socrates

Người ta cho hầu hết nội dung sáchCơ sở hình họcđược truyền lại từ tiền nhân Pythagoras, Plato, Eudoxus Tuy nhiên, khác với Pythagoras Plato, Euclid loại bỏ triệt để yếu tố siêu hình gán cho số hình Các số hữu tỉ khơng cịn coi minh chứng cho hài hoà vũ trụ, khối khơng gian khơng cịn coi ý niệm toán học nấp đằng sau phạm trù siêu kim thuỷ hoả thổ Hệ thống suy luận logic xuất phát từ hệ tiên đề Plato Euclid sử dụng cách triệt để, tiêu tính chặt chẽ chứng minh áp dụng cách không khoan nhượng Theo nghĩa đó,Cơ sở hình học sách tuý toán học nhân loại tờ giấy khai sinh toán học mơn độc lập, cịn phận triết học Cách Euclid xây dựng hệ thống kiến thức cao vút dựa số tiên đề lấy luật logic làm chất gắn kết, hình mẫu cho phát triển tốn học ngày hôm

(156)

Sách Cơ sở hình học Euclid HTN: Cuốn “Cơ sở hình học” Euclid lần được dịch xuất bản: NXB Tri Thức ZenBook, Quý IV năm 2015 Dự án dịch Euclid Đàm Thanh Sơn và Nguyễn Trí Dũng khởi xướng tháng năm 2013.

Lớn lên từCơ sở hình học, Tốn học bước xa Bây bạn tìm vơ số sách tốn với nhiều nội dung hơn, trình bày sáng sủa sách Euclid Tuy vậy, tơi tin người có học cần đọc Euclid vào thời điểm đời mình, cần cóCơ sở hình họcđặt giá sách

Cảm ơn nhà xuất Tri Thức Nhóm dịch giả đem sáchCơ sở hình họccủa Euclid đến với độc giả Việt nam

Ngô Bảo Châu

B Lời giới thiệu Giáo sư Richard Fitzpatrick cho bản tiếng Anh[1]

(157)

Tạp chí Epsilon, Số 08, 04/2016

từ năm tiên đề đơn giản Euclid đánh giá cao đưa chứng minh cụ thể tài tình cho định lý khám phá trước đó: chẳng hạn Định lý 48 Quyển Các phép dựng hình khai triển sách giới hạn thao tác thực thước thẳng compa Hơn nữa, chứng minh thực nghiệm dựa vào đo đạc hồn tồn khơng sử dụng: tức so sánh hai đại lượng phép kết luận chúng đại lượng lớn đại lượng kia.Cơ sở hình họcgồm 13 Quyển nêu định lý hình học phẳng, bao gồm ba định lý tam giác đồng dạng, số định lý liên quan đến đường thẳng song song, định lý tổng góc tam giác, định lý Pythagoras

Quyển thường cho liên quan đến “hình học giải tích”, hầu hết định lý có hệ đại số đơn giản

Quyển nghiên cứu hình trịn tính chất chúng, bao gồm định lý đường tiếp tuyến góc nội tiếp

Quyển nói đa giác thông thường nội tiếp – ngoại tiếp – hình trịn Quyển xây dựng lý thuyết tỷ lệ thức số học

Quyển áp dụng lý thuyết tỷ lệ thức vào hình học phẳng, bao gồm định lý hình đồng dạng [2]

Quyển liên quan đến lý thuyết số bản: ví dụ số nguyên tố, mẫu số chung lớn v.v Quyển nói cấp số nhân

Quyển bao gồm ứng dụng hai trước, định lý vô hạn số nguyên tố

Quyển 10 phân lớp đại lượng vô ước (tứ vô tỉ) sử dụng phương pháp gọi “phương pháp vét kiệt”, tiền thân phép tích phân

Quyển 11 liên quan đến định lý hình học khơng gian

Quyển 12 tính tốn thể tích tương đối hình nón, hình chóp, hình trụ hình cầu sử dụng phương pháp vét kiệt

Cuối cùng, 13 nghiên cứu năm khối đa diện Platon

Ấn trình bày nội dung tiếng Hy Lạp tác phẩm – J.L.Heiberg biên tập (1883-1885) - với dịch sang tiếng Anh đại, mục lục từ vựng Hy Lạp – Anh Các 14 15 giả mạo, bình giải, vốn thêm vào nhiều kỷ qua khơng có ấn Mục đích dịch làm cho luận điểm sáng sủa mạch lạc hơn, bám sát ý gốc tiếng Hy Lạp Các nội dung đặt ngoặc vuông (cả phần tiếng Hy Lạp tiếng Anh) Heiberg, suy từ văn gốc (một số rõ ràng không hữu dụng bỏ đi) Nội dung để ngoặc đơn (phần tiếng Anh) nội dung ngụ ý không viết phần tiếng Hy Lạp Xin cám ơn Mariusz Wodzicki (Berkeley) cố vấn chữ, Sam Watson & Jonathan Fenno (U Mississippi) Gregory Wong (UCSD) nhiều sai sót thực Quyển

(158)

C TIỂU SỬ EUCLID[3]

Người ta nhiều đời Euclid.Từ điển Tiểu sử Khoa học (Dictionary of Scientific Biography)mở đầu viết dài Euclid lời này:“Mặc dù Euclid nhà toán học trứ danh thời đại, người mà tên tuổi đồng nghĩa với hình học cho đến tận kỷ 20, có hai kiện đời ơng biết đến, mà kiện này chưa phải khơng cịn tranh cãi” Những “sự kiện” suy diễn hay đồn đoán dựa vào việc tham khảo tác phẩm cổ đại Đầu tiên ông sống thời sau Plato (mất vào năm 347 TCN) trước Archimedes (sinh năm 287 TCN) Thứ đến ơng làm việc Alexandria Ơng khơng phải Euclid Megara, người bạn Plato, người bị nhầm với ông Heath[4] cho biết khả Euclid (tác giả củaCơ sở hình học) tiếp nhận giáo dục tốn Athen từ học trị Plato hầu hết nhà hình học dạy ơng xuất thân từ trường đó, nhà tốn học mà Cơ sở hình học Euclid dựa vào sống dạy Athen Nếu đồng ý với điều xác định Euclid sống sau năm 347 TCN

Quan điểm cho ông sinh trước Archimede dựa vào mục tham khảo dẫn đến tác phẩmVề hình cầu hình trụcủa Archimede Tuy nhiên, mục tham khảo bị cho chèn vào sau

Mặc dù vậy, số người giữ quan điểm chắn ông sống trước thời với Appollonius (là người sống vào khoảng năm 200 TCN) Một chút chứng cho chuyện chứng cho việc ông làm việc Alexandria tham khảo từ Pappus (khoảng năm 320 CN) Pappus nhận xét Apollonius Apollonius“sống lâu với học trò Euclid Alexandria, nhờ mà ơng có thói quen tư quy khoa học như vậy” Nếu tin Pappus chuyện phải đặt Euclid vào thời gian trước năm 200 TCN

Heath rút từ nhận xét Pappus Euclid dạy mở trường Alexandria Tuy nhiên, người khác phản bác cho dù rõ ràng ơng có học trị Alexandria điều khơng chứng minh ơng làm việc Dù nữa, Apollinius tham khảo Euclid lời giới thiệu Quyển I bộCác đường conic Bởi Apollonious sinh vào khoảng năm 262 TCN Euclid phải trước 200 TCN

Đấy tất biết đời Euclid Thế cịn tác phẩm? Heath trích Proclus (410-485 CN) sau:

“Euclid biên soạn Cơ sở hình học từ việc tập hợp định lý Euxodus, hoàn chỉnh nhiều định lý Theaeterus, chứng minh cách chối cãi định lý vốn chứng minh cách lỏng lẻo bậc tiền bối”

Từ điển Tiểu sử Khoa họcnói rằng:

(159)

Tạp chí Epsilon, Số 08, 04/2016

hình học viết trước Euclid – đáng ý tác phẩm Hippocrates, Leo and Theudius Magnesia – tác phẩm Euclid vượt qua chúng cách ngoạn mục đến nỗi chúng biết đến qua mục tham khảo Eudemus’s là Proclus dẫn lại”.

Nhiều tác phẩm khác cho Euclid, thiên văn học, quang học, lý thuyết âm nhạc, tốn học Nhiều tác phẩm số khơng cịn hữu nữa, cịn lại mảnh vỡ SauCơ sở hình học, cơng trình tốn học quan trọng ơng cịn giá trị làDữ liệu (Data), tác phẩm rõ ràng viết cơng cụ để giải tốn phép phân tích QuyểnVề phân chia (của số)– lưu truyền qua tiếng Ả rập – nói việc phân chia số thành số khác không đồng dạng Quyển Các hệ cho Euclid bị mất, biết đến thơng qua tác phẩm Pappus Nó bao gồm hệ Cơ sở hình học, mà theo Heath,“các định lý về lý thuyết đường cát tuyến hình học xạ ảnh đại” Euclid cho viết chuyên luận mặt cắt với hình nón, dày bốn quyển, tác phẩm vượt qua tác phẩmCác hình nóncủa Apollonius, vốn bị

Hai tác phẩm quang học cho Euclid,Quang học (Optics)Phản xạ học (Catropics), mở truyền thống dài mơn quang hình học, trì tận đầu kỷ mười bảy [ ]

Những biết Eucid biết qua tác phẩm sáng chói ơng để lại Quyển sách mà bạn cầm tay cho bạn hội nghiên cứu tác phẩm cách trực tiếp tự nghiền ngẫm xem sao“nhà tốn học khả kính thời đại”lại vinh danh suốt hai thiên niên kỷ

[1] Được đồng ý Giáo sưRichard Fitzpatrick, người thực tiếng Anh mà dùng để dịch, đăng lại nguyên văn lời giới thiệu ơng để độc giả có thêm thơng tin tác phẩm (ND)

[2] Do điều kiện xuất lần này, giới thiệu nội dung từ đến dịch tiếng Việt Chúng tơi hy vọng có điều kiện thực giới thiệu tiếp lại đến độc giả Việt Nam tương lai gần (ND)

(160)(161)

LÊ VĂN THIÊM: CON NGƯỜI VÀ SỰ NGHIỆP

Hà Huy Khoái

1 Sơ lược tiểu sử

Lê Văn Thiêm sinh ngày29tháng3năm1918tại làng Trung Lễ, Đức Thọ, Hà Tĩnh Trung Lễ làng cổ, thành lập cách khoảng600năm vùng đất trũng, quanh năm bị đe doạ nạn hạn hán, lụt lội Dân Trung Lễ nông, nghèo hiếu học Từ kỷ XV có ơng Trần Tước đỗ Tiến sĩ (Khoa Bính Thìn,1496) Họ Lê Trung Lễ tiếng truyền thống Nho học yêu nước Cụ thân sinh Lê Văn Thiêm ông Lê Văn Nhiễu.1869 1929/, nhiều nơi viết Nhiệu (theo cách phát âm người Hà Tĩnh), đậu cử nhân Khoa Canh Tý.1900/:Mẫu thân cụ Cử Lê Văn Nhiễu, tức bà nội Lê Văn Thiêm, bà Phan Thị Đại, chị ruột nhà yêu nước Phan Đình Phùng Chú ruột Lê Văn Thiêm ông Lê Văn Huân, đậu Giải nguyên Khoa Bính Ngọ.1906/;tham gia phong trào yêu nước Duy Tân hội, Tân Việt Đảng, tự sát nhà lao Vinh năm1929:

Cử nhân Lê Văn Nhiễu

(162)

của Lê Văn Thiêm, ơng Lê Văn Ln, Bí thư Huyện uỷ Đảng Cộng sản Đông Dương Huyện Đức Thọ, bị Pháp xử tử hình năm1931:Trong số5người chị gái Lê Văn Thiêm có hai người tham gia phong trào cách mạng1930 1931;và công nhận Lão thành cách mạng Lê Văn Thiêm út nhà, nên bé, đặt tên “Thêm”, tức đứa “Trời cho thêm” Khi đời, cậu bé Thêm yếu, bà mẹ sinh nở đến lần thứ13:Mẹ cậu khơng cịn sữa, nên cậu phải bú nhờ người chị dâu tên Sâm, vợ anh Lê Văn Luân Vì thế, cậu, bà Sâm gần người mẹ thứ hai Ông Luân, bà Sâm hoạt động cho Tân Việt Đảng Bà đóng vai người bán hàng tơ lụa, ơng đóng vai người chở thuê, hai người khắp nơi tuyên truyền cách mạng, in tài liệu, rải truyền đơn Khi nhỏ, cậu bé Thêm học quê nhà với ruột, Giải nguyên Lê Văn Huân Cậu tiếng học giỏi, tiếng “khờ” Lớn lên, Lê Văn Thiêm theo anh - ông Nghè Kỷ, học Huế, Quy Nhơn

Sinh gia đình giàu truyền thống yêu nước, anh niên Lê Văn Thiêm sớm ni hồi bão học tập để phụng Tổ quốc Năm1941;Lê Văn Thiêm thi đỗ vào trường

Ecole Normale Supérieureở Phố d’Ulm Paris (trong tiếng Việt, người ta thường dịch

(163)

Tạp chí Epsilon, Số 08, 04/2016

Một chỗ làm việc tuỵêt vời, hướng nghiên cứu thời sự, kết đầu tay trở thành tiếng, tất mở trước mắt nhà toán học trẻ Lê Văn Thiêm đường thênh thang để đến đỉnh cao khoa học

Nhưng mục đích đời ơng trước hết đóng góp sức cho đấu tranh giành tự Tổ quốc Vì thế, nghe theo lời kêu gọi Chủ tịch Hồ Chí Minh, cuối năm1949;ơng rời bỏ đường công danh Châu Âu để bí mật trở nước tham gia kháng chiến Từ Châu Âu, ơng Băng Cốc, từ qua Campuchia để Nam Bộ

Giáo sư Lê Văn Thiêm, Thuỵ Sĩ,1943:

Ở Nam Bộ, Giáo sư Lê Văn Thiêm gia nhập Đảng Cộng sản Đông Dương cơng tác Sở Giáo dục Ơng góp phần đào tạo nhiều giáo viên cho vùng kháng chiến Ít lâu sau, ông lên đường Việt Bắc nhận nhiệm vụ mới: Lãnh đạo trung tâm đại học nước Việt Nam dân chủ cộng hoà Đây thật nhiệm vụ quan trọng phù hợp với khả năng, ý nguyện ông Sau6tháng gian nan từ Nam Bộ lên chiến khu Việt Bắc, Giáo sư Lê Văn Thiêm giao trọng trách Hiệu trưởng Trường Sư phạm cao cấp Trường Khoa học Ơng làm cương vị đó, trở thành người đặt móng cho giáo dục đại học nước Việt Nam mới, người thầy hầu hết nhà khoa học Việt Nam đào tạo mươi, mười lăm năm sau cách mạng Tháng Tám

(164)

Giáo sư Lê Văn Thiêm (người đứng giữa) Nam Bộ,1949:

2 Những đóng góp khoa học

2.1 Các cơng trình lý thuyết Phân phối giá trị hàm phân hình

Lý thuyếtPhân phối giá trị hàm phân hìnhđược xem lý thuyết đẹp Giải tích tốn học kỷ XX Có thể xem lý thuyết mở rộng củađịnh lý bản của đại số Theo định lý đó, đa thức bậcntuỳ ý có đúngnnghiệm, kể bội Về mặt đó, hàm chỉnh hình mở rộng tự nhiên đa thức, hàm chỉnh hình tồn mặt phẳng (hàm ngun) biểu diễn chuỗi vô hạn hội tụ Tuy nhiên, khác với lý thuyết đa thức, lý thuyết hàm chỉnh hình khó khai thác khía cạnh “đại số”, mà chủ yếu dựa vào công cụ giải tích Vấn đề phân bố khơng điểm hàm chỉnh hình, vấn đề phân bố nghiệm đa thức, vấn đề trọng tâm Và vấn đề này, ta gặp phải khó khăn Do hàm chỉnh hình biểu diễn chuỗi vơ hạn, có vơ hạn khơng điểm tồn mặt phẳng, nên khơng thể có kết đơn giản định lý đại số Vấn đề đặt làm để xét phân bố khơng điểm hàm chỉnh hình tương tự làm đa thức

Từđịnh lý bảncủa đại số suy rằng, đa thức có cấp tăng cao có nhiều khơng điểm Mặc dù cấp tăng đặc trưng quan trọng hàm chỉnh hình, thấy rằng, mở rộng trực tiếp củađịnh lý bảncủa đại số khơng cịn cho trường hợp hàm chỉnh hình Thật vậy, tồn hàm chỉnh hình có cấp tăng lớn (như hàmez/;

nhưng khơng có khơng điểm Trong trường hợp hàm phân hình vấn đề trở nên phức tạp: Hàm phân hình hàm nhận giá trị vơ hạn số điểm hữu hạn, cần phải có quan niệm cấp tăng Lý thuyết phân phối giá trị Nevanlinna đời nhằm giải vấn đề Trước hết, Nevanlinna định nghĩa cáchàm đếmhàm xấp xỉ, mà ta mô tả cách sơ lược sau Giả sửf z/ hàm phân hình toàn mặt phẳng,alà giá trị phức tuỳ ý Khi đó,hàm đếm N.f; a; r/có mục đích “đo độ lớn tập hợp các điểm nằm vòng tròn bán kínhr;tâm gốc, mà hàm nhận giá trịa” Như vậy,

(165)

Tạp chí Epsilon, Số 08, 04/2016

trong vịng trịn bán kínhr;tâm gốc, mà hàm nhận giá trị “gần bằnga” Hàm đặc trưng Nevanlinna định nghĩa bởi:

T f; a; r/Dm.f; a; r/CN.f; a; r/:

Như vậy, nói cách nơm na, hàm T f; a; r/ dùng để tính số nghiệm phương trình

f z/Datrong vịng trịn bán kínhr (kể số điểm hàm nhận giá trị gần vớia/:Khi nghiên cứu hàm phân hình, hàm đặc trưngT f; a; r/đóng vai trò gần giống bậc nghiên cứu đa thức Điều thể rõ cácĐịnh lý bảncủa Nevanlinna:

Định lý thứ nhất.Tồn hàmT f; r/sao cho với giá trịa;ta có T f; a; r/DT f; r/C0.1/;

trong đó0.1/là đại lượng giới nội khir tiến vô cùng.

Từ định lý trên, xem hàmT f; a; r/khơng phụ thuộc giá trịa;nghĩa hàm phân hình nhận giá trịa(kể giá trị “gần” với nó) số lần Đây tương tự củađịnh lý bảncủa đại số cho trường hợp hàm nguyên hàm phân hình Tuy nhiên, để đạt tương tự đẹp đẽ nói trên, ngồi hàmN.f; a; r/ta phải bổ sung thêm hàm xấp xỉm.f; a; r/mà thực chất dùng để đo điểm hàm cho nhận giá trị “gần” vớia:

Nếu “hiệu chỉnh” mà lớn hiển nhiên, Định lý thứ Nevanlinna trở nên ý nghĩa

Nevanlinna chứng minhĐịnh lý thứ hai, sâu sắc nhiều so vớiĐịnh lý thứ nhất Nói nơm na,Định lý thứ haicho thấy “đại lượng hiệu chỉnh”m.f; a; r/nói chung nhỏ.Định lý thứ haicủa Nevanlinna phát biểu sau:

Định lý thứ hai.Với số nguyên dương q và số phức phân biệt tuỳ ýai; i D 1; 2; : : : ; q (có thể bằng1/;ta có

q

X

iD1

m.f; ai; r/62T f; r/Co.logT f; r//:

Doq số tuỳ ý, mà vế phải bất đẳng thức củaĐịnh lý thứ haikhơng phụ thuộcq

nên từ thấy rằng, đại lượngm.f; a; r/nói chung nhỏ Để “định lượng” tính chất đó, Nevanlinna đưa hàm khuyết sau đây:

ı.a/ D lim

r!1inf

m.f; a; r/ T f; r/ ; a/ D lim

r!1sup

N1.f; a; r/ T f; r/ :

trong đóN1.f; a; r/là đại lượng tính nhưN.f; a; r/nhưng nghiệm phương trình

f z/Dachỉ kể lần (khơng tính bội)

(166)

rõ ràng phụ thuộc vào bội nghiệm phương trìnhf z/Da:Với định nghĩa đó, từĐịnh lý thứ nhất, ta có:

06ı.a/C.a/ 61: 1/

TừĐịnh lý thứ haita thu bất đẳng thức sau đây:

X

a2C[1

fı.a/C.a/g 62: 2/

Bất đẳng thức.2/được gọi làquan hệ số khuyết.

Từ quan hệ số khuyết, ta suy rằng, với hầu hết giá trịa;đại lượng.a/bằng0;trừ số đếm giá trị củaa;đồng thời tổng giá trị bị chặn bởi2:

CácĐịnh lý bảnthứ thứ hai, vớiquan hệ số khuyếtlàm nên “ba đá tảng” lý thuyết Nevanlinna

Từ quan hệ số khuyết, cách tự nhiên phải đặt vấn đề sau đây, thường gọi làBài toán ngược lý thuyết Nevanlinna.

Cho dãy (hữu hạn vô hạn) điểmak mặt phẳng phứcC(kể điểm vô cùng), số không âm tương ứng.ak/; ı.ak/thoả mãn điều kiện sau:

0 < ı.ak/C.ak/61; k D1; 2; : : :

X

k

fı.ak/C.ak/g62:

Vấn đề đặt tìm hàm phân hình có số khuyết (tương ứng, số bội) điểmak làı.ak/

(tương ứng.ak//và số khuyết (tương ứng, số bội) bằng0tại điểm lại

Nevanlinna (năm1932/đã cho lời giải toán trừơng hợp riêng với giả thuyết chặt sau đây:

1 Dãyfakglà hữu hạn ı.ak/là số hữu tỷ .ak/D0với mọik:

Trong khoảng15năm kể từ kết Nevanlinna, tốn khơng tiến triển thêm bước Cho đến năm1949;Lê Văn Thiêm tiến bước dài việc giải toán ngược lý thuyết Nevanlinna Kết mà ơng thu xây dựng nghiệm toán ngược với giả thiết sau đây:

1 Dãyfakglà hữu hạn

2 ı.ak/và.ak/là số hữu tỷ

3 Nếu.ak/ > 0thìı.ak/C.ak/ < 1:

(167)

Tạp chí Epsilon, Số 08, 04/2016

Đóng góp quan trọng Lê Văn Thiêm không việc chứng minh tồn nghiệm tốn ngược tình tổng qt so với cơng trình Nevanlinna, mà điều quan trọng lần đầu tiên, ông đưa công cụánh xạ bảo giác không gian Teichmulervào việc giải tốn ngược Tư tưởng ông nhà toán học tiếng khác sử dụng để tiếp tục thu kết cho toán ngược: Goldberg, Weitsman, Drasin Cuối cùng, năm1977;Drasin cho lời giải trọn vẹn toán ngược lý thuyết Nevanlinna,

45năm sau toán đặt Điều đáng nói là, cơng trình mình, Drasin sử dụng phương pháp mà Lê Văn Thiêm lần áp dụng

Công trình tốn ngược lý thuyết Nevanlinna đặt Lê Văn Thiêm vào hàng ngũ tác gia kinh điển lý thuyết Ngay cơng trình đời, người giới thiệu tờ

American Mathematical Reviewschính Ahlfors, người nhận Giải thưởng Fields năm 1936:

Ahlfors giới thiệu số cơng trình Lê Văn Thiêm tạp chí Cho đến tận ngày hôm nay, sách Lý thuyết hàm phân hình, nói đến lý thuyết Nevanlinna nhắc đến cơng trình Lê Văn Thiêm Không phải nhà khoa học có vinh dự nhắc đến kết mình60năm sau! Có thể tin rằng, cơng trình Lê Văn Thiêm cịn nhớ đến nhiều năm, cột mốc lý thuyết hàm phân hình

Bài báo Beitrag zum Typenproblem der Riemannschen Flachen (Về vấn đề phân loại diện Riemann) Lê Văn Thiêm đăng tờCommentarii mathematici Helverticinăm1947chính cơng trình tốn học người Việt Nam cơng bố tạp chí quốc tế Có thể xem năm1947là năm mở đầu cho Lịch sử toán học Việt Nam đại, thật đáng tự hào Toán học Việt Nam tham gia với toán học giới “cơng trình đầu tay” có ý nghĩa lịch sử! Trở Việt Nam năm1949theo lời kêu gọi Chủ tịch Hồ Chí Minh, Giáo sư Lê Văn Thiêm tạm dừng nghiên cứu khoa học để chuyên tâm vào nhiệm vụ quan trọng Nhà nước giao phó Tuy vậy, có chủ trương thúc đẩy phong trào nghiên cứu khoa học trường đại hoc, Giáo sư lại trở với lý thut diện Riemann u thích Theo lời kể ơng, hai cơng trình đăng tạp chíSibirskii Matematicheski JournalActa Scientiarum Vietnamicarumvào năm1964; 1965là kết việc nghiên cứu vấn đề mà ông suy nghĩ từ cịn Pháp, chưa có dịp thực Trong cơng trình đó, Lê Văn Thiêm đưa điều kiện để mặt phủ diện Riemann thuộc kiểu hypecbôlic thông qua việc tồn “đầu mút mơđula” Ơng đưa điều kiện để diện Riemann thuộc lớpOHB;tức khơng tồn hàm điều hoà giới nội khác số

Từ sau hai cơng trình kể trên, Giáo sư Lê Văn Thiêm chuyển hẳn sang nghiên cứu vấn đề toán học ứng dụng, theo chủ trương đưa khoa học vào phục vụ thực tiễn sản xuất chiến đấu

2.2 Các cơng trình tốn học ứng dụng

Vốn chuyên gia tiếng lý thuyết hàm phân hình diện Riemann, vấn đề toán học lý thuyết, Giáo sư Lê Văn Thiêm chuyển sang nghiên cứu lãnh đạo nhóm nghiên cứu toán học ứng dụng Điều đáng ngạc nhiên số cơng trình ơng tốn ứng dụng, có cơng trình trở thành kinh điển lĩnh vực này: Lời giải tường minh toán thấm qua hai lớp đất

(168)

Trong nhiều toán thấm, chẳng hạn xét nước thấm qua đê dài, ta đến mơ hình tốn thấm phẳng (tức khơng phụ thuộc chiều đó) Với số giả thiết chấp nhận được, việc mơ hình hố tốn học đưa tốn thấm qua mơi trường đồng chất việc xây dựng hàm chỉnh hình thực ánh xạ bảo giác miền thấm lên nửa mặt phẳng Đó việc khó khăn mặt tốn học, miền thấm thường phức tạp Tuy vậy, trường hợp đó, ta phải xét mơ hình xa với thực tiễn: Mơi trường mà nứơc thấm qua “đồng chất”, tức có lớp đất với hệ số thấm Trong thực tiễn, thường có nhiều lớp với hệ số thấm khác nằm cơng trình thuỷ lợi: Lớp đất sét, lớp đất cát, Đối với trường hợp miền thấm khơng đồng chất, trước cơng trình Lê Văn Thiêm, người ta có phương pháp giải gần Trong cơng trìnhSur un problème d’infiltratione travers un sol deux couches (Về toán thấm qua hai lớp đất)đăng tạp chí Acta Sci.Vietnam

1; 1964;pp.3 9;Lê Văn Thiêm dùng Nguyên lý đối xứng giải tích phức để xây dựng nghiệm tường minh cho toán thấm qua hai lớp đất với hệ số thấm khác Đây công trình lĩnh vực lý thuyết nước thấm cho phép xây dựng nghiệm giải tích tốn thấm khơng đồng chất Điều khẳng định sáchLý thuyết chuyển động nước ngầmcủa Palubarinova-Kochina xuất Matxcơva năm1977:

(169)

Tạp chí Epsilon, Số 08, 04/2016

phố Hồ Chí Minh Ơng lập nên Phịng Tốn học ứng dụng, nghiên cứu vấn đề toán học đặt lý thuyết đàn hồi chuyển động chất lỏng nhớt

Các vấn đề toán học ứng dụng mà giáo sư Lê Văn Thiêm quan tâm nghiên cứu vấn đề đặt thực tiễn Việt Nam: xây dựng đê điều cơng trình thuỷ lợi, cải tạo ruộng nhiễm mặn vùng ven biển, tính tốn trữ lượng dầu khí, nạo vét lịng kênh để phục vụ giao thông thời chiến Ngay giải nhiệm vụ ứng dụng trước mắt, với trình độ cao khoa học bản, ơng có đóng góp quan trọng vào phát triển lý thuyết

3 Xây dựng Toán học Việt nam

Với cơng trình khoa học xuất sắc, Lê Văn Thiêm người viết trang lịch sử tốn học Việt Nam đại Ơng người đặt móng xây dựng tốn học Việt Nam Uy tín ơng nguyên nhân khiến nhiều niên tài lên Chiến khu Việt Bắc để nghiên cứu giảng dạy tốn học: Hồng Tuỵ, Nguyễn Cảnh Tồn, Và khơng lơi cuốn, khuyến khích họ tiếng tăm mình, Giáo sư Lê Văn Thiêm dồn tâm sức để đào tạo lớp niên đầy nhiệt huyết ngày đầu cách mạng “Vốn liếng” ơng thật ỏi, sách mà ông số giáo sư khác cố gắng mang theo suốt chặng đường từ châu Âu đến chiến khu Ơng ln khuyến khích tài trẻ sâu vào nghiên cứu khoa học, cố gắng tạo cho họ điều kiện tốt Ngay sau hồ bình lập lại, trường đại học Việt Nam chưa có giáo trình đại học tốn tiếng Việt Vậy mà tâm lớn Nhà nước Việt Nam giảng dạy tiếng Việt bậc đại học Lê Văn Thiêm dịch viết giáo trình, từ Hàm biến phức Xác suất thống kê Đến năm1964;chúng Thư viện cho mượn giáo trình ơng dịch, đánh máy tiếng Việt khơng dấu: Có lẽ thói quen Pháp, để tiết kiệm thời gian viết, tiếng Việt Giáo sư Lê Văn Thiêm thường khơng có dấu! Nhận thức rõ tầm quan trọng Toán học việc xây dựng khoa học nước nhà, Giáo sư Lê Văn Thiêm với Giáo sư Tạ Quang Bửu, Hoàng Tuỵ vạch chiến lược lâu dài phát triển Toán học Việt Nam Sự đời Phịng Nghiên cứu Tốn năm1962(trực thuộc Uỷ ban Khoa học Kỹ thuật Nhà nước) cột mốc quan trọng trình xây dựng toán học Việt Nam

Năm1969;Thủ tướng Phạm Văn Đồng ký định thành lập Viện Toán học thuộc Uỷ ban khoa học Kỹ thuật Nhà nước Năm1970;Giáo sư Lê Văn Thiêm, lúc Phó Hiệu trưởng Trường Đại học Tổng hợp Hà Nội, chuyển giữ chức vụ Phó Viện trưởng, Phụ trách Viện Tốn học Từ lúc đó, Viện Tốn học thức vào hoạt động

(170)

với ứng dụng thực tiễn, đặc biệt vấn đề đặt Miền Nam thuỷ lợi Đồng sơng Cửu Long, dầu khí

Giáo sư Lê Văn Thiêm, với Giáo sư Hoàng Tuỵ, người gây dựng Khoa Toán Trường Đại học tổng hợp Hà Nội Ơng ln kiên trì phương châm giữ vững chất lượng đào tạo, năm chiến tranh, nhà trường phải sơ tán vào vùng núi Việt Bắc Ông phải trải qua nhiều đấu tranh gay go nội Khoa Toán Trường Đại học Tổng hợp Hà Nội năm60để giữ vững chiến lược đắn Nhờ thế, Khoa Tốn Đại học Tổng hợp Hà Nội (nay Đại học Khoa học tự nhiên thuộc Đại học Quốc gia Hà Nội) đào tạo nên nhiều nhà toán học hàng đầu nước

Giáo sư Lê Văn Thiêm Chủ tịch Hội Toán học Việt Nam Với uy tín, tài đức độ mình, Giáo sư người lãnh đạo, đồng thời hạt nhân gắn kết cộng đồng toán học Việt Nam

Suốt đời hết lịng hệ trẻ, Giáo sư Lê Văn Thiêm người sáng lập tờ báo

Toán học Tuổi trẻ, trực tiếp viết cho báo từ số Ông trực tiếp đề thi chọn học sinh giỏi tồn Miền Bắc năm1963 1964:Ơng khơng nề hà việc gì, dù to dù nhỏ, miễn có lợi cho việc dìu dắt hệ trẻ Nhiều học sinh giỏi gặp khó khăn xét tuyển vào đại học gia đình, họ hàng bị số định kiến “lý lịch” tìm đến ơng, giúp đỡ tận tình Nhiều người số họ trở thành nhà tốn học giỏi, có nhiều đóng góp cho đất nước

Ngay nước chiến tranh, máy bay Mỹ bắn phá dội miền Bắc, Giáo sư Lê Văn Thiêm người đứng sáng lập tờ báo Toán học Vật lý tiếng nước Việt Nam: TờActa Scientiarum Vietnamicarum (Sectio Mathematicarum et Physicarum) Phần toán học tờ báo ngày trở thành tờActa Mathematica Vietnamica, tờ báo có uy tín tốn việt Nam, có mặt thư viện nhiều trường đại học lớn giới Việc cho đời tờ báo nghiên cứu toán học (bằng tiếng Anh, Pháp, Nga, Đức) chiến tranh điều có giới Nhiều nhà khoa học nước ngồi tỏ ý ngạc nhiên khâm phục thấy Việt Nam, đất nước phải đương đầu với chiến tranh tàn khốc hai miền, lại nghĩ đến việc tờ tạp chí nghiên cứu khoa học tiếng nước ngồi Việc làm chứng tỏ tầm nhìn xa nhà lãnh đạo khoa học Việt Nam, tin tưởng vào thắng lợi tất yếu nghiệp cách mạng

Sự phát triển Toán học Việt Nam, khoa học Việt Nam nói chung từ sau Cách mạng Tháng Tám mang đậm dấu ấn Giáo sư Lê Văn Thiêm

4 Thay lời kết luận

(171)

Tạp chí Epsilon, Số 08, 04/2016

Giáo sư Lê Văn Thiêm,1950:

(172)(173)

CÁC VẤN ĐỀ CỔ ĐIỂN VÀ HIỆN ĐẠI

Trần Nam Dũng - Đại học Khoa học Tự nhiên - ĐHQG TP.HCM

LỜI GIỚI THIỆU

Chuyên mục dành cho vấn đề cổ điển đại trình bày dạng tốn xâu chuỗi Đó chuỗi để giải toán đẳng chu, chứng minh đẳng thức Euler kỳ diệu1C

22 C

32 C D

6 ;một chuỗi toán

vận trù Cách trình bày xuất phát từ vấn đề đơn giản, dễ hiểu, khái niệm định nghĩa ln để đọc tương đối độc lập Và chuỗi nêu vấn đề định, giải toán kinh điển hay nêu giả thuyết mới, vấn đề Lời giải thảo luận toán đăng sốN C3:

(174)

Đề tốt nghiệp THPT mơn Tốn Pháp

Bài viết GS Nguyễn Tiến Dũng giới thiệu hai bạn Lê Phúc Lữ, Huỳnh Công Bằng dịch

Một số thông tin kỳ thi: Thời gian làm là5giờ

2 Thí sinh sử dụng máy tính bỏ túi

3 Thí sinh cần trình bày rõ ràng, mạch lạc để copy làm dễ đọc Đề thi có tổng cộng3bài thí sinh làm theo thứ tự tùy ý

1 Tổng số lập phương

Với số nguyên dươngn;ta gọin3là lập phương củan:Trong toán này, ta quy ước rằng:

S tập hợp số nguyên dương biểu diễn dạng tổng số lập phương phân biệt

S0là tập hợp số nguyên dương biểu diễn dạng tổng số lập phương chẵn phân biệt

S1là tập hợp số nguyên dương biểu diễn dạng tổng số lập phương lẻ phân biệt

Ví dụ:

8; 1902S vì8D23 và190D13C43C53I

216; 10722S0vì216D63và1072D23C43C103I 125; 25682S1vì125D53và2568D13C33C73C133:

Bằng cách trả lời câu hỏi sau đây, chứng minh số nguyên dương đủ lớn thuộcS;tức tồn tạiN cho với số nguyên dươngnN;ta cón2 S:

1) Chứng minh rằng2016thuộcS0:

2) Chứng minh rằng:

a) Với số thựcx5thì

(175)

Tạp chí Epsilon, Số 08, 04/2016

b) Với số ngun dươngk>5khi với số ngunp kta ln có

.2pC1/3.2k 1/3C p

X

jDk

.2j 1/3:

3) Chứng minh tồn tại288sốs1; s2; : : : ; s288thuộcS1sao chosi i mod 288/với mọii D1; 2; 3; : : : ; 288:

Tiếp theo, ta cố định số.s1; s2; : : : ; s288/đã nêu đặtmlà số lớn chúng 4) Gọi n số nguyên dương thỏa mãn 288n m xét cấp số cộng nguyên dương

u1; u2; : : : ; uncó cơng sai là288:Chứng minh số nguyên thuộc đoạnŒmCu1; 288nCu1

đều biểu diễn dạngsiCuj, đó1i 288và1j n:

5) Cho biết với số thựcx, ta có đồng thức sau

.2x C12/3C.2xC4/3C.2xC2/3 2x C10/3 2xC8/3 2x/3D288:

Hãy chứng minh rằng:

a) Tồn số nguyên dươngusao chou; uC288vàuC576đều thuộcS0:

b) Với số nguyênn2, tồn tạinphần tử trongS0tạo thành cấp số cộng có cơng sai là288:

6) Choklà số nguyên dương không nhỏ hơn5thỏa mãn.2kC1/3> m:

a) Chứng minh tồn số nguyên dươngN 1sao cho số nguyên dương thuộc đoạnN; N C2.2k 1/3

đều biểu diễn dang si Cu với

1i 288vàu 2S0:

b) Chứng minh số nguyên dương lớn bằngN thuộcS:

Gợi ý.Với số nguyên dươngp k, ta xem xét trường hợp số nguyên dương nằm đoạnN; Np, đó

Np DN C.2k 1/3C p

X

jDk

.2j 1/3:

2 Thám hiểm rừng rậm

Một nhà thám hiểm khám phá khu rừng phát tất thân có bán kính Đây thật điều thú vị! Tạm thời bỏ qua chiều cao cây, ta đặt khu rừng vào mặt phẳng tọa độ vuông góc sau:

Nhà thám hiểm đứng gốc tọa độ muốn tìm góc nhìn xuyên qua khu rừng

(176)

Ta nói nhà thám hiểm có thể“nhìn xun qua”khu rừng có tia xuất phát từ vị trí đứng (tại gốc tọa độ) qua khu rừng mà không cắt gốc (ở hình trịn)

Ở ví dụ hình, tiaDbị vướng phải gốc khiD0thì khơng Và có tiaD0

như nên ta nói nhà thám hiểm nhìn xun qua khu rừng

Vớim2.0I C1/, ta gọiDmlà tia gốcO có phương trìnhy Dmxvàx > 0:

Tiếp theo, ta thừa nhận tính chất sau: Với số vơ tỷ dươngmvà số" > 0nhỏ tùy ý, tồn hai số nguyên dươnga; blẻ cho

jb maj< ":

Hãy trả lời câu hỏi sau:

1) Choa; b; mlà số thực dương Chứng minh rằngDmcắt đường trịn có bán kínhR >

có tâm đặt tại.a; b/khi

jb maj Rp1Cm2:

2) Từ suy nếumlà số vơ tỷ thìDmsẽ vướng phải gốc

3) Giả sửmD b

a vớia; blà số nguyên dương nguyên tố nhau:

(177)

Tạp chí Epsilon, Số 08, 04/2016

b) Giả sửa; bkhác tính chẵn lẻ vàDmbị vướng vào Chứng minh 1Rpa2Cb2:

4) Từ chứng minh tất đường thẳngDmvớim > 0đều vướng phải gốc

nào thìR p1 5:

5) Giả sử rằngR p1

5 Chứng minh tiaDmvớim > 0sẽ vướng phải gốc

tại tọa độ.˛; 1/hoặc.1; ˛/với˛ số nguyên dương lẻ

Ta gọi hàng tất có tâm nằm vị trí.1; ˛/hoặc.˛; 1/với˛là số lẻ 6) Chứng minh người quan sát nhìn xun qua hàng

nhìn xuyên qua khu rừng

3 Hành trình miền số phức

Trong tốn này, ta quy ước ký hiệuj De2i và xác suất biến cốAlàP A/:

1) a) Chứng tỏ rằngj3 D1và1Cj Cj2 D0:

b) Trong mặt phẳng phức, ba điểm biểu diễn1; j; j2tạo thành tam giác có đặc điểm gì? c) Chứng minh nếua; b; c số thực thỏa mãnaCbjCcj2 D0thìaDbDc:

Cho xúc sắc đồng chất có mặt (được đánh số từ đến 6) Ký hiệuF biến ngẫu nhiên số xuất tung xúc sắc vàZ biến ngẫu nhiênjF:

2) Chứng minh ta ln cóZ 2˚

1; j; j2

P Z D1/DP ZDj /DP Z Dj2/D 3:

Xét số nguyênn1và tung xúc sắcnlần độc lập với Ký hiệuFk kết lần tung thứkvà đặtZk DjFk ĐặtSn DZ1CZ2C CZnvàpnDP SnD0/.

GọiUn; Vn; Wnlần lượt biến ngẫu nhiên số sốk2Œ1Inmà

Zk D1; Zk Dj; Zk Dj2:

3) a) TínhUnCVnCWn b) Chứng minh

SnDUnCj VnCj2Wn:

c) Chứng minh nếuSnD0thìUn DVnDWn

(178)

4) Giả sử rằngncó dạng3mvớimlà số nguyên dương

a) Chứng minh rằngUnlà phân phối nhị thức Xác định tham số tương ứng phân phối

b) Từ suy

P UnDm/D

3m m

22m

33m:

Ký hiệu PUnDm.VnDm/ xác suất có điều kiện biến cốVn D m cho biết trước

Un Dm:

c) Chứng minh

PUnDm.Vn Dm/D2 2m

2m m

:

d) Từ suy

p3m D3 3m

3m m 2m m :

e) Chứng minh

p3mC3 p3m D

.3mC2/.3mC1/ 9.mC1/2 :

5) Với số nguyên dươngm1, chứng minh

m mC1

p3mC3

p3m vàp3m 9m:

ĐặtXnlà biến ngẫu nhiên số số nguyênk2Œ1; nsao choSk D0:

6) a) Xác định biến ngẫu nhiên BernoulliYk với1knthỏa mãn

XnDY1CY2C CYn:

b) Ký hiệu E.Xn/; E.Y1/; E.Y2/; : : : ; E.Yn/ giá trị kỳ vọng biến Xn; Y1; Y2; : : : ; Yntheo thứ tự Cho biết rằngE.Xn/D E.Y1/CE.Y2/C CE.Yn/;hãy chứng minh

E.Xn/Dp1Cp2C Cpn:

c) Từ suy

lim

n!C1E.Xn/D C1:

Đặtqn DP Xn> 0/, câu hỏi yêu cầu chứng minh rằngqnhội tụ về1:

7) a) Chứng minh dãy số.qn/hội tụ số thựcqvớiqn q 1với mọin:

b) Vớir; nlà số tự nhiên, chứng minh

P Xnr/qr:

c) Từ suy với số nguyênn1, ta có

E.Xn/qCq2C Cqn:

(179)

Tạp chí Epsilon, Số 08, 04/2016

Chia đoạn thẳng

Bài toán đề xuất A.K.Tolpygo, K.K.Kokhas A.Mogileva cho Hội nghị mùa hè, thi thành phố năm2014:

Ở số xin giới thiệu lời giải cho toán đăng số4của tạp chí, để tiện cho việc theo dõi độc giả xin đăng lại phần đề

1 Dẫn nhập

Chọn số˛;trên đoạnŒ0; 1ta đánh dấu điểmf˛g; f2˛g; f3˛g; : : : ; f.n 1/˛g:

Trong suốt này, khơng nói điều ngược lại, ta giả sử rằng˛vô tỷ Nếu như˛ D p q;

ta giả sử rằngp < q:Như vậy, cho dù˛bằng bao nhiêu, khơng có hai điểm trùng Như thế, đọanŒ0; 1sẽ chia thànhnphần Tiếp theo ta giả sửn > 10và0; <f˛g< 0; 7:

Các hạn chế thực không quan trọng, ta đưa điều kiện để loại bỏ hiệu ứng hiển nhiên cho số nhỏ Nhưng từ điều kiện suy phần nhỏ hơn˛:

Ta ý thay˛bằngnC˛ hayn ˛thì phần Vì tốn, nói tính nhất, ta có thêm điều kiện0 < ˛ <

2:

Nội dung toán – nghiên cứu xem ta có phần chúng xếp

Các toán

Tỷ lệ đoạn dài đoạn ngắn ta ký hiệu làLDL.˛; n/:

A1.Giả sử˛ D p

q là số hữu tỷ Chứng minh tồn tạinsao choL.n/ D1:

A2.Với số nguyên hay hữu tỷknào,k > 1ta khẳng định với số hữu tỷ˛; tồn tạinsao choL.n/ Dk‹

Lời giải. LD2vàLD3:Các số khác khơng thỏa mãn dễ dàng đưa ví dụ: Chẳng hạn lấy˛ D

12 và˛ D

13;thì chúng khơng có giá trịLnào giống ngồi hai

giá trị nói

Tiếp theo ta khơng giả sử˛ hữu tỷ

(180)

Lời giải. Xét đoạn thẳng mà ta thu bước thứn:Giả sử đầu mút điểm

fk˛g;fl˛g:Hiển nhiên đoạn thẳng có độ dài với độ dài đoạn giới hạn điểmf.k 1/˛g;f.l 1/˛g;f.k 2/˛gvàfl 2/˛g; : : :Vì chuỗi hữu hạn nên có phần tử đầu tiên, chẳng hạn làf.k s/˛gvàf.l s/˛g:Hiển nhiên có3nguyên nhân khơng có đoạn thẳng trước đó:

hoặck Ds; hoặcl Ds;

hoặc, cuối cùng, đoạn thẳngf.k s 1/˛g;f.l s 1/˛gtồn bị chia thành2phần Trong trường hợp cuối bị chia điểmn˛;nếu không đoạn thẳng với đầu mút làf.k s/˛gvàf.l s/˛gcũng bị chia

Hai chuỗi đầu ln tồn tại, cịn chuỗi thứ ba khơng tồn Từ mà ta có hoặc2;hoặc3

chuỗi mà chuỗi độ dài đoạn thẳng yêu cầu

Từ suy đoạn thẳng chuỗi thứ ba (nếu chúng tồn tại) có độ dài tổng độ dài hai đoạn thẳng hai chuỗi lại Như vậy, độ dài đoạn thẳng

a < b < aCb/trong độ dài thứ ba khơng có

Từ suy điểm ln chia đoạn thẳng dài có độ dàiaCbra2phần

a; b(trong trường hợp ngược lại ta có4độ dài khác nhau, điều khơng thể)

Bây ta nghiên cứu câu hỏi sốLsẽ thay đổi chuyển từnlênnC1:Từ điều nói rõ ràng số n số bậc3nếu đoạn thẳng dạng.aCb/;

và vào thời điểm đoạn thẳng cuối biến mất, lạiavàb;tức số trở thành bậc3:Khẳng định:

(a) NếunvànC1là số bậc3thìLkhơng thay đổi

(b) Khi chuyển từ số bậc3sang số bậc2 n– bậc3; nC1/– bậc2/Lchuyển thànhL 1:

(c) Khơng có chuyển đổi từ số bậc2sang số bậc2:

(d) Khi chuyển từ số bậc2sang số bậc3có hai trường hợp Giả sửa < b độ dài đoạn ngắn đoạn dài củan:Khi đó:

(d1) Nếua < b

2 thìLkhơng thay đổi

(d2) Nếua > b

2 thìLđổi thành L L 1:

Thật vậy:

(a) Trong trường hợp thứ nhất, đoạn thẳng lúc đầu sau thế, có độ dài là:

(181)

Tạp chí Epsilon, Số 08, 04/2016

(b) Trong trường hợp thứ hai đoạn thẳng có độ dàiaCbbiến mất, cịn lạia < b trước đóLD aCb

a bây giờLD b a:

(c) Trong trường hợp thứ3cần nhắc lại giả thiết > 10và0; <f˛g< 0;

(nếu khơng có điều khẳng định không đúng)

(d) Cả hai trường hợp.d1/; d2/đều xét giống như.a/và.b/:

Với số˛đã cho ta nói sốnlà bậc hai có2độ dài khác bậc ba có3

độ dài khác

B2.Cho˛ là số vô tỷ, chứng minh tồn vơ sốnbậc hai vơ sốnbậc ba.

B3.Cho số vô tỷ˛;nchạy qua giá trịnD1; 2; : : : ; m:Chứng minh rằngm! 1;thì tỷ lệ số bậc hai chúng dần đến0:

Gọi.m/là số số bậc2trong sốnD1; 2; : : : ; m:Hãy đánh giá tốc độ dần đến0của tỷ số m/

m khim! 1:

Lời giải. Từ lý luận bàiB1ta thấy tiếp sau số bậc2;số bậc3sẽ số đoạn thẳng dài Vì số đoạn thẳng tăng vô hạn, hiển nhiên số đoạn thẳng có độ dài lớn dần đến vơ cùng, điều ta cần

B4.Số˛ phải để tỷ lệ tiến đến0chậm có thể? Nhanh ? (Chỉ cần đưa số ví dụ, phải thuyết phục).

B5.Hãy đánh giá chặn chặn số số bậc hai trong1triệu số (càng đúng càng tốt, đánh giá “lớn hơn3” không chấp nhận)

Lời giải. Đánh giá chặn trên: Không quá500:000- đánh giá làm chặt Để thu số lượng số bậc2như vậy, ta cần chọn˛ D

2 với <

106:Trong trường hợp

nãy dễ thấy nửa số bậc2:

Từ suy tỷ lệ số bậc2có thể tiến đến0một cách chậm Cụ thể hơn, có nhiều số bậc2ở1triệu số sau chúng xuất Nhưng ta lấy là˛ D

2

106 với <

1018 chúng lâu với tần xuất “một phần

106”, tức đến lúc tỷ lệ ổn định Rõ ràng cấu trúc

tiếp tục, lấy ví dụ˛ D

1 106

1

10106 : : :Ở bước chuyển tỷ lệ số bậc hai,

thật là, giảm cách đáng kể sau RẤT, RẤT lâu trở thành khơng đổi Và điều có nghĩa dãy dẫn đến0rất chậm

(182)

C1.Tồn hay không số˛;sao choL > 10với n, bắt đầu từnD10 ‹

Lời giải. Khơng tồn tại, từ cơng thức biến đổi sốLta thấy ln nhận giá trị nhỏ hơn2:

C2.VớinD2000000:Có thể xảy trường hợp đoạn

a; aC

nào có hơn1100000 điểm ? (Nhắc lại theo giả thiết, ta có0; < ˛ < 0; 7/:

Lời giải. Có, ví dụ: Giả sử˛ D

3Ctrong đórất nhỏ Khi tất cả2triệu điểm

nằm gần3điểm gồm điểm0;điểm

3 điểm

3:Tương ứng đoạnŒ0:3; 0:8sẽ chứa khoảng

3 tất điểm lớn hơn10

rất nhiều

C3.Tồn hay không˛;sao choLnhận: Vô hạn.

Hữu hạn giá trị khác khinchạy qua giá trị từ10đến1:Nếu tồn tại, hãy đưa ví dụ.

Lời giải. Có, tồn loại loại Ví dụ, nếu˛ D p2thì Lnhận chỉ3giá trị khác (ví dụ xét chi tiết bên dưới) Mặt khác, nếuLnhận giá trị siêu việt (ví dụLD) thìLcó thể biến đổi thànhL 1;khi thành L

L ta thấy

nhận số khác, tức chúng có vô hạn giá trị

C4.Hãy đưa điều kiện đủ đểLnhận hữu hạn (vơ hạn) giá trị (nếu có thể, hãy tìm điều kiện cần đủ, giới hạn điều kiện đó).

Lời giải. Nếu nhưLchỉ nhận hữu hạn giá trị đến lúc giá trị củaLsẽ vào vòng lặp: Các giá trị xuất trước lặp lại

Nhưng sau số lần biến đổi (mà biết có dạng L ! L

L! L

L ta lại thu giá trịLđã gặp trước dễ thấyLphải thỏa mãn phương

trình bậc2với hệ số nguyên

Như điều kiện cần tương ứng, nếuLkhông phải nghiệm phương trình bậc hai (ví dụ L siêu việt) điều đủ đểLnhận vơ số giá trị

Điều kiện đủ đểLnhận hữu hạn giá trị, ví dụ điều kiện:L2–.nC2/LCnD0:

Ví dụ sốp3và số mà xét toán thỏa mãn điều kiện

(183)

Tạp chí Epsilon, Số 08, 04/2016

Lời giải. Lsẽ liên tiếp nhận giá trị ta gặp số bậc3;nói cách khác số lần số đoạn thẳng dài xuất nước Nhưng số đoạn thẳng dần đến vơ (như nói lời giải bàiB3)

Hiển nhiên số đoạn thẳng dần đến vơ số “đoạn thẳng dài” dần đến vơ

D2.Một cách logic, xảy ba trường hợp:

.1/ Dù˛ bao nhiêu, tồn giá trị choLnhận vô số lần

.2/ Dù˛ bao nhiêu,Lnhận giá trị hữu hạn lần

.3/ Với số˛ điều đúng, với những˛khác điều

Điều ? Nếu điều thứ.3/ với˛ điều 1/đúng, với ˛ điều 2/

đúng?

Lời giải. Xảy điều thứ ba

D3.Giả sử với˛ đóLnhận giá trịAvàB lần (vớin > 10/:Liệu hai khẳng định sau có tương đương:

.a) Lnhận giá trịAhữu hạn lần

.b/ Lnhận giá trịB hữu hạn lần ?

Lời giải. Không, không Ví dụ ban đầuLnhận giá trị 100C

p

99Cp2 :Vì số nhỏ

hơn2nên giá trị củaLphải tính theo cơng thức L

L 1;và có nghĩa giá trị tiếp

theo củaLlà100Cp2:Tiếp theoLsẽ nhận giá trị99Cp2; 98Cp2; : : : ;p2:

Sau đóLchỉ nhận giá trịp2;p2C1;p2C2(và khơng cịn giá trị khác), giá trị nhận vô hạn lần

2 Các toán liên quan

Phần này, ta chủ yếu xét toán liên quan đến giá trị cụ thể của˛: Ký hiệu tỷ số vàng: D 1C

p

2 1; 618 : : :

E1.Ta đưa ra3phương pháp viết dãy ký tựAABAABABAAB : : :Hãy chứng minh ba phương pháp cho kết giống (một cách xác mệnh đề phát biểu bên đây)

(184)

.1/ Trên lưới vng kẻ tia, nút lưới tạo góc với đường nằm ngang lưới

Tại điểm nút ta viết ký tựA;sau viết ký tự vào điểm giao tia với đường thẳng lưới: Ký tựAđánh dấu giao điểm với đường thẳng đứng, ký tựB

đánh dấu giao điểm với đường nằm ngang (xem hình)

.2/ Đầu tiên ta viết ký tựA;sau thực số bước, bước, ký tựAđược thay

AAB;còn ký tựB thay bởiAB:Ví dụ sau3bước ta thu

ı Đầu tiên làAAB; ı sau làAABAABAB;

ı bước thứ baAABAABABAABAABABAABAB:

Chứng minh dãy số thu cách nói đoạn đầu dãy số định nghĩa ở.1/:

.3/ ĐoạnŒ0; 1được chia phương pháp mô tả phần dẫn nhận với, đó˛ D:

Dãy số xác định sau: Với mọinlà số bậc2;ta viết theo thứ tự độ dài đoạn thẳng cuối (tức từ1/;đoạn dài ký hiệu làA;đoạn ngắn làB:

Chứng minh số dãy số hữu hạn, có vơ số dãy số đoạn đầu dãy số ở.1/:

Các dãy số vớin1 D3 < n2 D8 < dãy số ?

Lời giải. Ta đưa định nghĩa1và3về2:

Định nghĩa 1.Ta nâng điểmAđang nằm đường thẳng đứng thứk lên thêmk đơn vị Hiển nhiên tất điểm trơi vào đường thẳng tạo với trục hồnh góc arctg (chứ khơng phải arcctg) Mặt khác, sau ta đặt tất điểmB;như u cầu, hai chữAsẽ có điểmB:Ta thực phép biến đổi thứ hai: Tất chữAthay bằngB;tất chữBthay bằngA:

(185)

Tạp chí Epsilon, Số 08, 04/2016

hiện phép đổi hình hai lần thìAsẽ thay bằngAAB vàB thay bằngAB mà dãy lần lại không đổi

Ghi chú.Về chất chứng tỏ biến đổiA!AAB; B !AB“có thể biểu diễn dưới dạng tích”: Đầu tiên ta thực biến đổiA!BA; B !Avà sau đóA!AB; B !A

ta thu biến đổi cho

Định nghĩa 3.Ta lý luận tương tự Giả sửN số bậc2:Điều có nghĩa cór đoạn thẳng dài độ dàiavàsđoạn thẳng ngắn độ dài a

; r Cs DN:Khi số bậc2tiếp theoM DN Cr

và di chuyển số này, ta chia đoạn thẳng dài dài đoạn ngắn “ngắn nhất” có độ dài thêm lần nhỏ

Khi vòng lặp kết thúc, ta bắt buộc phải đổi đoạn thẳng ngắn cũ thành đoạn thẳng dài Trong đoạn thẳng “dài cũ” tác thành “dài – ngắn”, “ngắn - dài” Đây biến đổi mô tả phần nhận xét lý luận phần trước Từ ta thấy rõ ràng số bậc2nào được: Ta phải lấy nhảy cách1để thu “bình phương” phép biến đổi

E2.Dãy số xác định theo quy tắc Giả sử ta lấy hai khúc, khúc cónký tự: Từ ký tự thứkC1đến ký tự thứkCnvà từ ký tự thứmC1đến ký tự thứmCn:Chứng minh số ký tựAở khúc gần nhau, cụ thể là: Số ký tựAở khúc cách không quá1:

Lời giải. Ta sử dụng định nghĩa1:Giả sử khẳng định khơng Khi tia tồn hai đoạn thẳng có số chữ nhau, đoạn có số chữ cáiBnhiều hay bằng2

so với đoạn (và số chữ cáiAtương ứng hơn)

Nhưng đoạn có nhiều hơn2chữ cáiB hình chiếu lên trục tung lớn Tương ứng, có hai chữ cáiAthì hình chiếu lên trục hồng nhỏ Mâu thuẫn hiển nhiên

E3.Hãy tìm thêm dãy mà xây dựng ba phương pháp tương tự (hay hai)

Lời giải. Điểm mấu chốt lời giải toán1là kiện “nâng” giao điểm tia với đường thẳng đứng lên số đơn vị tương ứng ta thu tia đối xứng với tia ban đầu Nhưng ta nâng điểm nằm đường thẳng đứng thứkkhông phải lênkđơn vị, mà2kđơn vị dễ thấy tia nhận đối xứng với tia ban đầu nếu˛ thỏa mãn phương trình

˛2C2˛ 1D0:

Tất nhiên có nhiều cách xây dựng khác

Trong chuỗi toán sau ta lấy˛ D:

P1.Tìm tất giá trị củaL(với cácnkhác nhau)

(186)

Lời giải. NếunD2;thì ta có hai đoạn thẳng tỷ số hai đoạn thẳng tỷ lệ vàng Tiếp theo, theo công thức đưa đầu (trong lời giải bàiB1),Lchỉ thay L

L DL

DLC1:

Giá trị là.LC1/ 1DL; : : :Từ ta thấy rằngLchỉ nhận2giá trị, cịn có thêmnlà số bậc2thìLnhận giá trị nhấtLD:

P3.Tìm tất số bậc2:

P4.Tìm số số bậc2trong1triệu số tiên nếu˛ D:Chỉ cần đáp số xác đến10%:

Lời giải. Các số bậc hai đối với˛ cho số Fibonacci, tức số2; 3; 5; 8; 13; : : :

(mỗi số tổng hai số trước đó)

Số Fibonacci thứk xấp xỉ k;và số lượng chúng trong1triệu số gần log106theo số;tức khoảng28:

Tất nhiên ta viết rõ ra30số Fibonnaci Số thứ28bằng832040;còn số thứ29

đã lớn hơn1triệu

Trong chuỗi toán sau ta lấy˛ Dp21; 4142:::

T1.Tìm tất giá trị củaL(với cácnkhác nhau)

T2.Lcó thể nhận giá trị đềunlà số bậc2 ‹

Lời giải. Bài toán tương tự với tốn trước Vì giá trị ban đầu củaL(khin D2/bằng

2 p2 p

2 D p

2;và giá trị tiếp theoLsẽ hoặcp2;hoặcp2C1hoặcp2C2(với sốnbậc2thì có hai giá trị đầu)

T3.Tìm tất giá trị củaLcho số khác (ngồi˛ D và˛ Dp2/:

K1.Ta khơng biết số˛:Nhưng ta biết dùnbằng bao nhiêu, sốLchỉ nhận hai giá trị Với những˛nào điều có thể?

Lời giải. Từ lý luận trước ta thấy rằngLcần thỏa mãn điều kiện L

L 1–1DL;từ Llà phép chia vàng Từ dễ dàng tìm được˛:

K2.Bài tốn để nghiên cứu Ta biết số˛ ta biết vớinlớn tùy ý, số điểm đoạn có độ dài

2 khác với n

2 khơng lớn hơn10:Ta nói số˛ ‹Cụ thể: Hãy nêu ví dụ vài số

Hãy đưa dấu hiệu đủ để điều khơng xảy ra: Nếu˛có tính chất tính chất khẳng định tốn khơng

Hãy đưa dấu hiệu để khẳng định số điểm đoạn độ dài

2 nằm

trong phạm vi từađếnbvới sốa; bnào đó.a <

2 < bvà ta muốn số

gần

(187)

Tạp chí Epsilon, Số 08, 04/2016

K3.Cho số hữu tỷ˛ D 113

248 vànnhận giá trịn D1; 2; 3; : : : ; 246:Trong số có

bao nhiều số bậc2;bao nhiêu số bậc3 ‹

Lời giải. Khai triển số cho thành liên phân số (với số 113

248 khai triển có dạng

2C

5C 7C

3 :

Khi số bậc2sẽ xuất cáchk;trong đóklà mẫu số phân số thành phần liên phân số, tức phân số

1; 2;

1 2C15

D 11;

1 2C 5C11

7 D 36

79; : : :

Như vậy, số bậc hai cách1(tức tất cả, vớinD2vàn D3/;sau dãy số bậc2cách2;sau dãy cách11và cách79:Trong số số bậc2trong dãy mẫu số tương ứng phân số ban đầu, tức dãy đầu có2số, sau là5số và7số

Nếu số˛ vơ tỷ điều tiếp tục vơ cùng, số hữu tỷ cuối có quy luật khác: Tất số, bắt đầu từ169sẽ có bậc2:

Đáp số:

Số2:

Số3; 5; 7; 9; 11:

Số13; 24; 35; : : :cho đến90:

Cuối cùng, tất số từ169đến248:

K4.Hãy nêu phương pháp cho phép với số hữu tỷ˛cho trước

˛ D p

q; p < q < 1000000;

tìm số số bậc2và số số bậc3;khinchạy qua giá trị từ1đếnq 1;trong thời gian cho phép (bằng tay)

K5.Cho sốLDL.100; ˛/:

Làm xác định với giá trị củanvà˛cho trước tồn hữu hạn hay vô hạn giá trị khác củaL ‹

(188)

.1/ NếuLlà nghiệm phương trình bậc2với hệ số nguyên

L2CnLCm D0:

.2/ NếuLlà nghiệm phương trình bậc3với hệ số nguyên

L3CnL2CmLCqD0:

K6.Biết số˛được viết thành liên phân số với mẫu số là3; 5; 12tức

˛ D

1C

3C

5C 12 C

:

(189)

VỀ KỲ THI VIỆT NAM TST 2016 VÀ DANH SÁCH ĐỘI TUYỂN VIỆT NAM

Ban Biên tập Epsilon

Kỳ thi chọn đội tuyển Việt Nam tham dự Olympic Toán quốc tế (viết tắt IMO) 2016 diễn ngày 24, 25 tháng Đề thi gồm toán, ngày làm 30 phút Các toán thuộc phân mơn Đại số, Hình học, Số học Tổ hợp với mức độ khó dễ khác Cụ thể phân bố toán sau

Bài 1: Số học

Bài 2: Nằm tổ hợp đại số

Bài 3: Hình học

Bài 4: Hình học

Bài 5: Tổ hợp Đại số

Bài 6: Đại số

Năm nay, có tổng số 50 thí sinh tham gia kỳ thi này, gồm Vũ Xuân Trung, học sinh lớp 12 trường THPT chuyên Thái Bình, tỉnh Thái Bình, thí sinh HCV IMO 2015 49 học sinh từ 27,5 điểm trở lên kỳ thi HSG quốc gia diễn vào đầu năm Dưới đề thi đầy đủ

1 Đề thi

Bài 1.1 Tìm a; nnguyên dương với a > 2để ước nguyên tố củaan 1cũng ước nguyên tố củaa32016 1:

Bài 1.2 GọiAlà tập hợp2000số nguyên phân biệt vàB là tập hợp2016số nguyên phân biệt K là số cặp.m; n/có thứ tự vớimthuộcAnthuộcB

jm nj 1000: Tìm giá trị lớn củaK.

(190)

1 Chứng minh rằngAH đi qua điểm cố định.

2 Giả sửBE; CF giao.O/lần lượt tạiK; L Đường thẳngAH cắtKLtạiP Giả sửQ một điểm trênEF sao choQP DQI:GọiJ là điểm nằm trên.BI C /sao choIJ ?IQ Chứng minh trung điểmIJ chuyển động đường tròn cố định.

Bài 1.4 Cho tam giác ABC nhọn có∠ACB < ∠ABC < ∠ACB C ∠BAC

2 Lấy điểm D thuộc cạnhBC sao cho∠ADC D∠ACBC∠BAC

2 Tiếp tuyến với đường tròn ngoại tiếp tam giácABC tạiAcắtBC tạiE Phân giác∠AEB cắtAD và cắt.ADE/tạiGF,DF giao AE tạiH:

1 Chứng minh đường trịn đường kínhAE; DF; GH có điểm chung.

2 Trên phân giác ngoài∠BAC và tia AC lần lượt lấy điểm K M sao cho KB DKD DKM, phân giác ngoài∠BAC và tiaABlần lượt lấy điểmL N sao choLC DLD DLN:Đường tròn quaM; N và trung điểmI củaBC cắt BC tạiP (P ÔI) Chng minh rngBM; CN; AP ng quy.

Bài 1.5 Choa1; a2; : : : ; an 1; an(n3), sốai nhận giá trị là0hoặc1 Xétnbộ số sau:

S1 D.a1; a2; : : : ; an 1; an/; S2 D.a2; a3; : : : ; an; a1/; :::

SnD.an; a1; : : : ; an 2; an 1/ Với sốr D.b1; b2; : : : ; bn/, đặt

!.r/Db12n 1Cb22n 2C: : :Cbn20:

Giả sử số!.S1/I!.S2/; : : : ; !.Sn/nhận đúngkgiá trị phân biệt. 1 Chứng minh rằngkjn

n 1

2k 1 j!.Si/với mọii D1; n:

2 Kí hiệuM mlần lượt max của!.S1/; !.S2/; : : : ; !.Sn/ Chứng minh M m

n 1/.2k 1/ 2k 1 :

Bài 1.6 Cho số thực phân biệt là˛1; ˛2; : : : ; ˛16 Với đa thức hệ số thựcP x/, ta đặt V P /DP ˛1/CP ˛2/C: : :CP ˛16/:

Chứng minh tồn đa thứcQ.x/bậc8có hệ sốx8bằng1thỏa mãn đồng thời hai điều kiện:

(191)

Tạp chí Epsilon, Số 08, 04/2016

2 Nhận xét chung.

Về độ khó khó toán lấy tư tưởng toán cao cấp đem xuống (cụ thể đại số tuyến tính) Để giải tốn này, học sinh phải vừa bắt hướng quy nạp, tư tưởng trực giao hóa Gram-Schmidt (hay tư tưởng tương tự) vừa phải có kỹ xử lý kỹ thuật tốt Bài thứ hai 3b với cấu hình rối theo dạng hình chồng hình

Các tốn cịn lại có độ khó ngang nhau, làm hay không tùy thuộc vào gu thí sinh Bài số học tốn có ý tưởng cũ mà lời giải chủ yếu dựa vào bổ để

.am 1; an 1/Da.m;n/ Đó chưa kể dùng đến định lý Zsigmondy toán gần hiển nhiên

Bài tổ hợp số mà kết đốn dễ dàng Bài hình số tốn đẹp vừa sức, mang tính phân loại cao Bài lại tiếp tục toán túy kỹ thuật Chú ý số số khơng khó để trình bày chặt chẽ khó số nhiều

Có lẽ lý nên lý thuyết số học sinh làm bài, rưỡi nhiều thực tế có bạn có số điểm 27 trở lên (tức coi làm trở lên) điểm chuẩn để vào đội tuyển 23.5, tức ++ Đây điều thí sinh phải rút kinh nghiệm Làm ln phải trình bày cho thật

3 Kết đội tuyển

Thông tin đội tuyển công bố vào tuần vừa qua sau:

Họ tên Trường, lớp Điểm

Đào Vũ Quang Lớp 12 THPT chuyên Hà Nội – Amsterdam 30.5

Vũ Xuân Trung Lớp 12 THPT chun Thái Bình 29

Hồng Anh Dũng Lớp 12 THPT chuyên Lam Sơn, Thanh Hoá 27

Phạm Nguyễn Mạnh Lớp 11, PTNK - ĐHQG TP HCM 24,5

Lê Nhật Hoàng Lớp 12, THPT chuyên Lê Quý Đơn, Bình Định 23,5 Vũ Đức Tài Lớp 12, THPT chuyên Lê Hồng Phong, Nam Định 23,5

Kỳ thi IMO năm tổ chức Hong Kong, từ ngày 06/7 đến ngày 16/7/2016 Xin chúc thí sinh có thời gian ơn tập, chuẩn bị thật tốt có kết cao kỳ thi

http://www.msri.org/communications/vmath/special_productions/. bài viết Freiman theorem blog https://www.ima.umn.edu/preprints/Feb92Series/918.pdf http://kskedlaya.org/putnam-archive/2012s.pdf https://gilkalai.wordpress.com/2012/11/25/happy-birthday-ron-aharoni/ http://robertborgersen.info/Presentations/GS-05R-1.pdf http://arxiv.org/abs/0806.1178 (Topology Leonhard Euler Vương quốc Phổ Kaliningrad đặc số Euler – Poincaré Poincaré Riemann Picasso Hồ Xuân Hương Sharygin http://mathforum.org/wagon/ http://blogcuaquangduong.blogspot.com/2016/03/generalizations-of-some-triangle.html

Ngày đăng: 08/02/2021, 07:44

w